You are on page 1of 227

PFRS 15

THEORIES

1. The first step in the process for revenue recognition is to:


A. Determine the transaction price
B. Identify the contract with customers
C. Allocate the transaction price to the separate performance obligations
D. Identify the separate performance obligations in the contract.

Explanation: Revenue Recognition under PFRS 15 has the 5-Step Process.


First, identify contract with customers. Second, identify separate performance
obligations within a contract. Third, determine the transaction price. Fourth,
allocate the transaction price to the separate performance obligation and lastly.
Recognize revenue when (or as) each performance obligation is satisfied.

2. Which of the following is an exception for application of PFRS 15?


A. Lease contracts
B. Insurance Contracts
C. Financial Audit Contracts
D. All of the above
E. A and B

Explanation: An entity shall apply this Standard to all contracts with customers,
except the

following:
(a) lease contracts within the scope of IFRS 16 Leases;
(b) contracts within the scope of IFRS 17 Insurance Contracts;. However, an
entity may choose to apply this Standard to insurance contracts that have as their
primary purpose the provision of services for a fixed fee in accordance with
paragraph 8 of IFRS 17;
(c) financial instruments and other contractual rights or obligations within the
scope of IFRS 9 Financial Instruments, IFRS 10 Consolidated Financial
Statements, IFRS 11 Joint Arrangements, IAS 27 Separate Financial Statements
and IAS 28 Investments in Associates and Joint Ventures;
and
(d) non-monetary exchanges between entities in the same line of business to
facilitate sales to customers or potential
customers. [IFRS 15.5]

3. Entity shall recognise revenue to depict the transfer of promised goods or


services to customers in the _ amount that reflects the consideration to which
the entity expects to be entitled in exchange for those goods or services.
A. Net
B. Residual
C. Gross
D. Cumulative
Explanation: The core principle of this Standard is that an entity shall recognise
revenue to depict the transfer of promised goods or services to customers in an
amount that reflects the consideration to which the entity expects to be entitled in
exchange for those goods or services. [IFRS 15.2

4. A contract does not exist if …


A. The contract was not approved in a written form
B. The parties of the contract have reached unanimous consent regarding
termination of the contract
C. The contract has no fixed duration and can be terminated or modified by
either party at any time
D. Each party to the contract has the unilateral enforceable right to
terminate a wholly unperformed contract without compensating the
other party.

Explanation: For the purpose of applying this Standard, a contract does not exist
if each party to the contract has the unilateral enforceable right to terminate a
wholly unperformed contract without compensating the other party (or parties).
[IFRS 15.12]

5. A contract is wholly unperformed if …


A. The entity has not yet transferred any promised goods or services to the customer.
B. The entity has not yet entitled to receive any consideration in exchange for
promised goods and services.
C. The entity is not yet entitled to receive any consideration in exchange for
promised goods and services
D. All of the above
E. A and B

Explanation: A contract is wholly unperformed if both of the following criteria are met:
(a) the entity has not yet transferred any promised goods or services to the
customer; and
(b) the entity has not yet received, and is not yet entitled to receive, any
consideration in exchange for promised goods or
services. [IFRS 15.12]

6. Stay man Associates has sold a good to a buyer and wants to recognize
revenue. Which of the following is an indicator that control of a good has passed
from Stay man to the buyer?
A. Buyer has scheduled delivery.
B. Buyer has a strong credit history, such that bad debts are reasonably estimable.
C. Buyer has not scheduled delivery.
D. Buyer has assumed the risk and rewards of ownership.

Explanation: One of the indicators that control has passed from a seller to a
buyer is if the buyer has assumed the risk and rewards of ownership.
7. Which of the following is not an indicator that the customer is likely to have
control over a good?
A. Asset warehoused by seller-affiliated third party
B. Accepted the asset
C. Legal title to the asset
D. Physical possession of the asset

Explanation: Asset warehoused by seller-affiliated third party" is an indicator that


control has not passed.

8. Which of the following is not an indicator that revenue can be recognized over time?
A. The seller is enhancing an asset that the buyer controls as the service is performed.
B. The customer consumes the benefit of the seller's work as the seller
performs the service.
C. The seller is creating an asset that has an alternative use for the seller,
and the seller can receive payment for its progress even if the customer
cancels the contract.
D. None of the other answers is correct.

Explanation: An indicator that revenue recognition over time is appropriate is that


the seller is creating an asset that does not have an alternative use to the seller,
and the seller can receive payment for-its progress even if the customer cancels
the contract.

9. Which of the following is not an approach for estimating stand-alone selling prices?
A. Adjusted market assessment approach
B. Expected cost plus margin approach
C. Residual approach
D. Fair market appraisal approach

Explanation: The other three answers are the three methods indicated for
assessing stand-alone selling prices.

10. Which of the following is not something that revenue recognition disclosures
typically should help investors to understand?
A. Timing of revenue and cash flows
B. Outstanding performance obligations
C. Significant judgments used to estimate transaction prices
D. Significant fluctuations in long-term debt necessary to increase revenue
in the future

Explanation: Long-term debt fluctuations to finance future revenue increases are


not specific to revenue recognition practices, so are least likely to appear with
revenue recognition disclosures.
PROBLEMS

Items 1 to 3 are based on the following information:


Other competing sellers in the same region charge an average of P250 for a set
of goggles and P750 for the lessons, if sold separately. New Company usually
sells at a 5% discount compared to other shops, since it is a bit farther away from
the ocean.
1. What would be New's stand-alone selling price of the goggles and the lessons,
based on adjusted market assessment approach?
a.P2
37.5
0
b.P2
50.0
0 c.
P71
2.50
d. P750.00

Solution: Under the adjusted market assessment approach, New would base its
estimate of the stand-alone selling price of the goggles and the lessons on the
prices charged by other sellers for those goods, adjusted as necessary. Because
New typically sells at a 5% discount, it would estimate the stand-alone selling
price of the goggles and the lessons to be P250x 95% = P237.50 and P750x 95%
= P712.50, respectively.

2. Typically, New incurs P375 on compensation and other costs to provide the
private lessons, and earns an average of 40% profit over cost on service
offerings. Assuming that the diving equipment and the certification lessons are
separate performance obligations, estimate the stand-alone selling price of the
certified lessons based on the expected cost plus margin approach.
a. P150.00
b. P375.00
c. P525.00
d. P700.00

Solution: Under the expected cost plus margin approach, New would base its
estimate of the stand-alone selling price of the private lessons on the P375 cost it
incurs to provide the services, plus its normal margin of 40% x P375 = P150.
Therefore, New would estimate the stand-alone selling price of the private lessons
to be P375 + P150 = P525.

3. Typically, if New were to sell the equipment only, it would ask for 2,000.
Assuming that the diving equipment and the certification lessons are separate
performance obligations, estimate the stand-alone selling price of the lessons
based on the residual approach.
a. 500
b. 2000
c
.
2
5
0
0
d
.
3
0
0
0

Solution: Under the residual approach, New would base its estimate of the
stand-alone selling price of the private lessons on the total selling price of the
package (P2,500)
minus the observable stand-alone selling price of equipment (P2,000). Therefore,
New would estimate the stand-alone price of the lessons to be P2, 500-P2, 000 =
P500.

4. CompuLand Center sells a full assortment of computer parts, including


motherboards, video cards, and cables, and also offers complementary computer
assembly services. The assembly service is offered by other vendors for P100 on
average, and Compuland typically charges approximately 20% more than other
vendors for similar services on a stand-alone basis. Estimate the stand-alone
selling price of the assembly service using the adjusted market assessment
approach.
a. Zero
b. P100
c. P120
d. P200

Solution: Under the adjusted market assessment approach, CompuLand would


base its estimate of the stand-alone selling price of the assembly service on the
prices charged by other vendors for the same service, adjusted as necessary.
Because CompuLand typically charges 20% more than competitors, it would
estimate the stand-alone price of the assembly service to be P100x 120% =
P120.

5. CompuValue Center sells a full assortment of computer parts, including


motherboards, video cards, and cables, and also offers complementary computer
assembly services. CompuValue estimates that it incurs P50 in labor and
materials on average fo complete one assembly order, with an average of 757%
profit based on cost. Assuming that computer parts and assembly service are
separate performance obligations, estimate the stand- alone selling price of the
assembly service based on the expected cost plus margin approach.
a. Zero
b. P37.50
c. P50.00
d. P87.50

Solution: Under the expected cost plus margin approach, ComputValue would
base its estimate of the stand-alone selling price of the assembly service on the
P50 cost it incurs, plus its normal margin of 75% x P50= P37.50. Therefore,
CompuValue would estimate the stand-alone selling price of the assembly service
to be Ps0 P37.50 P87.50.

6. ET received an order to supply a customer with 10,000 units of Product A


every month for two years. The customer had negotiated a low price of 200 per
1,000 units and agreed to pay 12,000 in advance every six months. The customer
made the first payment on July 01, 20x6 and ET supplied the goods each month
from that date. ET’s year end is September 30. In addition to the effect of cash
received, what is the effect of this order on ET’s financial statement for the year
ended September 30,20x6, in accordance with PFRS 15 Revenue from Contract
with Customers?
Revenue Statement of financial position
a. 6,000 36,000 trade receivable
b. 6,000 6,000 current liability
c. 12,000 36,000 trade receivable
d. 12,000 No effect

Solution: In the end of the year, the revenue that should be recognized is 6,000
and the other 6,000 will be recognized as unearned revenue which is classified as
current liability in the Statement of Financial Position. See the entries below.
July 1, 20x6
Cash 12,000
Unearned Revenue 12,000

September 30, 20x6 (Adjusting Entry)


Unearned Revenue 6,000 (2,000 x 3
months)
Revenue 6,000

7. RGB signed a contract to provide office services to PQ for one year from 1
October 20x6 for 500 per month. The contract required PQ to make a single
payment to RGB for all 12 months at the beginning of the contract. RGB received
6,000 on October 1, 20x6. What amount of revenue should RGB recognize in its
statement of profit or loss for the year ended March 31, 20x7?
a. Nil
b. 300
c. 3,000 profit
d. 6,000 profit

Solution: Same approach as # 6.

Mercedes Consultants provided Benz Construction with assistance in


implementing various cost-savings initiatives. Mercedes’ contract specifies that it
will receive a flat fee of 50,000 and additional 20,000 if Benz reaches a pre
specified target amount of cost- savings. Mercedes estimates that there is a 20%
chance that Benz will achieve the cost-saving target.

8. Assuming Mercedes uses the expected value as its estimate variable


consideration, calculate the transaction price.
a. 14,000
b. 40,000
c. 50,000
d. 54,000

Solution: Flat fee 50,000


Add: Additional 4,000 (20,000 x 20%)
Total Consideration 54,000

9. Assume Mercedes uses the expected value as its estimate of variable


consideration, but is very uncertain of that estimate due to a lack of experience
with similar consulting arrangements. Calculate the transaction price.
a. 14,000
b. 40,000
c. 50,000
d. 54,000

Solution: The answer is 50,000. The estimation for variable consideration is not
recognized because of the uncertainties of the estimate due to a lack of
experience.

10. On July 1, 2020, RGB Company handed over to a client a new computer
system. The contract price for the supply of the system and after-sales support
for 12 months was 1,000,000 and Cindy Company estimated the cost of the
after- sales support at 120,000. It normally marks up such cost by 50% when
tendering for support contracts. The Revenue to be recognized in its financial
statement year ended 2019 is?
a. 710,000
b. 810,000
c. 910,000
d. 1,120,000

Solution: Stand-alone price (without after-sales support) 880,000 (1M – 120,000)


After-sales support 120,000
Mark up x 50%
Total 60,000
Expired Portion 6/12 30,000
Revenue to be recognized 910,000
PARTNERSHIP FORMATION

THEORIES

1. Each of the following statement/s is/are true, except:

I. One of the features of a general partnership is unlimited liability which means


all the partners in a partnership are responsible for liabilities and have all the
authority to act on his behalf.

II. According to the standard, a partner’s capital account is prohibited to have a


debit balance because other partners are required to suffice such deficiency.

III. There are three types of partnership: General partnership, Limited partnership
and Joint Control partnership.

IV. Partnership is an association of two or more persons who contribute money,


property, or industry to a common fund with the intention of dividing the profits
among themselves.

V. One of the major disadvantages of partnership is the pooling of capital and


other resources with so much complexities compared to other forms of business.

A. I, II, III only


B. II, III, V only
C. III, V only
D. I, IV, V only

2. Partner’s equity is increased by _ , while it is decreased by _ .

A. Investment of non-monetary assets; withdrawal of cash or other assets.


B. Additional investments; withdrawal of noncurrent assets.
C. Investment of non-monetary assets; withdrawal of noncurrent assets.
D. Additional investments; withdrawal of cash or other assets.

3. Statement 1: Liabilities assumed by the partnership should be valued at the


present value of the remaining cash flows.

Statement 2: The formula used to divide profits and losses in a partnership


should only be through acquisition method because pooling of interest method is
prohibited.

Statement 3: Bonuses are sometimes used as means of providing additional


investment to partners who have provided services for the partnership.

A. All statements are false.


B. Only one statement is false.
C. Only one statement is true.
D. All statements are true.

4. Statement 1: Partnership law provides that partnership has a juridical


personality separate and distinct from that of each partner.

Statement 2: In accounting for the formation of a partnership, cash under


receivership is recorded at its face value.

Statement 3: Generally, the capital balance is determined by the proportionate


share of each partner’s capital contribution.

A. All statements are false.


B. Only one statement is false.
C. Only one statement is true.
D. All statements are true.

5. Statement 1: Propriety Theory looks at the entity through the eyes of the
owner, and the Entity Theory views business as a separate and distinct entity.
Statement 2: Partnership law provides that partnership has a judicial personality
separate and distinct from that of each partner, therefore, acts beyond the
normal scope of business never bind the partnership.

A. Statement 1 is true, statement 2 is true


B. Statement 1 is true, statement 2 is false
C. Statement 1 is false, statement 2 is true
D. Statement 1 is false, statement 2 is false

6. Which of the following is true, if the partnership has only internal reporting needs?
A. The partnership must use GAAP accounting methods
B. Financial reports should conform from those required by GAAP
C. Accounting and financial reporting should meet those internal information
needs of the partners
D. None of the above

7. Which of the following is false regarding a partner’s capital interest in the


partnership?
A. Partners may agree to have capital interest ratio to be different from
the profit and loss interest ratio.
B. A partner’s capital interest is a claim against net assets of the
partnership as shown by the balance in the partner’s capital accounts.
C. The capital interest ratio will determine how the partner’s capital
interest will increase or decrease as a result of subsequent
operations
D. None of the above.

8. Statement 1: The ease of ownership transferability is one of the features and


advantages of forming a partnership.

Statement 2: When a partner receives cash from the partnership, the


transaction is recorded as a reduction to the partner’s capital account.
A. Only statement 1 is true.
B. Only statement 2 is true.
C. Both statements are true.
D. Neither of the statements is true.

9. Choose the best answer.

A partnership records a partner’s noncash investments at

A. the value given by an independent appraiser.


B. the book value of the assets invested.
C. the fair value of the assets invested.
D. the value agreed by the partners.

10. Corporations and Partnerships share some similar characteristics which are?

Formation/Agreement Separate Legal Taxable

is required to be in writing Personality Entity

A. Yes Yes No
B. Yes No Yes
C. No No No
D. No Yes No
PROBLEMS

1. On December 1, 2020, Jibs and Ran formed a partnership, agreeing to share


for profits or losses in the ratio of 2:3, respectively. Jibs invested a parcel of land
that cost him P150,000 and some intangible assets costing P40,000. Ran
invested P180,000 cash. On the same date, two hours after formation of the
partnership, the land was sold for P200,000, and intangible assets were agreed
to be written-off. How much should be the capital balance of Jibs right after the
formation?

A. P180,000
B. P150,000
C. P200,000
D. P240,000

Use the following information for the next two questions:

On January 1, 2020, Hope, Elizabeth, Kathryn and Chandria agreed to form a


partnership contributing their respective assets and equities subject to
adjustments. On the same date, the following were provided:

Hope Elizabeth Kathryn Chandria

Cash P47,000 P33,880 P60,000 P55,565

Accounts
13,980 15,670 37,000 28,900
Receivable

Inventories 88,995 73,950 99,000 78,000

Land 164,050 200,000 230,000 170,000

Building 90,000 89,920 124,000 98,920

Furnitures
65,870 75,100 50,905 55,200
and
Fixtures
Intangible Assets 16,954 10,200 45,000 33,005

Accrued
98,335 86,670 96,000 73,000
Expenses

Long-term
180,300 178,605 208,000 199,005
Liabilities
Additional information:

 Mr. Blues, customer of Hope, is currently facing a bankruptcy. It is


determined that out of P2,335 payable on account, only P730 is
recoverable.
 The following amount of inventories are worthless:
o Hope – P4,000
o Elizabeth – P3,560
o Kathryn – P6,600
o Chandria – P805
 The land of Kathryn was sold for 250,000, three hours after formation
of the partnership.
 Elizabeth’s long-term debt has a present value of P175,000.
 Intangible assets are to be written off.

2. What are the adjusted capital balances of Hope, Elizabeth, Kathryn, and Chandria, ?

Hope Elizabeth Kathryn Chandria

A. P185,655 P223,290 P213,775 P310,305

B. P180,650 P310,300 P220,200 P213,775

C. P185,655 P223,290 P310,305 P213,775

D. P185,655 P223,290 P310,350 P213, 757

3. On the partnership books, total assets contributed by each partner should be


valued at:

Hope Elizabeth Kathryn Chandria

A. P474,290 P464,960 P641,350 P485,780

B. P464,290 P484,960 P614,305 P485,708

C. P464,290 P484,960 P640,005 P485,780

D. P464,290 P484,960 P614,305 P485,780


4. As of July 1, 2020, DALMI and DOSAN decided to form a partnership. Their

balance sheets on this date are:

Chulsan Dosan

Cash 18,500 22,500

Accounts Receivable 945,000 575,000

Land 2,000,000 1,575,000

Merchandise Inventory – 225,500

Machinery and Equipment 200,000 315,750

Total 3,163,500 2,713,250

Accounts Payable 363,500 420,000

Bonds Payable 1,000,000 498,250

Chulsan, Capital 1,800,000 –

Dosan, Capital – 1,795,000

Total 3,163,500 2,713,250

The partners agreed that the Machinery and Equipment of Chulsan is

underdepreciated by P30,000, and that of Dosan by P60,000. Allowance for

doubtful accounts is to be set


up amounting to P150,000 for Chulsan and P75,000 for Dosan. The partnership

agreement provides for a profit and loss ratio and capital interest of 60% to

Chulsan and 40% to Dosan. How much cash must Chulsan invest to bring the

partner’s capital balances proportionate to their profit and loss ratio?

a. P870,000 c. P2,700,000

b. P1,106,667 d. P2,692,500

Use the following information for the next two questions:

Jackie admits Chan as a partner in business. Just before the admission of

Chan, accounts in the ledger of Jackie shows the following balances:

Cash 12,850

Accounts Receivable 20,250

Merchandise Inventory 26,750

Accounts Payable 15,000

Jackie, Capital 44,850

It is agreed that for purposes of establishing Jackie’s interest, the following

adjustments shall be made:

a. Prepaid salary expenses of P1,200 and accrued rent expense of P1,600


are to be recognized
b. An allowance for bad debts of 5% of accounts receivable is to be recognized.
c. The merchandise inventory is to be valued at
P35,000. Chan is to invest cash to obtain 2/5 interest in
the partnership.

Compute for:

5. Jackie’s adjusted capital before the admission of Chan

a. P55,637.5 c. P51,637.5

b. P51,687.5 d. P61,687.5

6. The amount of cash to be invested by Chan

a. P36,458.3 c. P34,458.3

b. P37,450.3 d. P30,458.3

Use the following information for the next two questions:

The business assets of Lira and Mira appear below:

Lira Mira

Cash P15,500 25,604

Accounts Receivable 239,036 571,140

Inventories 124,535 263,352

Land 607,500 –
Building – 431,517

Furniture and Fixture 54,845 38,039

Other Assets 6,500 6,850

Total 1,047,916 1,336,502

Accounts Payable 183,440 246,900

Notes Payable 204,500 348,250

Lira, Capital 659,976 –

Mira, Capital – 741,349

Total 1,047,916 1,336,502

Lira and Mira agreed to form a partnership by contributing their respective

assets and equities subject to the following adjustments:

a. Inventories of P6,750 and P7,950 are worthless in Lira’s and Mira’s


respective books
b. Accounts Receivable of P18,500 in Lira’s books and P32,500 in Mira’s
books are uncollectible.
c. Other Assets of P6,500 and P6,850 in Lira’s and Mira’s respective
books are written off.
The capital account of Lira after the adjustments will be:
7. The capital account of Lira after the adjustments will be:

a. P638,226 c. P626,226

b. P628,526 d. P628,226

8. The capital account of Mira after the adjustments will be:

a. P696,049 c. P640,049

b. P694,049 d. P660,600

9. Anne and Margot formed a partnership with each partner contributing the

following items:

Anne Margot

Cash………………………………………… P 120,000 P 60,000

Building-Cost to Anne……………………. 420,000

-Fair Value………………………... 520,000

Inventory-Cost to Margot ………………… 300,000

-Fair Value ……………………… 420,000

Mortgage Payable …………………… 160,000

Account Payable ……………………….. 80,000


Assume that for tax purposes Anne and Margot agree to share equally in the

liabilities assumed by the Anne and Margot partnership. What is the balance in

each partner’s capital account for financial accounting purposes?

Anne Margot Anne Margot

A. P 480,000 P 400,000 C. P 380,000 P 300,000

B. P 470,000 P 410,000 D. P 620,000 P 440,000

Use the following information for the next two questions:

On October 6, 2020, Cindy and Kateu formed a partnership with each contributing the

following assets:

Cindy Kateu

Cash P 56,220 P 64,000

Machinery 48,000 98,000

Building 268,000

Furniture and Fixtures 14,000

The building is subject to a mortgage loan of P110,000 which is to be assumed

by the partnership agreement provided that Cindy and Kateu’s share in profits

and losses are 28% and 72%, respectively.

10. On October 6, 2020, the capital account of Kateu would show a

balance of: A. P 339,000 C. P 410,000

B. P 345,000 D. P 320,000
11. Assuming that the partners agreed to bring their respective capital in

proportion to their respective profit or loss ratio, and using Kateu’s capital as a

base, how much cash is to be invested by Cindy?

A. P 6,224 C. P 7,234

B. P 7,244 D. P 7,254
ANSWE

R KEY

THEORI

ES

1. B

I. True.

II. False. Partner’s capital account may have a debit balance called deficit.

III. False. There are only two types of partnership: General partnership and
Limited partnership.

IV. True.

V. False. One of the major advantages of partnership is the pooling of capital and
other resources without complexities and formalities of a corporation.

2. D

Partner’s equity is increased by additional investments, while it is


decreased by withdrawal of cash or other assets.

3. C

Statement 1 – True.

Statement 2 – False. The formula used to divide profits and losses in a


partnership is determined through negotiations among partners.

Statement 3 – False. Bonuses are sometimes used as means of providing


additional compensation (not investment) to partners who have provided
services for the partnership.

4. B

Statement 1: True.

Statement 2: False. In accounting for the formation of a partnership, cash under


receivership is recorded at its estimated recoverable amount.

Statement 3: True
5. B

Explanation: Generally, acts beyond the normal scope of the business do not

bind the partnership unless specific authority has been given to the partner.

6. C

Explanation: If a partnership has only internal reporting needs, then the

accounting and financial reporting should meet those internal information needs

of the partners. In this case, the partnership may use non-GAAP accounting

methods and have financial reports in a format different from those required

under GAAP.

7. C

Explanation: interest in profit or loss and not the partner’s capital interest ratio

will determine how the partner’s capital interest will increase or decrease as a

result of subsequent operations.

8. D

I. The ease of transferability of ownership is a feature and advantage in

forming a corporation.

II. Cash received by partners from the partnership is recorded in the books as

a Loan Receivable from Partners.


9. D

Noncash investments are measured at its agreed value which is normally its fair

value. However, in case of conflict between agreed value and fair value, the

agreed value prevails.

10. D

i. Partnership agreements are preferably done in writing but oral agreement can

be equally acceptable. On the other hand, corporations are required to create

articles of incorporation upon formation which is also needed in registering to

the SEC.

ii. The law provides that Partnerships and Corporations have a juridical

personality separate and distinct from its partners and stockholders.

iii. A partnership is not a taxable entity, only the partners are taxed individually.

A corporation by its own name is a taxable entity.


PROBLEMS

1. C

In the formation of a partnership, one or more partners may contribute noncash


assets. If such asset will be retained at cost, it would then be inequitable to any
partners investing appreciated property. Thus, the noncash asset contributed by
any partner should be recorded based on its fair value. In this case, the fair value
of the land is equal to its sales price on the date of sale, P200,000.

Intangible asset is not included in the contributed capital of Jibs because the
partners have agreed to write it off.

2. C

Hope Elizabeth Kathryn Chandria

Cash P47,000 P33,880 P60,000 P55,565

Accounts
13,980 15,670 37,000 28,900
Receivable

Inventories 88,995 73,950 99,000 78,000

Land 164,050 200,000 230,000 170,000

Building 90,000 89,920 124,000 98,920

Furnitures
65,870 75,100 50,905 55,200
and Fixtures

Intangible Assets 16,954 10,200 45,000 33,005

Accrued
(98,335) (86,670) (96,000) (73,000)
Expenses

Long-term
(180,300) (178,605) (208,000) (199,005)
Liabilities

Unadjusted
208,214 233,445 341,905 247,585
Capital

Doubtful Accs.
(1,605) - - -
(2,335-730)

Invty write-down (4,000) (3,560) (6,600) (805)


Gain on sale
of Land - - 20,000 -

Decrease in
- 3,605 - -
Long-term debt

Write-off of
(16,954) (10,200) (45,000) (33,005)
Intangibles

Adjusted capital
Balances P185,655 P223,290 P310,305 P213,775

3. D

Hope Elizabeth Kathryn Chandria

Cash P47,000 P33,880 P310,000* P55,565

Accounts
12,375** 15,670 37,000 28,900
Receivable

Inventories*** 84,995 70,390 92,400 77,195

Land 164,050 200,000 - 170,000

Building 90,000 89,920 124,000 98,920

Furnitures
65,870 75,100 50,905 55,200
and Fixtures

Intangible Assets - - - -

Total Assets P464,290 P484,960 P614,305 P485,780

*Cash balance of Kathryn before formation P60,000


Proceeds from sale of land 250,000
Cash contributed by Kathryn after formation P310,000

**Accounts Receivable P13,980


Less: Doubtful accounts 1,605
(2,335-730) Accounts
Receivable, net P12,375

*** 88,995 – 4,000 = 84,995


73,950 – 3,560 = 70,390
99,000 – 6,600 = 92,400
78,000 – 805 = 77,195

4. A

Computation of adjusted contributed capital:

Chulsan Dosan

Unadjusted capital P1,800,000 P1,795,000

Add (deduct) adjustments:

Accumulated Depreciation (30,000) (60,000)

Allowance for Doubtful Accounts (150,000) (75,000)

Adjusted contributed capital P1,620,000 P1,660,000

Dosan’s adjusted contributed capital P1,660,000

Divided by: Dosan’s P&L percentage 40%

Total Agreed Capital 4,150,000

Multiplied by: Chulsan’s P&L percentage 60%

Chulsan’s Agreed Capital 2,490,000

Less: Chulsan’s adjusted contributed capital 1,620,000


Additional cash to be contributed by Chulsan P870,000

5. B

Unadjusted capital of Jackie P44,850

Add (deduct) adjustments:

Allowance for Doubtful (1,012.5)

Accounts (5% x 20,250)

Increase in Merchandise 8,250

Inventory (35,000 – 26,750)

Prepaid Salary 1,200

Accrued Rent Expense (1,600)

Adjusted capital balance of P51,687.5

Jackie

6. C

Adjusted Capital Balance 51,687.5

of Jackie

Divided by: Capital 3/5


interest of Jackie

Total capital balance of 86,145.8

the partnership

Less: Adjusted Capital (51,687.5)

balance of Jackie

Capital balance of Chan P34,458.3

(cash to be invested)

7 and 8. (D
and B)
Solution:
Lira Mira
Unadjusted capital P659,976 P741,349
Add (deduct) adjustments:
Uncollectible Receivables (18,500) (32,500)
Write-off of inventories (6,750) (7,950)
Write-off of other assets (6,500) (6,850)
Adjusted contributed capital P628,226 P694,049
9. A

Anne Margot

Cash P 120,000 P 60,000

Building- Fair Value 520,000

Inventory-Fair Value 420,000

Mortgage Payable (160,000)

Account Payable 80,000

Capital P 480,000 P 400,000


10. D

Total Assets:

Cash P 64,000

Machinery 98,000

Building 268,000

Less: Liabilities assumed 110,000

Net assets (Capital contributed by Kateu) P 320,000

11. A

Kateu’s Capital (see number 2) P

320,000 Divide by Kateu’s P&L percentage 72%

Total Partnership Capital P 444,444

Required Capital of Cindy 28%

P 124,444

Less: Assets contributed

Cash P 56,220

Machinery and Equipment 48,000

Furniture and Fixtures 14,000 118,220

Cash to be invested by Cindy P 6,224


PARTNERSHIP DISSOLUTION

THEORIES

1. of partnership changes the existing relationship between partners

but the firm may continue its business as before.

a. Liquidation c. Dissolution
b. Formation d. Birth

Dissolution of partnership changes the existing relationship between partners

but the firm may continue its business as before. It is the change in the relation

of the partners caused by any partner being disassociated from the business.

Partnership dissolution does not necessarily terminate the business.

2. The dissolution of partnership may take place in any of the following ways.

Which of the following is not included?

a. Change in existing profit sharing ratio among partners


b. Admission of a new partner
c. Insolvency of a partner
d. Accomplishment of the purpose of the partnership

The dissolution of partnership may take place in any of the following ways:

(1) Change in existing profit sharing ratio among partners;

(2) Admission of a new partner;

(3) Retirement of a partner;


(4) Death of a partner;

(5) Insolvency of a partner;

(6) Completion of the venture, if partnership is formed for that; and

(7) Expiry of the period of partnership, if partnership is for a specific period of

time; Accomplishment of the purpose of the partnership causes partnership

liquidation.

3. Which of the following statement/s is/are true?

Statement 1: Dissolution of a partnership firm may take place without the

intervention of court or by the order of a court.

Statement 2: It may be noted that dissolution of the firm necessarily brings in

dissolution of the partnership.

a. Statement 1 only c. Both statements are correct.

b. Statement 2 only d. Neither of the two statements is correct.

Dissolution of a partnership firm may take place without the intervention of court

or by the order of a court. It may be noted that dissolution of the firm necessarily

brings in dissolution of the partnership. Dissolution of a firm takes place in any of

the following ways:

1. Dissolution by Agreement

2. Compulsory Dissolution
3. On the happening of certain contingencies

4. Dissolution by Notice

5. Dissolution by Court

4. A new partner may be admitted when he purchases part or all of the interest

of one or more of the existing partners. Which of the following is false?

a. This transaction is personal between and among the partners.

b. There is no new capital account established for the new partner but there

is a corresponding decrease on the capital account(s) of the selling

partner.

c. Any consideration paid or received is not recorded in the partnership books.

d. No gain or loss is recognized in the partnership books.

A new partner may be admitted when he purchases part or all of the interest of

one or more of the existing partners. Note that this transaction is personal

between and among the partners and as such, any consideration paid or

received is not recorded in the partnership books. A new capital account

will be established for the new partner and a corresponding decrease is made

on the capital account(s) of the selling partner. No gain or loss is recognized in

the partnership books.

5. When the outgoing partner’s interest is settled at an amount greater than or

less than the value of his interest, the _ method is used.

a. bonus c. exact
b. goodwill d. acquisition
Two scenarios may happen when a new partner invests in partnership:
1. The new partner’s capital account is credited at an amount equal to the fair

value of his investment; or

2. The new partner’s capital account is credited at an amount greater than or

less than the fair value of his investment. This is accounted for under the bonus

method.

When the outgoing partner’s interest is settled at an amount greater than or less

than the value of his interest, the bonus method is used. Under the bonus

method, any excess/deficiency in the payment is accounted for as a

deduction/addition to the remaining partner’s capital accounts.

6. A partnership dissolution differs from a liquidation in that

a. payments are made to creditors before partners receive value.

b. periodic payments to partners are made when cash becomes available.

c. a partner withdraws from the business and the enterprise continues to

function.

d. full payment is made to all outside creditors before remaining cash is

distributed to partners in a final lump sum payment.

Based on the availability of funds, the sale proceeds are distributed in the

following order: creditors, debt security owners, preferred shareholders and lastly

common shareholders. In the case of dissolution, shareholders or owners do not

receive any proceeds, whereas in case of liquidation, the shareholders or owners

receive proceeds
from the company if there are enough assets, after paying the debts owed to the

creditors of the company.

7. Statement 1: A partnership may be dissolved without being liquidated.

Statement 2: Liquidation is always preceded by dissolution.

A. Both statements are true


B. Only statement 1 is true
C. Only statement 2 is true
D. Both statements are false

Dissolution should be distinguished from liquidation of the partnership. A

partnership is said to be liquidated when the business is terminated, a

partnership may be dissolved without being terminated but liquidation is always

preceded by dissolution.

8. When a partnership dissolves, the first step in the dissolution process is to _.

A. allocate the gain or loss on sale based on income sharing ratio


B. pay off liabilities
C. sell noncash assets
D. divide the remaining cash among the partners

The liquidation or dissolution of a partnership is synonymous with closing the


business. four steps remain in the accounting for the liquidation, each requiring an
accounting entry. They are:

 Step 1: Sell noncash assets for cash and recognize a gain or loss on
realization. Realization is the sale of noncash assets for cash.
 Step 2: Allocate the gain or loss from realization to the partners based
on their income ratios.
 Step 3: Pay partnership liabilities in cash.
 Step 4: Distribute any remaining cash to the partners on the basis of their
capital balances.

9. All the assets of the firm are _ _ and all outsiders’ liabilities and partners’
loan and partners capitals are at the time of dissolution of firm.

a. Disposed Off, Paid


b. Paid, Disposed Off
c. Acquired , Paid
d. Disposed Off, Acquired

Dissolution is when a partnership firm ceases operations and its assets are
disposed of. A firm may be dissolved in accordance with a contract between the
partners, by law, on happening of certain contingencies or by court. Or where the
partnership is at will it may be dissolved by any partner giving notice in writing to
all other partners of his intention to dissolve the partnership or firm.

10. When a partnership dissolves, the last step in the dissolution process is to
.

A. allocate the gain or loss on sale based on income sharing ratio


B. pay off liabilities
C. sell noncash assets
D. divide the remaining cash among the partners

The liquidation or dissolution of a partnership is synonymous with closing the


business. four steps remain in the accounting for the liquidation, each requiring an
accounting entry. They are:

 Step 1: Sell noncash assets for cash and recognize a gain or loss on
realization. Realization is the sale of noncash assets for cash.
 Step 2: Allocate the gain or loss from realization to the partners based
on their income ratios.
 Step 3: Pay partnership liabilities in cash.
 Step 4: Distribute any remaining cash to the partners on the basis of their
capital balances.
PROBLEMS

1. On January 31, 2020, partners of Jib, Ran and Tomin, LLP, had the following
loan and capital account balances (after closing entries of January):
Loan Receivable from Jib 20,000 dr
Loan Payable to Tomin 60,000 cr
Jib, Capital 30,000 cr
Ran, Capital 120,000 cr
Tomin, Capital 70,000 cr

The partnership’s income sharing ratio was Jib, 50%; Ran, 20%; and Tomin,
30%. On January 31, 2020, Dog was admitted to the partnership for a 20%
interest in total capital of the partnership in exchange for an investment of
40,000 cash. Prior to Dog’s admission, the existing partners agreed to increase
the carrying amount of the partnership inventories to current fair value, a 60,000
increase. The capital account to be credited to Dog:

a. 60,000 c. 52,000

b. 40,000 d. 46,000

2. Jibjib, Ranran and Tomins are partners who share profits and losses 30%,
25% and 45%, respectively. Jibjib informed Ranran and Tomins that he is
withdrawing from the partnership. The partners’ capital accounts at that fate of
Jibjib’s withdrawal are 150,000, 135,000, and 225,000, respectively. The
partnership agreement states that the goodwill, if any, of the withdrawing
partner will be recognize of all the partners immediately prior to the withdrawal
of any partner. In this instance, the partners determine that the goodwill
associated with Jibjib is 22,500. Assuming that Jibjib’s equity is purchased by
a new partner (SiCrush) approve by Ranran and Tomins, what is the amount
of SiCrush initial account?
a. 150,000

b. 170,000

c. 172,500

d. The amount cannot be determined because the amount of SiCrush paid for
Jibjib’s equity is not known.

3. In the JR partnership, Jib’s capital is 140,000 and Rans’ is 40,000 and they
share in a 3:1 ratio, respectively. They decide to admit Nikki to the
partnership. Jib and Ran
agree that some of the inventory is obsolete. The inventory account is
decreased before Nikki is admitted. Nikki invest 40,000 for a one-fifth interest.
What is the amount of inventory written down?
a. 4,000 c. 15,000
b. 10,000 d. 20,000

4. The partnership capital of Asta, Yuno, and Secre on June 1, 2020 are
presented below with their respective profit and loss ratios:

Asta 139,200 1/2


Yuno 208,800 1/3
Secre 96,000 1/6
On June 1, 2020 Noelle is admitted to the partnership when Noelle purchased for
132,000, a proportionate interest from Asta and Secre in the net assets and
profits of the partnership. As a result of a transaction Noelle acquired one-fifth
interest in the net assets and profits of the firm. What is the combined gain
realized by Asta and Secre upon the sale of a portion of their interest in the
partnership to Noelle?
a. 0 c. 62,400
b. 43,200 d. 82,000

5. Naruto, Sasuke, and Nagato are partners in a manufacturing concern.


Relevant data regarding income-sharing relationship and capital balances are
as follows:

Partner Capital Balance Income Share


Naruto 150,000 35%
Sasuke 100,000 30%
Nagato 200,000 35%
Sasuke decides to be a rouge partner, so he retires and receives 145,000 in cash
from the partnership.
If the bonus method is used to account for the retirement, Naruto’s capital balance
subsequent to Sasuke’s retirement will be:
a. 105,000 c. 134,250
b. 127,500 d. 150,000

6. Eren and Mikasa formed a partnership and agreed to divide initial capital
equally, even though Eren contributed 125,000 and Mikasa contributed
100,000 in
identifiable assets. Under the bonus approach to adjust the capital accounts,
Mikasa’s unidentifiable assets should be debited for:
a. 69,000 c. 0
b. 80,000 d. 21,000

7. Lexi Lore, a partner in an accounting firm, decided to withdraw from the


partnership, Lore’s share of the partnership profits and losses was 20%. Upon
withdrawing from the partnership, she was paid 69,000 in final settlement for
her interest. The total of the partners’ capital accounts before recognition of
partnership goodwill prior to Lore’s withdrawal was 300,000. After her
withdrawal the remaining partners’ capital accounts, excluding their share of
goodwill, totaled 269,000. The total goodwill of the firm was:
a. 300,000 c. 178,000
b. 190,000 d. 154,000

8. RAM, ULIE, and OCCO have been partners sharing net incomes and losses in
a 3:5:2 ratio. On October 31, the date OCCO retires from the partnership, the
equities of the partners are RAM, P130,000; ULIE P200,000; and OCCO,
P50,000. The estimated profit to October 31 is P100,000 and the partners
have appropriately decided to adjust the understated assets to fair value by
P25,000. What is the journal entry to record OCCO’s retirement assuming
Occo is paid 85,000 in partnership cash for his equity.
a. Occo, Capital 75,000
Cash 70,000
Ram, Capital 1,875
Ulie, Capital 3,125

b. Occo, Capital 75,000


Ram, Capital 3,750
Ulie, Capital 6,250
Cash 85,000

c. Cash 85,000
Ram, Capital 3,750
Ulie, Capital 6,250
Occo, Capital 95,000

d. Occo, Capital 85,000


Cash 85,000
9. Using the same information in Number 8, except that Occo is paid 70,000 in
partnership cash for his equity. What is the journal entry to record OCCO’s
retirement?

a. Occo, Capital 75,000


Cash 70,000
Ram, Capital 1,875
Ulie, Capital 3,125

b. Occo, Capital 75,000


Ram, Capital 3,750
Ulie, Capital 6,250
Cash 85,000

c. Cash 85,000
Ram, Capital 3,750
Ulie, Capital 6,250
Occo, 95,000
Capital

d. Occo, Capital 85,000


Cash 85,000

10. The following condensed balance sheet is presented for the partnership of
Mia, Sasha, and Lexi, who share profits and losses in the ratio of 5:3:2,
respectively:

Cash 90,000
Other Assets 690,000
Mia, loan 30,000
Total 810,000
Accounts Payable 130,000
Lexi, loan 30,000
Mia, Capital 300,000
Sasha, Capital 200,000
Lexi, Capital 150,000
Total 810,000
Assume that the assets and liabilities are fairly valued on the balance sheet and
that the partnership decides to admit Johnny as a new partner, with a 20%
interest. No goodwill or bonus is to be recorded. How much should Johnny
contribute in cash or other assets?
a. 175,500 c. 140,000
b. 269,000 d. 162,500
SOLUTIONS
Problem 1
Total agreed capital of the new partnership
(equal to total contributed capital*) 260,000
Multiplied by: interest acquired 20%
Capital account to be credited to Dog 52,000 (c)
*Total Contributed capital (120,000 + 40,000 cash investment + 30,000
adjustment to fair value) = 260,000

Problem 2
150,000 + 22,500 = 172,500 (c)

Problem 3

Total agreed capital after the admission of Nikki:


(40,000 x 5) 200,000

Less: Contribution/Investment of Nikki (40,000)

Capital balances of JR before the admission of Nikki 160,000

Less: Capital Contribution (140,000 + 40,000) 180,000

Reduction of Inventory 20,000 (d)


Problem 4
Amount Paid 132,000

Less: Book Value of Interest Acquired:

(139,200 + 208,800 + 96,000) x 1/5 88,800

Gain 43,200 (B)

Problem 5
(B) 127,500 - Naruto’s capital balance is reduced by (35%/70%) x 45,000 bonus to

Sasuke or 22,500. Therefore, the capital of Naruto amounted to 127,500

(150,000- 22,500).

Problem 6

(c) 0 - Under the bonus method, unidentifiable assets (i.e., goodwill) are not

recognized. The total resulting capital is the FMV of the tangible investments

of the partners. Thus, there would be no unidentifiable assets recognized by

the creation of

this new partnership.

Problem 7

Amount paid 69,000

Less: Book value of interest of Lore (20%)

Total partner’s capital before withdrawal 300,000

Less: Total partners’ capital after withdrawal 269,000 31,000


Excess/Partial Goodwill 38,000

Divided by: 20%

Total Goodwill 190,000 (b)

Problem 8

RAM ULIE OCCO

Unadjusted, Capital 130,000 200,000 50,000

Profit* 30,000 50,000 20,000

Understated assets** 7,500 12,500 5,000

Adjusted Capital 167,500 262,500 75,000

Share in Profit Share in Adjustment of Understated

Assets

Profit= 100,000 Understatement in Assets= 25,000

Capital Interest= 3:5:2 Capital Interest= 3:5:2


Ram: 100,000*3/10 = 30,000 Ram: 25,000*3/10 = 7,500

Ulie: 100,000*5/10 = 50,000 Ulie: 25,000*5/10 = 12,500

Occo: 100,000*2/10 = 20,000 Occo: 25,000*2/10 = 5,000

Amount paid 85,000

Less: Adjusted Occo’s Capital 75,000

Deficit (10,000)

Journal Entry:

(b) Occo, Capital 75,000

Ram, Capital (10,000*3/8) 3,750

Ulie, Capital (10,000*5/8) 6,250

Cash 85,000
Problem 9

Amount paid 70,000

Less: Adjusted Occo’s Capital 75,000

Gain 5,000

Journal Entry:

(a) Occo, Capital 75,000

Cash 70,000

Ram, Capital (5,000*3/8) 1,875

Ulie, Capital (5,000*5/8) 3,125

Problem 10

Mia, Capital 300,000


Sasha, Capital 200,000
Lexi, Capital 150,000
Total Contribution 650,000
Divided by: 80%
Total agreed capital of the new partnership 812,500

Less: Contribution of old partners 650,000


Cash Investment of Johnny 162,500 (d)
Or, alternatively:
Total agreed capital of the new partnership 812,500

Multiplied by: Capital Interest of Johnny 20%


Cash Investment of Johnny 162,500 (d)
Note: The loans to or from partners are ignored in the admission of a new
partner because the focus will be more on capital interest rather than total
interest.
PARTNERSHIP OPERATION AND LIQUIDATION

THEORIES
1. Under the Lump-sum liquidation;
Statement 1: All of the non-cash assets are converted into cash
Statement 2: The total gains or loss on the sale allocated to the partners’
capital balances based on their P/L rations
a. True; True
b. True; False
c. False; True
d. False; False

2. The partnership's available cash is used to settle the claims using the
priority order below:

I. Outside creditor
II. Owner’s capital balances
III. Inside creditors

a. I, II, III
b. II, I, III
c. I, III, II
d. III, II, I

3. The legal characteristic of a partnership whereby each partner is an agent


of the partnership and is able to bind the partnership to contracts within the
normal scope of the partnership business is known as:

a. unlimited liability
b. partnership accounting
c. partnership contract
d. mutual agency

4. Which of the following is NOT true regarding a partnership?

a. A partnership is a voluntary association.


b. Partnerships pay income taxes.
c. Partnerships have limited life.
d. Partners in general partnerships have unlimited liability.

5. Which is NOT a condition of a limited partnership?


a. Limited partners are expected to have an active role in management.
b. A limited partner's liability will be limited to his/her investment.
c. One partner of the limited partnership must be a general partner.
d. Limited partnerships will have more than one class of partner.
6. Which component of the partnership profit and loss allocation
compensates partners for the routine time and effort expended in the business?

a. Interest on capital balance


b. Bonus
c. Salary
d. Residual interest

7. Which component of the partnership profit and loss allocation is most


commonly offered to the partner who manage the business?

a. Interest on capital balance


b. Bonus
c. Salary
d. Residual interest

8. A partnership’s income-sharing ratio

a. applies to partnership income after salaries and interest are deducted.


b. applies to partnership income before salaries are deducted but after
interest is deducted.
c. applies to partnership income after salaries are deducted but before
interest is deducted.
d. applies to partnership income before both salaries and interest are deducted.

9. Partnership net income is defined as

a. the interest allocation to the partners, based on the weighted average


invested capital.
b. partnership income after deducting partner salaries and interest.
c. partnership income after deducting partner salaries.
d. partnership income before deducting partner salaries and interest.

10. Which of the following is false regarding the measurement of partnership


income?

a. Partnerships employ the same revenue and expense recognition criteria


as corporations.
b. Salaries to partners are deducted as expenses in measuring partnership income.
c. Interest allocated to partners is not deducted as an expense in measuring
partnership income.
d. Partnership do not report income tax expense.

11. All partners will get the same amount of cash distributed to them as
part of liquidation.
a. True
b. False

12. Which of the following statement is correct?


1. Personal Creditors have first claim on partnership assets
2. Partnership Creditors have first claim on partnership assets
3. Partnership Creditors have first claim on personal assets
a.1
b.3
c.2
d. Both 2 and 3

13. Which item is not shown on the schedule of partnership liquidation?

a. Current Cash Balances


b. Liabilities still to be paid
c. Personal Assets of the partners
d. Property owned by the partnership

14. Which of the following is incorrect regarding interest on capital balances?


a. Its purpose is to give recognition to the similarities on capital
contributions by partners.
b. Its purpose is to give recognition to the differences on capital
contributions by partners.
c. It is appropriate to use it when the business is capital intensive.
d. None of the above.

15. Which of the following is/are true?

i. The partners share in partnership profits or losses in accordance with their


partnership agreement.

ii. If only the profit-sharing ratio has been agreed upon, each partner must share
in the losses with different ratio.

iii. In the absence of an agreement, partners must share in the profit or loss
based on their capital contributions.

a. I, II, III
b. I, III
c. II only
d. III only

16. Which of the following capital balances provide the fairest basis for
allocating partnership profit?
a. Original Capital
b. Beginning Capital
c. Ending Capital
d. Average Capital

17. Which of the following procedures is acceptable when accounting for a


deficit balance in a partner’s capital account during partnership liquidation?

a. A partner with negative capital balance must contribute personal assets to


the partnership that are sufficient to bring the capital account to zero.
b. If a partner with a negative capital balance is personally insolvent, the
negative capital balance may be absorbed by those partners having a positive
capital balance according to the residual profit and loss sharing ratios that apply
to all the partners.
c. If a partner with a negative capital balance is personally insolvent, the
negative capital balance may be absorbed by those partners having a positive
capital balance according to the residual profit and loss sharing ratios that apply
to those partners having positive balances.
d. All of the above procedures are acceptable.

18. In partnership liquidation, how are partner salary allocations treated?

a. Salary allocations take precedence over creditor payments.


b. Salary allocations take precedence over amounts due to partners with
respect to their capital interest, but not profits.
c. Salary allocations take precedence over amounts due to partners with
respect to their capital profits, but not capital interests.
d. Salary allocations are disregarded.

19. A partner’s loss absorption balance is calculated by:


a. dividing the partner’s capital balance by his percentage interest in capital
b. multiplying distributable assets by the partner’s profit sharing percentage
c. dividing the partner’s total interests by his profit and loss
sharing percentage
d. multiplying the partner’s total interests by his profit and loss
sharing percentage
20. Which of the following statements is/are true?
Statement 1: If the deficient partner is solvent, then he should invest cash
to eliminate his deficiency.
Statement 2: If the deficient partner is insolvent, then the other partners
should absorb his deficiency.
Statement 3: If the deficient partner has a loan balance, the right of offset
cannot be exercised and he should invest additional cash to eliminate his
deficiency.
a. Statements 1 and 2
b. Statements 1 and 3
c. Statements 2 and 3
d. All statements are true
PROBLEMS

1. J, K, and T are in the process of liquidating their partnership. Since it may


take several months to convert the other assets into cash, the partners
agree to distribute all available cash immediately, except for $10,000 that is
set aside for contingent expenses. The balance sheet and residual profit
and loss sharing percentages are as follows:
Cash ₱400,000 J, capital (40%)
₱135,000 Other assets
₱200,000 K, capital (30%)
₱216,000 Total assets
₱600,000 T, capital (30%)
₱49,000
Accounts payable

₱200,000 Total
liab./equity

₱600,000
How much cash should K receive in the first
distribution? a. ₱146,000
b. ₱147,000
c. ₱153,000
d. ₱156,000

2. Killua, Gon, and Kurapika are in the process of liquidating their partnership.
Lane has agreed to accept the inventory, which has a fair value of $60,000, as
part of her settlement. A balance sheet and the residual profit and loss sharing
percentages are as follows:
Cash ₱198,000 Jade, capital (40%)
₱79,000 Inventory
₱80,000 Kahl, capital (40%)
₱140,000 Plant assets
₱230,000 Lane, capital (20%)
₱140,000 Total assets
₱508,000 Accounts payable
₱149,000
Total liab./equity ₱508,000
If the partners then distribute the available cash, Kurapika will
receive a. ₱23,000
b. ₱29,000
c. ₱30,000
d. ₱34,000

3. The year-end balance sheet and residual profit and loss sharing
percentages for the A, B, and C partnership on December 31, 2005, are as
follows:
Cash ₱30,000 Accounts payable
₱200,000 Loan to A
₱40,000 Loan from B
₱50,000 Other assets
₱480,000 A, capital (25%)
₱70,000
Total assets ₱550,000 B, capital (25%)
₱80,000 C, capital
(50%) ₱150,000
Total liab./equity ₱550,000
The partners agree to liquidate the business and distribute cash when it
becomes available. A cash distribution plan for the A, B, and C partnership
will show that cash available, after outside creditors are paid, will initially go
to
a. A in the amount of ₱20,000
b. B in the amount of ₱45,000
c. B in the amount of ₱55,000
d. C in the amount of ₱90,000

4. The partnership agreement between Killua and Gon provided for a salary
allowance of $6,000 per month to Killua, and the balance to be divided equally
between partners Killua and Gon. Killua made no additional partnership
investments during the year, but withdrew $7,000 per month. Net income for the
year was $120,000. The net change in Killua's capital account was a:

a. $12,000 increase
b. $60,000 increase
c. $54,000 decrease
d. $12,000 decrease

5. Mr. Kaseki invested $20,000 in the partnership of Senku and Chrome. The
capital balance of Senku and Chrome were $40,000 and $60,000, respectively.
Income and loss is shared according to the ratio of equity balances. Mr. Kaseki
was to receive 25% interest in the new partnership. The journal entry to record
this transaction would include:
a. a credit to cash for $20,000
b. a credit to Senku's capital account for $4,000
c. a credit to Chrome's capital account for $6,000
d. a credit to Mr. Kaseki's capital account for $30,000

6. The partnership has the following accounting amounts:

Sales ₱ 70,000
Cost of goods sold 40,000
Operating expenses 10,000
Salary allocations to partners 13,000
Interest paid to banks 2,000
Partners’ withdrawals 8,000
a. ₱20,000
b. ₱18,000
c. ₱5,000
d. ₱(3,000)

7. Luffy is trying to decide whether to accept a salary of ₱40,000 or a salary of


₱25,000 plus a bonus of 10% of net income after salary and bonus as means of
allocating profits among partners. Salaries traceable to the other partners are
estimated to be ₱100,000. What amount of income is would be necessary so that
Luffy would consider the choices to be equal?
a. ₱165,000
b. ₱290,000
c. ₱265,000
d. ₱305,000

8. Sanji and Zoro are considering forming a partnership whereby profits will
be allocated through the use of salaries and bonuses. Bonuses will be 10% of net
income after total salaries and bonuses. Sanji will receive a salary of ₱30,000
and a bonus. Zoro has the option of receiving a salary of ₱40,000 and a 10%
bonus or simply receiving a salary of ₱52,000. Both partners will receive the same
amount of bonus. Determine the level of net income that would be necessary so
that Zoro would be indifferent to the profit sharing option selected.
a. ₱240,000
b. ₱300,000
c. ₱94,000
d. ₱334,000

9. Miguel is trying to decide whether to accept a salary of ₱60,000 or a salary of


₱45,000 plus a bonus of 15% of net income after salary and bonus as means of
allocating profits among partners. Salaries traceable to the other partners are
estimated to be ₱150,000. What amount of income is would be necessary so that
Luffy would consider the choices to be equal?
a. ₱210,000
b. ₱125,000
c. ₱260,000
d. ₱195,000

10. The partnership agreement of Miguel, John and Erick provides for the
year- ended allocation of net income in the following order:
 First, Miguel is to receive 10% of net income up to P200,000
and 20% over P200,000.
 Second, John and Erick each are to receive 5% of the remaining
income over P300,000.
 The balance of income is to be allocated equally among the three
partners.
The partnership’s 20x5 net income was P500,000 before any allocations
to partners. What amount should be allocated to Miguel?
a. ₱202,000
b. ₱216,000
c. ₱206,000
d. ₱220,000

11. A local partnership was considering the possibility of liquidation since one
of the partners is solvent (Lisa) and the others are insolvent. Capital balances at
the time were as follows. Profits and losses were divided on a 4:2:2:2 bases,
respectively.

Jennie, capital P60 000


Jisoo, capital 67 000
Rosé, capital 17 000
Lisa, capital 96 000
Jennie’s creditors file a P25 000 claim against the partnership’s assets. At
the time, the partnership held assets reported at P360 000 and
liabilities of P120
000. If the assets could be sold for P228 000, what is the minimum
amount of Jennie’s creditor would have received?
a. ₱38,720
b. ₱36,500
c. ₱2,500
d. ₱0

12. Ae Ra, Dong Man, Seol Hee and Joo Man are partners, sharing earnings
in the ratio of 3/21, 4/21, 6/21 and 8/21, respectively. The balances of their
capital accounts on December 31, 20x8 are as follows:

Ae Ra P1 000
Dong Man 25 000
Seol Hee 25 000
Joo Man 9 000
P60 000
The partners decide to liquidate and they accordingly convert the non-cash
asset into P23 200 of cash. After paying the liabilities amounting to P3000,
they have P22 200 to divide. Assume that a debit balance in any partner’s
capital is uncollectible. After the P22 200 was divided, the capital balance
of Dong Man was:
a. ₱3,920
b. ₱4,500
c. ₱3,250
d. ₱17,800

13. The partnership agreement of Chikaletta and Heihei provides that interest
at 10% per year is to be credited to each partner on the basis of weighted
average capital balances. A summary of Heihei’s capital account for the year
ended 31 December 2019 is as follows:
Balance, January 1 P 280,000
Additional Investment, July 1 80,000
Withdrawal, August 1 ( 30,000)
Balance, December 31 330,000
The amount of interest that should be credited to Heihei’s capital
account for 2019 is
a. ₱30,750
b. ₱30,500
c. ₱34,500
d. ₱33,000

14. Finn, Jake and Marceline are partners in the accounting firm. Their capital
account balances at year-end were: Finn, P90,000; Jake, P110,000; Marceline,
P50,000. They share profits and losses in a 4:4:2 ratio, after the following special
terms:

i. Marceline is to receive a bonus of 10% of the net income after bonus.


ii. Interest of 10% shall be paid on that portion of a partner’s capital in
excess of P100,000.
iii.Salaries of P10,000 and P12,000 shall be paid to Finn and Marceline,
respectively.
Assuming a net income of P44,000 for the year, the total profit
share of Marceline would be:
a. ₱7,800
b. ₱16,800
c. ₱19,400
d. ₱19,800

15. S, A, and D are liquidating their partnership. At the date the liquidation
begins S, A, and D have capital account balances of P162,000, P192,500, and
P215,000, respectively and the partners share profits and losses 40%, 35%, and
25%, respectively. In addition, the partnership has a P36,000 notes payable to S
and a P20,000 notes receivable from D. When the liquidation begins, what is the
loss absorption power with respect to A?
a. ₱192,500
b. ₱67,375
c. ₱550,000
d. ₱770,000

16. The J, O, and Y partnership became insolvent on January 1, 20x5, and the
partnership is being liquidated as soon as practicable. In this respect the following
information for the partners has been marshaled:

Capital Balances Personal Assets Personal Liabilities


J P 70,000 P 80,000 P 40,000
O (60,000) 30,000 50,000
Y (30,000) 70,000 30,000
Total P(20,000)
Assume that residual profits and losses are shared equally among the
three partners. Based on this information, calculate the maximum amount
that J can expect to receive from the partnership liquidation is:
a. ₱20,000
b. ₱40,000
c. ₱70,000
d. ₱110,000

17. When Happy and Lonely, partners who share earnings equally, were
incapacitated in an airplane accident, a liquidator was appointed to wind up their
business. The accounts showed cash, P35,000; other assets, P110,000;
liabilities, P20,000; Happy, capital, P71,000; and Lonely, capital, P54,000.
Because of highly specialized nature of the noncash assets, the liquidator
anticipated that considerable time would be required to dispose them. The
expenses of liquidating the business (advertising, rent, travel, etc.) are estimated
at P10,000. How much cash can be distributed safely to each partner at this
point?
a. P5,000 to Happy; and P0 to Lonely
b. P5,000 to Happy; and P500 to Lonely
c. P3,000 to Happy; and P0 to Lonely
d. P5,000 to Happy; and P1,000 to Lonely

18. Jin, Jun and Jay are partners in a public relations firm and share
profits and losses in the ratio of 2:2:1, respectively. They decided to liquidate
their business on December 31, 20x5. The following is the condensed
statement of financial position prepared prior to liquidation:

Cash ₱200,000
Non-cash assets 3,400,000
Total Assets ₱3,600,000

Liabilities ₱1,120,000
Jun, Loan 50,000
Jay, loan 80,000
Jin, capital 950,000
Jun, capital 600,000
Jay, capital 800,000
Total ₱3,600,000

Assume that the non-cash assets are sold at ₱2,500,000 with a resulting
loss on realization of ₱900,000 which was distributed in the ratio of 4:4:2.
The capital balance of each partner was sufficient to fully absorb the share
in the loss.
After all liabilities to outside creditors have been paid, how much will Jin receive?
a. ₱240,000
b. ₱590,000
c. ₱620,000
d. ₱640,000

19. Lisa, Jennie and Jisoo are partners in a public relations firm and share
profits and losses in the ratio of 2:2:1, respectively. They decided to liquidate
their business on December 31, 20x8. The following is the condensed statement
of financial position prepared prior to liquidation:

Cash ₱200,000
Non-cash assets 3,400,000
Total Assets ₱3,600,000

Liabilities ₱1,120,000
Jennie, Loan 50,000
Jisoo, loan 80,000
Lisa, capital 950,000
Jennie, capital 600,000
Jisoo, capital 800,000
Total ₱3,600,000

Assume that the non-cash assets are sold at ₱1,850,000 and the resulting
loss was distributed in the ratio of 2:2:1. How much cash will Jisoo receive
excluding the loan balance?

a. ₱310,000
b. ₱330,000
c. ₱490,000
d. ₱570,000

20. Jake, Jihyo and Jae are partners with a profit and loss ratio of 4:3:3. The
partnership was liquidated, and prior to the liquidation process, the partnership
statement of financial position was as follows:

Cash ₱30,000
Non-cash assets 270,000
Total Assets ₱300,000
Jake, capital ₱108,000
Jihyo, capital 120,000
Jae, capital 72,000
Total ₱300,000

After the partnership was liquidated and the cash was distributed, Jihyo received
₱48,000 in cash as full settlement of her interest. How much was the
partnership liquidation loss?

a. ₱72,000
b. ₱180,000
c. ₱240,000
d. ₱252,000
SOLU
TIONS
THEO
RIES
1. Lump-sum liquidation is a method of partnership liquidation where;
a. All of the non-cash assets of the partnership are converted into cash;
b. The total gain or loss on the sale is allocated to the partners’ capital
balances based on their P/L ratios;
c. Actual liquidation expenses are allocated to the partners’ capital
balances based on their P/L ratios.
d. The liabilities to outside creditors are fully settled;
e. The liabilities to inside creditors are fully settled;
f. Any remaining cash is distributed to the owners in full settlement of their
interests;

2. It is stated in the partnership liquidation that when settling the available


cash of the partnership, this hierarchy must be followed:
1st. Outside Creditor
2nd. Inside Creditor (e.g., payables to
partners) 3rd. Owner’s capital balances

3. Mutual agency is being described. It is one of the main reasons why care
should be taken in selecting partners. A corporation's shareholders have no
mutual agency relationship with the corporation.

4. Partnerships do not pay income taxes. Each partner will pay taxes on
his/her share of partnership income.

5. Limited partners have limited liability, therefore, they have no active role in
the management of the partnership. There are two classes of partners: (a) limited
partners and (2) at least one general partner.

6. Salaries – normally, an industrial partner receives salary in addition to his


share in the partnership’s profits compensation for his services to the partnership.

7. Bonuses – the managing partner may be entitled to a bonus for excellent


management performance. Partners are not entitled to any bonus if the operation
of the partnership is at loss.

8. Salaries and interest paid to partners are considered as an allocation of net


income rather than as determinant of net income and therefore deducted prior to
income distribution.

9. Partners are not considered employees or creditors of the partnership, but


the salaries and interest paid to these partners only affect their respective capital
accounts, thus, these transactions should not be included in determining the
partnership net income.
10. Salaries to partners are not expenses of the partnership but part of profit
and loss sharing plan and shall not be deducted as expenses in measuring
partnership income.

11. The amount of cash distributed to each partner depends on the capital
balances and the profit and loss ratios. With this, it is unlikely that partners will
receive the same amount of cash.

12. The order of priority concerning the availability of the assets for each
class of creditors is as follows:
A. Partnership Assets
1. Partnership Creditors
2. Personal Creditors that did not recover their claims in full from
personal assets.
B. Personal Assets
1. Personal Creditors
2. Partnership creditors who were not satisfied from partnership assets.
3. Amounts owed to partners by way of contribution.

13. Personal Assets of the partners is not shown in the Schedule of partnership
liquidation because the schedule is prepared to provide information to the
creditors and partners about liquidation transactions to date, property still
being held by the partnership, liabilities remaining to be paid, and current
cash and capital balances.

14. The purpose of allowing interest on capital is to give recognition to


differences on capital contributions by partners. It also recognizes the
contribution of the partners’ capital contribution to the partnership’s profit-
generating capacity. The use of interest on capital as a means of allocating profits
would be appropriate when the business is capital intensive versus labor
intensive.

15. General rule: The partners share in partnership profits or losses in


accordance with their partnership agreement.
Article 1797 of the Philippine Civil Code’s additional rules:
i. If only Profit-Sharing Ratio has been agreed upon, each partner must
share in the losses with the same proportion.
ii. In the absence of an agreement/stipulation, partners must share in the
income or loss based on their capital contributions.
(Note: Industrial partners only contribute industry to the
partnership, thus unmeasurable. They shall receive share as
may be just and equitable under circumstances.)
16. Average Capital must have provided the fairest basis for allocating
partnership profit because it reflects the capital actually available for use by the
partnership during the year. An agreement for the use of average capital also acts
as an incentive for additional investment when these can be profitably employed.
17. If any partner has a deficit balance, an additional contribution should be
made to offset the negative amount. In some situations, a question may arise as
to whether compensation for a deficit will ever be forthcoming from the
responsible party. The remaining partners may choose to allocate the available
cash immediately based on the assumption that the deficit balance eventually will
prove to be a total loss.

18. As partners are the owners of the business, they do not receive a salary
but each has the right to withdraw assets up to the level of his/her capital account
balance.

19. The formula in computing a partner’s loss absorption capacity is by dividing


the partner’s total interest in the partnership by his profit and loss sharing
percentage.

20. Only Statements 1 and 2 are true. If a deficient partner has a loan
balance, the right of offset should be exercised.
PROBLEMS

1.

2.

3.
4. The distribution of net income would be: $72,000 to Killua for the salary
allowance, $24,000 each to Killua and Gon of the remaining $48,000
($120,000 -
$72,000). Killua's capital was credited for $96,000 and debited for $84,000 (12 x
$7,000), a net increase of $12,000.

Killua Gon Net Income


Salaries $72,000 - 72,000
Remaining (50:50) 24,000 24,000 48,000
Total Share $96,000 $24,000 $120,000
Withdrawals (84,000) - -
Net increase (decrease) $12,000 $24,000 $120,000

5. The new equity balance would be $120,000 ($40,000 + $60,000 + $20,000).


A 25% interest is $30,000 ($120,000 x .25). This creates a bonus of $10,000 to
the new partner. The bonus is divided between Senku and Chrome in proportion
to their previous investment ratio of 4:6, or 40%, 60% and the amounts would be
debited to their accounts.
The entry would be:
Cash 20,000
Senku, capital 4,000
Chrome, capital 6,000
Mr. Kaseki, capital 30,000

6.

Sales ₱ 70,000
Less: Cost of goods sold  40,000
Gross profit ₱ 30,000
Less: Operating expenses  10,000
Operating income ₱ 20,000
Less: Other expenses; Interest expense  2.000
Net income ₱ 18,000
Salaries to partners are considered as an allocations of net income rather than
as determinant of net income. In other words, salaries to partners are not
expenses of the partnership, but part of profit and loss sharing plan.

7. To equate ₱40,000 to ₱25,000 plus bonus, the bonus should amount to


₱15,000 (₱40,000 less ₱25,000). Based on the foregoing, the following equation
should be developed:
Bonus = 10% (NI – Salaries – Bonus)
₱15,000 = .10 [NI – (₱100,000 + ₱25,000) – ₱15,000]
₱15,000 = .10 (NI – ₱140,000)
₱15,000 = .10NI – ₱14,000
₱29,000 = .10NI
₱29,000/.1 = NI
NI = ₱290,000

8. To equate ₱52,000 and ₱40,000 plus bonus, the bonus should amount to
₱12,000 (₱52,000 less ₱40,000) to be indifferent under the two profit-sharing
options. Since Zoro would receive the same bonus, the total bonus would have to
be ₱24,000 (₱12,000 x 2). Based on the foregoing, the following equation should
be developed:

Bonus = 10% (NI – Salaries – Bonus)


₱24,000 = .10 [NI – (₱30,000 + ₱40,000) – ₱24,000]
₱24,000 = .10 (NI – ₱94,000)
₱24,000 = .10NI – ₱9,400
₱33,400 = .10NI
₱33,400/.1 = NI
NI = ₱334,000

9.

Bonus = 15% (NI – Salaries – Bonus)


₱15,000 = .15 [NI – (₱100,000 + ₱45,000) – ₱15,000]
₱15,000 = .15 (NI – ₱160,000)
₱15,000 = .15NI – ₱24,000
₱39,000 = .15NI
₱39,000/.15 = NI
NI = ₱260,000

10.

Miguel John Erick Total


Miguel First P200,000 x 10% P20,000 P20,000

Over P200,000: (P500,000-


60,000 60,000
P200,000) x 20%

5% of remaining income
over P300,000:
John and (P500,000
Erick - P20,000 - P60,000 - P6,000 P6,000 P12,000
P300,000) x 5%
Balance: Allocate equally 136,000 136,000 136,000 408,000
P216,000 P142,000 P142,000 P500,000
11.

Jennie Jisoo Rosé Lisa Total


Balances before Liquidation P 60 P67 P17 P96 P240
000 000 000 000 000
Loss on Realization – 4:2:2:2 (P228 (52 (26 (26 (26 (132
000 – P360 000) 800) 400) 400) 400) 000)
Balances P7 200 P40 P(9 P69 P108
600 400) 600 000
Loss on possible insolvency (4:2:2) (4 700) (2 350) 9 400 (2 -0-
350)
Balances P2 500 P38 0 P67 P108
250 250 000

12.

Ae Ra Dong Seol Joo Total


Man Hee Man
Balances before Liquidation P1 P25 P25 P9 P60
000 000 000 000 000
Loss on Realization – 3/21: 4/21: 6/21: (5 (7 200) (10 (14 (37
8/21 (P22 200 – P60 000) 400) 800) 400) 800)
Balances P(4 P17 P14 P(5 P22
400) 800 200 400) 200
Loss on possible insolvency of Ae Ra and 4 400 (3 920) 5 880 5 400 -0-
Joo Man 4:6 (P4 400 + 5 400)
Cash Received 0 P13 P8 0 P22
880 320 200

Therefore, the capital balance of Domingo after cash settlement is:


Capital Balance after loss on realization but before
payment to partners ₱17, 800
Less: Cash Received 13, 880
₱ 3 920
13.

Date Capital Balances No. of Months Unchanged Weighted Average


January 1 P 280,000 6 P 1,680,000
July 1 360,000 1 360,000
August 1 330,000 5 1,650,000
Total 12 P 3,690,000
Divided by: 12
Total P 307,500
Multiplied by: 10%
Interest P 30,750

14.

Finn Jake Marceline Total


Bonus - - P4,000 P4,000
Interest - 1,000 - 1,000
Salaries 10,000 - 12,000 22,000
Balaance 6,800 6,800 3,400 17,000
Total P16,800 P7,800 P19,400 P44,000
Bonus computation:
B = 10% of Net Income after
Bonus B = 10% (P44,000 –
B)
B = P4,400 –
0.10B 1.10B
= P4,400
B = P4,000
Interest computation:
10,000 x 10% = P1,000

15. P192,500/35% = P550,000


16.

J O Y
Balances before realization 70,000 (60,000) (30,000)
Additional investment 30,000
Balances 70,000 (60,000)
Additional loss (1:1) (30,000) 60,000 (30,000)
Balances 40,000 (30,000)
Additional investment (P70,000 – P30,000 – P30,000) 10,000
Balances 40,000 (20,000)
Additional loss (20,000) 20,000
Balances 20,000

17.

Happy Lonely Total


Balances before liquidation P71,000 P54,000 P125,000
Loss on possible unrealization of noncash assets (55,000) (55,000) (110,000)
(equally)
Balances P16,000 P (1,000) P15,000
Liquidation expenses (equally) ( 5,000) ( 5,000) (10,000)
Balances P11,000 P (6,000) P 5,000
Loss for possible insolvency of Lonely ( 6,000) 6,000 -
Cash received P 5,000 0 P 5,000
18.

Jin, Jun and Jay


Partnership Statement of
Realization and Liquidation
December
31,
20x5
(in
Pesos)

No
n- Jin, Jun Jay,
Cash Liabilitie Jun, Loan Jay, Capi , Capit
Ca s Loan
sh tal Capi al
tal
Asset
s
Balances
before 200,000 3,400,0 1,120,0 50,000 80,000 950,000 600,0 800,00
liquidation 00 00 00 0
Realizatio
n and 2,500,00 (3,400,0 (360,00 (360,00 (180,0
distributio 0 00) 0) 0) 00)
n
of losses
Balances
after 2,700,00 1,120,0 50,000 80,000 590,0 240,0 620,00
realization 0 00 00 00 0
Payment of (1,120,0 (1,120,0 _
liabilities 00) 00)
Balances
after 1,580,00 50,000 80,000 590,0 240,0 620,00
payment 0 00 00 0
of
liabilities
Payment
to (130,00 (50,000) (80,000 _
partners - 0) )
loan
Balances
after 1,450,00 590,0 240,0 620,00
payment 0 00 00 0
of
partners’
loans
Payme
nt to (1,450,0 (590,00 (240,00 (620,0
partner 00) 0) 0) 00)
s -
capital
19.

Lisa, Jennie and Jisoo


Partnership Statement of
Realization and Liquidation
December 31, 20x8
(in Pesos)

No

n-
Jiso Lis Jenn Jisoo
Cash Liabilitie Jennie,
Ca o, a, ie, ,
s Loan
Loa Capi Capi Capit
sh n tal tal al

Assets
Balances
before 200,000 3,400,0 1,120,0 50,000 80,000 950,00 600,00 800,0
liquidation 00 00 0 0 00
Realizatio
n and
distributio
n 1,850,00 (3,400,0 (620,00 (620,0 (310,0
0 00) 0) 00) 00)
of losses
Balances
after realization 2,050,00 1,120,0 50,000 80,000 330,0 (20,00 490,0
0 00 00 0) 00
Payment of (1,120,0 (1,120,0 _
liabilities 00) 00)
Balances 930,000 50,000 80,000 330,0 (20,00 490,0
00 0) 00
Offset of
Jennie’s loan (20,000) 20,000
against her
deficiency

Balances 930,000 30,000 80,000 330,0 - 490,0


00 00
Payment to
partners -
(930,00 (30,000) (80,000 (330,00 - (490,0
0) ) 0) 00)
capital

20.

Jihyo, capital ₱120,000


Less: Cash settlement received 48,000
Jihyo’s loss from liquidation 72,000
Divide by: Jihyo’s profit and loss ratio 30%
Total partnership liquidation loss ₱240,000
CORPORATE LIQUIDATION
THEORIES

1. The financial reports for a corporate bankruptcy liquidation are:

a. Balance sheet and statement of affairs


b. Statement of affairs and statement of realization and liquidation
c. Balance sheet and income statement
d. Statement of affairs and income statement

2. Which one of the following assets of a corporation is most likely to

realize the smallest percentage of its book value in bankruptcy?

a. Accounts receivable
b. Plant & equipment
c. Goodwill
d. Inventories

3. Which of the following is not included in the category of unsecured

liabilities with priority in a liquidation case?

a. Unpaid wages
b. Amounts due to the BIR
c. Amounts due to suppliers
d. Administrative expenses of the trustee

4. Which statement is false concerning the statement of affairs?

a. Total unsecured liabilities do not include unsecured debts with priority.


b. Unsecured liabilities consist of debts for which no assets are pledged as
security, as well as debts in excess of the liquidation value of assets
pledged.
c. Net free assets are the excess of liquidation value of assets pledged to
fully secured creditors over the amount of fully secured liabilities plus free
assets less unsecured liabilities with priority.
d. The estimated deficiency to unsecured creditors is total unsecured
liabilities less total free assets.
5. Which monthly report shows the results of the trustee's fiduciary actions

beginning at the point the trustee accepts the debtor's assets?

a. Statement of affairs
b. Statement of realization and liquidation
c. Statement of financial position
d. Statement of activities

6. Under the Bankruptcy Code, an insolvent corporation may be:

I. Reorganized.

II. Liquidated.

a. I
b. II
c. Either I or II
d. Neither I nor II

7. _ have liens, or security interests, on specific assets.

a. Secured creditors
b. Creditors with priority
c. Unsecured creditors
d. Assured creditors

8. As defined by the Bankruptcy Code, creditors with priority:

I. have collateral claim against specific assets.

II. are unsecured creditors who have priority over other unsecured creditors.

III. are the first to be paid from any proceeds available to unsecured creditors.

a. I only
b. II only
c. I, II and III
d. Both II and III
9. In the statement of affairs, the expected recovery percentage for a company’s

unsecured creditors is calculated as

a. Net free assets divided by unsecured liabilities


b. Net free assets divided by unsecured liabilities plus liabilities with priority
c. Total free assets divided by unsecured liabilities other than unsecured
liabilities with priority
d. Total free assets divided by unsecured liabilities plus liabilities with priority

10. Which of the following statements is true?

a. Certain debts are not dischargeable


b. The goal of liquidation is to give the company a new start
c. All secured claims are paid in full
d. The expenses to administer the estate are paid last because they are unsecured

11. Which of the following does not describe the accounting statement of affairs?

a. The emphasis is an asset net realizable value, not historical cost


b. The statement of affairs is concerned is only with the assets of the
debtor organization, not the claims
c. The statement can also be used in reorganization
d. The statement of affairs is based on estimated values; actual realized
values may be different

12. The document used to estimate amounts available to each class of claims is

called a(n)

a. Statement of Assets and Liabilities


b. Legal Statement of Affairs
c. Accounting Statement of Affairs
d. Statement of Realization and Liquidation

13. The duties of the trustee include:

A. Appointing creditors’s committees in liquidation cases.


B. Approving all payments for debts incurred before the bankruptcy filing.
C. Examining claims and disallowing any that are improper.
D. Calling a meeting of the debtor’s creditors.
14. An involuntary petition filed by a firm’s creditors whereby there are twelve or

more creditors must be signed by at least:

a. two creditors.

b. three creditors.

c. five creditors.

d. six creditors.

15. A bankruptcy petition filed by a firm is a

a. Involuntary activities
b. Voluntary deed
c. Involuntary giving up
d. Voluntary petition

16. These are assets with realizable values equal to, or in excess of the

liabilities for which they have been pledged as collateral.

a. Assets Pledged to Fully Secured Creditors


b. Assets Pledged to Partially Secured Creditors
c. Free Assets
d. Fully Secured Creditors

17. Statement 1: The basic focus of accounting for liquidation is that of a

“quitting concern”.

Statement 2: “Going concern” is the focus of accounting for liquidation.

a. Only statement 1 is correct.


b. Only statement 2 is correct.
c. Both statements are correct.
d. Neither of the two statements are correct.
18. A debtor may file which type of petition when seeking judicial protection

under the Bankruptcy Law?

a. Voluntary
b. Involuntary
c. Voluntary and Involuntary
d. A debtor cannot file a petition.

19. It is an activity statement that is intended to show progress toward the

liquidation of a debtor’s estate.

a. Statement of Affairs
b. Statement of Realization and Liquidation
c. Statement of Deficiency
d. Statement of Financial Position

20. It refers to a condition in which a company is unable to meet debts as the

debts mature.

a. Bankruptcy
b. Credit squeeze
c. Deficit
d. Insolvency

21. which of the following is first-ranked of the unsecured liabilities with

priority in bankruptcy liquidation?

a. Claims of governmental entities for various taxes and duties

b. Administrative costs

c. Claims for wages, salaries, and commissions, subject to limitations of

amount and time

d. None of the foregoing


22. The Statement of Realization and Liquidation differs from the Statement of

Affairs because

a. The Statement of Realization and Liquidation reports estimated realizable


values rather than actual liquidation results.
b. The Statement of Realization and Liquidation is a summary of
secured debt activity only.
c. The Statement of Realization and Liquidation is prepared only at final
completion of the liquidation process.
d. The Statement of Realization and Liquidation reports actual liquidation
results rather than estimated realizable value.

23. Jangga Corporation’s accounting statement of affairs shows a dividend of 120%.

The dividend means that:

a. Unsecured creditors will receive an amount in excess of the book value


of their claims.
b. Secured creditors will receive an amount in excess of the book value
of their claims.
c. An error was made in the preparation of the statement.
d. Stockholders may expect some return on their interests.

24. A document which original purpose is to inform the bankruptcy court and

interested creditors of the accomplishment of the trustee.

a. Statement of Assets and Liabilities


b. Legal Statement of Affairs
c. Accounting Statement of Affairs
d. Statement of Realization and Liquidation

25. Liabilities of ₱100,000 were settled in full for ₱80,000. The ₱20,000

difference would be reported on the statement of realization and liquidation as:

a. a loss
b. a gain
c. liabilities to be liquidated
d. liabilities not liquidated
PROBLEMS

1. Land Bank of the Philippines holds a P500,000 note secured by a building

owned by Hampsland Software, which has filed for bankruptcy. If the property

has a book value of P600,000 and a fair market value of P450,000, what is the

best way to describe the notes held by the Land Bank of the Philippines? The

bank has

a. A secured claim of P500,000.


b. An unsecured claim of P500,000.
c. A secured claim of P450,000 and an unsecured claim of P50,000.
d. A secured claim of P50,000 and an unsecured claim of P50,000.

2. J and F Inc. owes the Maurer Corporation P60,000 on account, which is

secured by accounts receivable with a book value o P50,000. The unsecured

portion is considered a claim under the bankruptcy law, J and F has filed for

bankruptcy. Its statement of affairs lists the accounts receivable securing the

Maurer account with an estimated realizable value of P45,000. If the dividend to

general unsecured creditors is 80%, how much can Maurer expect to receive?

a. 60,000
b. 58,000
c. 57,000
d. 48,000

3. Wang Corporation is a parent, having purchased 60% of Jackson Company’s

common stock at par value for 600,000. Jackson Company is in financial

difficulty. The parent granted an unsecured loan of P200,000 to the subsidiary.

An accounting statement of affairs for Jackson Company shows a dividend of

30%. Wang Corporation can expect to receive on the loan of appropriately:


a. 120,000
b. 60,000
c. 36,000
d. 0

4. Auqona, Inc. has forced into bankruptcy and has begun to liquidate.

Unsecured claims will be paid at the rate of 40 cents on the peso. Lebenpe Co.

holds a non- interest-bearing note receivable from Auqona in the amount of

P100,000, collateralized by machinery with a liquidation value of P25,000. The

total amount to be realized by Lebenpe on this note receivable is:

a. 25,000
b. 40,000
c. 55,000
d. 65,000

5. Jimin Co. filed a voluntary bankruptcy petition on October 15, 20x5, and the

statement of affairs reflects the following amounts:

Book Value Estimated

Current Value

Assets:

Assets pledged with fully secured creditors ₱300,000 ₱370,000

Assets pledged with partially secured creditors 180,000 120,000

Free Assets 420,000 320,000

₱900,000 ₱810,000
Liabilities:

Liabilities with priority ₱70,000

Fully secured creditors 260,000

Partially secured creditors 200,000

Unsecured creditors 540,000

₱1,070,000

Assume that the assets are converted to cash at the estimated current values

and the business is liquidated. What amount of cash will be available to pay

unsecured nonpriority claims?

a. 240,000
b. 280,000
c. 320,000
d. 360,000
6. Jin and Co., Inc. purchased a Cadillac automobile with little cash down and

signed a note, secured by the Cadillac for 48 easy monthly payments. When the

company files for bankruptcy, the balance due on the Cadillac amount to

₱6,000,000. The car has a book value of 8,000,000 and a net realizable value of

₱4,000,000. The unsecured creditors of Jin and Co. can expect to receive 50% of

their claims. In the liquidation, the bank that holds the note on the Cadillac should

receive:

a. 6,000,000

b. 5,000,000
c. 3,000,000

d. 4,000,000

7. The following data are provided by the BTS Company:

Assets at book value ₱150,000

Assets at net realizable value 105,000

Liabilities at book value:

Fully secured mortgage 60,000

Unsecured accounts and notes payable 70,000

Unrecorded liabilities:

Interest on bank notes 500

Estimated cost of administering estate 6,000

The court has appointed a trustee to liquidate the company. The journal entry

made by the trustee to record the assets and liabilities should include any estate

deficit of:

a. 31,500
b. 31,000
c. 25,500
d. 25,000

8. A statement of affairs shows 30,000 of assets pledged to partially secured

creditors liabilities of ₱65,000 to partially secured creditors. ₱25,000 to

unsecured creditors with priority, and ₱90,000 to other unsecured creditors. If the

deficiency to unsecured creditors is ₱40,000, what is the amount of net free

assets?

a. 50,000
b. 75,000
c. 85,000
d. 110,000
9. Bana Na Corporation is in bankruptcy and being liquidated by a court-

appointed trustee. The financial report that follow was prepared by the trustee just

before the final cash distribution:

Assets:
Cash P 100,000

Approved Claims:
P 80,000
Mortgage payable (secured by property that was sold for P50,000)
Accounts payable, unsecured 50,000
Administrative expense payable, unsecured 8,000
Salaries payable, unsecured 2,000
P 140,000

The administrative expenses are for the trustees and other costs of administering
the debtor corporation’s estate. How should the P100,000 be distributed to the
following creditors?

Unsecured Partially Unsecured


Creditors Secured Creditors
With Priority Creditors Without Priority

a. P - P 80,000 P 20,000
b. 10,000 80,000 10,000
c. 5,000 65,000 25,000
d. 10,000 65,000 25,000

10. On December 18, 20x5, the statement of affairs of SamSan Tech Company,

which is bankruptcy liquidation, included the following:

Assets pledged for fully secured liabilities P 100,000


Assets pledged for partially secured liabilities 40,000
Free assets 120,000
Fully secured liabilities 80,000
Partially secured liabilities 50,000
Unsecured liabilities with priority 60,000
Unsecured liabilities without priority 90,000

Compute the estimated amount to pe paid to:

Fully Unsecured Partially Unsecured


Secured Liabilities Secured Liabilities
Liabilities w/ Priority Liabilities w/out Priority

a. P80,000 P60,000 P50,000 P70,000


b. 64,000 60,000 48,000 88,000
c. 80,000 48,000 60,000 72,000
d. 80,000 60,000 48,000 72,000

11. The following data were taken from the statement of affairs of in Injae Company:

Assets pledged for fully secured liabilities (fair value, P75,000) P 90,000
Assets pledged for partially secured liabilities (fair value, P52,000) 74,000
Free assets (fair value, P40,000) 70,000
Unsecured liabilities with priority 7,000
Fully secured liabilities 30,000
Partially secured liabilities 60,000
Unsecured liabilities without priority 112,000

Compute the: (1) total estimated deficiency to unsecured creditors, and

(2) the expected recovery per peso of unsecured claims.

a. (1) P42,000: (2) P .65


b. (1) P 3,000: (2) P .98
c. (1) P 0: (2) P1.00
d. (1) P42,000: (2) P .70

12. Orangu Tan, a CPA, has prepared a statement of affairs. Assets which

there no claims or liens are expected to produce P70,000, which must be

allocated to unsecured
claims of all classes totaling P105,000. The following are some of the

claims outstanding:

1. Accounting fees for Orangu Tan, P1,500.


2. An unrecorded note for P1,000, on which P60 of interest has accrued,
held by Chimpan Zee.
3. A note for P3,000 secured by P4,000 receivables, estimated to be 60%
collectible held by Mon Key.
4. A P1,500 note, on which P30 of interest has accrued, held by Gor Illa.
Property with a book value of P1,000 and a market value of P1,800 is
pledged to guarantee payment of principal and interest.
5. Unpaid income taxes of P3,500

Compute the estimated payment to partially secured

creditors: a. P1,060
b. P1,950
c. P2,490
d. P2,790

13. On July 1, 2021, the records of Mr. Midoriya, trustee in bankruptcy for One
for All Corporation, showed the following:

Cash P 77,400
Assets to be realized:
Furnitures 90,000
Buildings 321,000
Machinery 216,000
Copyright 50,800
Liabilities to be liquidated:
Accounts payable 580,000
Notes payable 300,000
Estate Deficit 204,800

During July, Mr. Midoriya sold machinery having a book value of P125, 000 for P81,
600 and sold the copyright for P104, 000. Mr. Midoriya was paid P29, 100 as
trustee fee and P167, 000 was distributed proportionately to the creditors.
Compute for Net Income (Loss) and Estimated Deficiency.
a. 9,800; (204,800)
b. 240,800; (204,800)
c. 700; (204,100)
d. 29,100; (204,100)

14. If a statement of realization shows asset to be realized of P 820,000, assets


not realized of P 280,000, assets acquired of zero, and a gain on realization of P
20,820, the amount that would be reported for assets realized is:

a. P 840,280
b. P 1,120,820
c. P 560,820
d. P 519,180

15. The following data were taken from the statement of realization and

liquidation of AKMU Corporation for the quarter ended September 30, 2021:

Assets to be realized P 610,000


Assets acquired 640,000
Assets realized 700,000
Assets not realized 430,000
Liabilities to be liquidated 820,000
Liabilities assumed 460,000
Liabilities liquidated 640,000
Liabilities not liquidated 730,000
Supplementary credits 790,000
Supplementary charges 748,000
The ending balances of capital stock and retained earnings are P300,
000 and P120, 000, respectively.
What is the net income (loss) for the period?

a. 252,000
b. (48,000)
c. (12,000)
d. 168,000

16. A statement of affairs shows P 180,000 of assets pledged to partially secured

creditors liabilities of P 215,000 to partially secured creditors. P 175,000 to

unsecured liabilities with priority, and P 240,000 to other unsecured creditors. If

the deficiency to unsecured creditors is P 190,000, what is the amount of free net

assets?

a. 85,000
b. 260,000
c. 275,000
d. 190,000

17. The following data are taken from the statement of affairs of Saitama Company.

Assets pledged with fully secured creditors


(Realizable value, $585,000)

$750,000 Assets pledged with partially secured creditors


(Realizable value, $250,000) 315,000
Free assets (Realizable value, $290,000) 485,000
Fully secured creditor claims 266,000
Partially secured creditor claims 350,000
Unsecured creditor claims with priority 50,000
General unsecured creditor claims 1,115,000
Compute the amount that will be paid to each class of creditor.

Fully Partially Unsecured General


Secured Secured w/ Priority unsecured
a. 266,000 296,000 50,000 513,000
b. 266,000 286,000 60,000 513,000
c. 276,000 296,000 50,000 518,000
d. 276,000 286,000 60,000 518,000

18. Assets of ₱500,000 existed at the beginning of a period. During the period,
carried at ₱200,000 were sold for ₱280,000, and new assets of ₱100,000 were
acquired. A statement of realization and liquidation would show “assets not
realized” of:
a. ₱220,000
b. ₱300,000
c. ₱320,000
d. ₱400,000

19. Liabilities of ₱95,000 existed at the beginning of a period. During the period,
liabilities recorded at ₱49,000 were settled for ₱43,000, and new liabilities of
₱17,000 were incurred. A statement of realization and liquidation would show
“liabilities not liquidated” of:

a. ₱51,000
b. ₱63,000
c. ₱69,000
d. ₱107,000
20. Paubaya Co. is insolvent, and its statement of affairs shows the following
information:

Estimated gains on realization of ₱1,440,000


assets

Estimated losses on realization of 2,000,000


assets

Additional assets 1,280,000

Additional liabilities 960,000

Capital stock 2,000,000

Deficit 1,200,000

The pro-rate payment on the peso to stockholders (estimated amount to be


recovered by stockholders) is:
a. P.30
b. .43
c. P.57
d. .70
21. Paalam Corp. has been undergoing liquidation since January 1. As of March
31, its condensed statement of realization and liquidation is presented below:

Assets:
Assets to be realized ₱1,375,000
Assets acquired 750,000
Assets realized 1,200,000
Assets not realized 1,375,000
Liabilities:
Liabilities liquidated ₱1,875,000
Liabilities not liquidated 1,700,000
Liabilities to be liquidated 2,250,000
Liabilities assumed 1,675,000
Revenues and Expenses:
Supplementary charges ₱3,125,000
Supplementary credits 2,800,000
The net gain (loss) for the three-month period ending March 31 is:
a. ₱250,000
b. (₱325,000)
c. ₱425,000
d. ₱750,000

22. Using the same information on No. 19, compute the ending cash balance of
cash account assuming that common stock and deficits are ₱1,500,000 and
₱500,000, respectively.
a. ₱425,000
b. ₱575,000
c. ₱1,325,000
d. ₱1,375,000

23. Seri’s choice company has the following balance sheet:

Plant and
equipment ₱300,000 Accounts Payable ₱200,000
Mortgage Payable 80,000
Stockholder’s
equity 20,000

Total ₱300,000 Total ₱300,000

The plant and equipment has a current fair value of ₱100,000, and is

pledges as security for the mortgage. The estimated deficit to unsecured

creditors is

a. ₱0
b. ₱90,000
c. ₱170,000
d. ₱180,000
24. A statement of affairs shows ₱30,000 of assets pledged to partially secured
creditors liabilities of ₱60,000 to partially secured creditors. ₱25,000 to
unsecured creditors with priority, and ₱90,000 to other unsecured creditors. What
is the amount of net free assets if the deficiency to unsecured creditors is
₱55,000?

a. ₱90,000
b. ₱65,000
c. ₱85,000
d. ₱120,000

25. Assets of ₱60,000 existed at the beginning of a period. During the period,
assets carried at ₱25,000 were sold at ₱35,000 and new assets of ₱15,000 were
acquired. A statement of realization and liquidation would show “assets not
realized” of:

a. ₱35,000
b. ₱25,000
c. ₱50,000
d. ₱40,000
ANSWE

R KEY

THEORI

ES 1.B

Statement of affairs is a financial condition prepared for a corporation

entering into the stage of liquidation or bankruptcy. Statement of realization and

liquidation is an activity statement that is intended to show progress toward the

liquidation of a debtor's estate. It also informs the bankruptcy court and interested

creditors of the trustee's accomplishment.

2. C

Liquidation value does not include intangible assets such as a company's

intellectual property, goodwill, and brand recognition. However, if a company is

sold rather than liquidated, both the liquidation value and intangible assets

determine the company's going-concern value.

3. C

Unsecured liabilities with priority are liabilities that must, by statute, be

paid off before any secured debts can be satisfied, therefore, amounts due to

suppliers is not included. Section 50 of insolvency law does not mention amounts

due to suppliers as one of unsecured liabilities with priority.


4. D

Estimated deficiency to unsecured creditors can be computed as : total

unsecured liabilities (partially secured and unsecured liabilities without priority)

less net free assets.

5. B

Statement of realization and liquidation is an activity statement that is

intended to show progress toward the liquidation of a debtor's estate. Its original

purpose was to inform the bankruptcy court and interested creditors of the

trustee's accomplishment. It also shows the actual transaction that transpired

during the period covered.

6. C
A bankrupt company, the "debtor," might use Chapter 11 of the Bankruptcy
Code to "reorganize" its business and try to become profitable again.
Management continues to run the day-to-day business operations but all
significant business decisions must be approved by a bankruptcy court. Under
Chapter 7, the company stops all operations and goes completely out of
business. A trustee is appointed to "liquidate" (sell) the company's assets and the
money is used to pay off the debt, which may include debts to creditors and
investors.

7. A
secured creditor is any creditor or lender associated with an issuance of a credit
product that is backed by collateral. Secured credit products are backed by
collateral; in the case of a secured loan, collateral refers to assets that are
pledged as security for the repayment of that loan. In the event that a borrower
defaults on the repayment of a secured loan, the property is forfeited to the
secured creditor.

8. D

Bankruptcy Code, creditors with priority have priority over other unsecured

creditors without priority and to be paid first from any proceeds available to

unsecured creditors.
9. A

Expected recovery percentage or dividend to general unsecured

creditors refers to the interest of general unsecured creditors without priority.

The dividend is an estimate of how much will be received by general unsecured

creditors without priority for each peso owed to them, and is expressed either in

an absolute amount or in percentage form. It is computed as follows:

Estimated recovery percentage= Net Free Assets_

Total Unsecured Creditors without Priority

10. A

Non-dischargeable debt is a type of debt that cannot be eliminated

through a bankruptcy proceeding. Such debts include, but are not limited to,

student loans;

most federal, state, and local taxes; money borrowed on a credit card to pay

those taxes; and child support and alimony.

11. B

The statement of affairs provides information how much money each class

of creditors can expect to receive on liquidation of the company, assuming

assets are converted into cash at their estimated realizable values used in

preparing the statements.


12. C

The statement of affairs is a financial condition prepared for a corporation

entering into the stage of liquidation or bankruptcy. It is a report designed to

show the estimated amount that would be received by each class of claim

in the event of liquidation. It is essentially prepared on the basis of an

assumption of liquidation, that is a quitting concern, than a going concern

assumption.

13. C

Among the powers and duties of the trustee of the following:


1. Prepare and file in court a list of creditors of each class and their claims and
a list of stockholders of each class.
2. Investigate the acts, conduct, property, liabilities, and business operations of
the enterprise, consider the desirability of continuing operations, and formulate
a plan for such continuance for submission to the bankruptcy judge if
management of the debtor has not done so.
3. Report to the court (or Security and Exchange Commission) any facts
ascertained as to fraud against or mismanagement of the debtor’s enterprise.

14. B

In an involuntary petition, creditors initiate the action by filing a petition for

liquidation or reorganization with the bankruptcy court. If there are twelve or more

creditors, the petition must be signed by three or more of such creditors whose

claims aggregate at least either 1 million pesos or at least 25% of the subscribed

capital stock, whichever is higher, may apply for and seek the liquidation of an

insolvent corporation by filing a petition in court.


15. D

In a voluntary petition, the debtor files a petition with a bankruptcy court for

liquidation. The bankruptcy judge may refuse a voluntary petition if refusal is

considered to be in the best interest of the creditors.

16. A

Assets pledged to fully secured creditors are expected to realize an

amount at least sufficient to satisfy the related debt (estimated realizable value

(ERV) of an asset

≥ the liability secured).

17.A

A corporation that faces bankruptcy or liquidation is a quitting concern

because the company is not considered a going concern. As a result, a statement

of financial position, which reports the financial position of a going concern, is

inappropriate for a corporation in liquidation.

18.A

There is voluntary insolvency when the insolvent entity itself

petitions the Regional Trial Court that it be declared as insolvent.

19.B

The Statement of Realization and Liquidation is an activity statement

progress toward the liquidation of a debtor’s estate that shows the actual

transaction occurred
during the period covered and informs the proper authorities and interested

creditors of the accomplishments of the trustee.

20. D

Insolvency is defined as a financial condition such that the sum of such

entity debts is greater than all such entity’s property at fair valuation.

21. B

Unsecured liabilities with priority have priority under the law, Section 50 of the

insolvency law provides the following preferred claims which shall be pain in the

order named:

a. Necessary funeral expenses of the debtor, or his wife, or children, who are
under their parental authority and have no property of their own, when
approved by court;
b. Debts due for personal services rendered to insolvent by employees,
laborers, or domestic servants immediately preceding the commencement
of proceedings in solvency;
c. Compensation due to laborers of their dependents under the provisions of
Philippine Laws;
d. Legal expenses, and expenses incurred in the administration of insolvent’s
estate for the common interest of the creditors, when properly authorized
and approved by the court;
e. Debts, taxes, and assessments due the National Government;
f. Debts, taxes, and assessments due to any province or provinces of the
Philippines;
g. Debts, taxes, and assessments due to any municipality or municipalities of
the Philippines.

Based on letter (b) in the foregoing enumeration, premiums payable by the

employer arising from employment of workers and that will accrue to their

benefit must also be considered as liability with priority.


22. D

Difference between Statement of Realization and Liquidation and

Statement of Affairs: 1) the statement of realization and liquidation reports the

actual liquidation results. In contrast, the Statement of Affairs is of a pro forma

nature and is based on estimated rather than actual results. 2) The statement of

realization and liquidation provides an on-going reporting of the trustee’s activities

and is updated throughout the liquidation process. The Statement of Affairs is a

summary of the estimated results of a completed liquidation.

23. D

A return on interest may be expected to stockholders in the distribution of

dividends. Letters A and B are wrong as it should be measured at expected net

realizable value.

24. D

A Statement of Realization and Liquidation is an activity statement that is

intended to show progress toward the liquidation of a debtor’s estate. Its original

purpose was to inform the bankruptcy court and interested creditors of the

accomplishment of the trustee.


25. B

On the statement of realization and liquidation, the ₱20,000 would be

reported as gain as the liability was settled in full lower than the amount.
PROBLEMS

1. C

The notes payable to Land bank of the Philippines is considered partially

secured liabilities wherein a property with a fair market value of P450,000 is used

as collateral. Therefore, Land Bank of the Philippines is secured to receive

P450,000 because of the property while the balance P50,000 of the note is

unsecured.

2. C

The 60,000 owes to Maurer Corporation is considered a partially secured

liabilities. Accounts receivable with a realizable value of 45,000 is pledged to

secure

liability. Therefore, the estimated amount to be paid to Maurer Corp. would be as

follows:

Accounts receivable 45,000

Add: Portion of free asset used to pay unsecured amount

[(60,000 – 45,000) * 80%] 12,000

Estimated amount to be paid to partially secured liabilities 57,000

3. B

Since the Wang Corp. expect to recover .30 for every P1 of liability.

Therefore, the unsecured liability of Jackson Company that would be paid were

as follows:

Unsecured loan 200,000

Multiplied by: Expected recovery per peso of


30%
unsecured creditors

Total 60,000
4. C

Lebenpe Co. has a secured claim for the 25,000 liquidation value of the

machinery. The remaining 75,000 (100,000 – 25,000)is an unsecured claim.

Given that unsecured claims will be paid at the rate of .40 cents on the peso,

therefore, Lebenpe will receive:

Machinery at liquidation value

25,000 Add: Portion of free assets used to pay unsecured

amount

[(100,000 – 25,000) * .40] 30,000

Total 55,000

5. D

The total cash available to pay all unsecured claims, including priority

claims, is the cash obtained from free assets, ₱320,000 and any excess cash

available from assets pledged with fully secured creditors after they are used to

satisfy those claims (370,000-260,000= ₱110,000).

Therefore, the amount of cash to pay unsecured nonpriority claims:

Assets pledged to fully secured creditors, current value 370,000

Less: Fully secured creditors 260,000

Excess cash from assets pledged to fully secured creditors ₱110,000

Add: Free assets, current value 320,000

Total Free Assets 430,000

Less: Priority claims 70,000

Net Free Assets ₱360,000


6. B

Car- Cadillac, net realizable value

₱4,000,000 Add: Portion of free assets used to pay unsecured

amount

(6,000,000-4,000,000) *50% 1,000,000

₱5,000,000

7. C

To compute the estate deficit before the actual realization and

liquidation is simply to formulate the basic accounting equation. Therefore:

Assets, net realizable value

₱105,000 Less:

Liabilities per books ₱130,000

Add: Unrecorded interest 500 130,500

Estate (deficit) equity before realization and

Liquidation (25,500)

8. C

Partially secured creditors 65,000

Less: Assets pledged to partially secured creditors 30,000 35,000

Unsecured creditors 90,000

Total unsecured liabilities 125,000

Total free assets 110,000


Less: Unsecured liabilities with priority (25,000)

Net Free Assets 85,000


9. D

Cash available P100,000


Less: mortgage payable secured by property 50,000
Amount available to unsecured creditors P 50,000
Less: unsecured creditors with priority
Administrative expenses P 8,000
Salaries payable 2,000 10,000
Net free assets or amount available to unsecured
Creditors without priority P 40,000

Expected recovery percentage of unsecured creditors


P .50
P40,000 / (P80,000 - P50,000) + P50,000

Therefore, the cash is distributed as follows:

Unsecured creditors with priority P 10,000


Partially secured creditors:
Property at selling price P 50,000
Add: portion of free assets used to pay the
unsecured
Amount (P80,000 – P50,000) x 50% = P15,000 P 65,000
Unsecured creditors without priority (P50,000x.50) P 25,000
P100,000

10. D

Assets pledged to fully secured liabilities P100,000


Less: fully secured liabilities 80,000
Excess of assets pledged to fully secured liabilities
(free assets of fully secured liabilities) P20,000
Add: free assets 120,000
Total free assets to unsecured liabilities P140,000
Less: unsecured liabilities with priority 60,000
Net free assets P 80,000

Unsecured liabilities:
Partially secured liabilities P50,000
Less: assets pledged to partially secured
liabilities 40,000 P 10,000
Unsecured liabilities without priority 90,000
Total unsecured liabilities P100,000
Expected Recovery Percentage of Unsecured
Liabilities
P80,000 / P100,000 P .80

Therefore, the estimated amount to be paid to each creditor are:


Amount % of recovery
Fully secured liabilities P80,000 100% (80/80)
Unsecured liabilities with priority 60,000 100% (60/60)
Partially secured liabilities:
Assets P40,000
Add: portion of free assets to pay
unsecured
Creditors (50,000-40,000)x80% 8,000 48,000 96% (48/50)
Unsecured liabilities without priority:
(P90,000x80%) 72,000 80% (72/80)
Total P260,000*

*the total estimated amount to be paid to creditors of P260,000 can be

counterchecked by determining the total assets (at fair value):

Assets pledged to fully secured liabilities

P100,000 Assets pledged to partially secured liabilities

40,000 Free assets

120,000

P260,000
11. A

(1) Estimated deficiency to unsecured creditors:


Assets pledge to fully secured liabilities, @ fair value P75,000
Less: fully secured liabilities 30,000
Free assets of fully secured liabilities 45,000
Add: free assets, @ fair value 40,000
Total free assets to unsecured liabilities P85,000
Less: unsecured liabilities with priority 7,000
Net free assets P78,000
Less: unsecured liabilities:
Partially secured liabilities P60,000
Less: assets pledged to partially secured
liabilities, FV 52,000 8,000
Unsecured liabilities without priority 112,000
Total unsecured liabilities P120,000
Estimated deficiency to unsecured liabilities P42,000

Or, alternatively
Estimated (gain) loss on realization:
Loss on realization of assets pledged to fully
secured
Liabilities (P90,000-P75,000) P15,000
Loss on realization of asset pledged to partially
secured
Liabilities (P74,000-P52,000) 22,000
Loss or realization of free assets (P70,000-
P40,000) 30,000
Add: administrative expenses 0
Unrecorded expenses/liability 0
Total estimated net loss P67,000
Less: loss borne by the owners/stockholder’s equity:
Total assets at book value
(P90,000 + P74,000 + P70,000) P234,000
Less: Total liabilities
(P7,000 + P30,000 + P60,000 + P112,000) 209,000 25,000
Estimated deficiency to unsecured creditors P42,000

(2) Expected recovery per peso of unsecured claims:


P78,000 / P120,000 P .65
12. D

Total free assets P70,000


Less: unsecured creditors with priority:
Administrative expenses – accounting fees P 1,500
Unpaid income taxes 3,500 5,000
Net free assets P65,000

Total unsecured creditors without priority:


Total unsecured claims of all classes P105,000
Less: unsecured creditors with priority 5,000
Total unsecured creditors without priority P100,000

% of recovery: P65,000/P100,000 = 65%

Estimated payment to partially secured creditors:


Realizable value of A/R (60% x P4,000) P 2,400
Add: unsecured portion: 65% (P3,000 – P2,400) 390
Total P2,790

13. C

Assets to be realized 677,800 Assets Realized 205,600

Assets Assumed - Assets not realized 482,000*

Liabilities Liquidated 167,000 Liabilities to be Liquidated 880,000

Liabilities not Liquidated 713,000 Liabilities Assumed -

Supplementary Debits 9,100 Supplementary Credits -

1,566,900 1,567,600

Net Income 700

Estate Deficit (204,800)

Estimated Deficiency (204,100)

*Assets to be realized 677,800

Assets realized (205,600)


Increase in assets Copyrights 53,200

Decrease in assets Machinery (43,400) 9,800

Assets not realized 482,000

14. C

Asset to be realized P

820,000

Less: Asset not realized

280,000

Add: Gain on realization 20,820

Asset realized P 560,820

15. D

Assets to be realized 610,000 Assets Realized 700,000


Assets Acquired 640,000 Assets not realized 430,000
Liabilities Liquidated 640,000 Liabilities to be Liquidated 820,000
Liabilities not Liquidated 730,000 Liabilities Assumed 460,000
Supplementary Debits 748,000 Supplementary Credits 790,000
3,368,00 3,200,00
0 0
Net loss 168,000

16. A

Partially secured creditors P 215,000


Less: Assets pledged to partially secured 180,000 P 215,000
Unsecured creditors 240,000
Total unsecured liabilities P 275,000

Total Free assets P 260,000


Less: Unsecured liabilities with priority (175,000)
Net Free assets P 85,000
Estimated deficiency to unsecured creditors 190,000
Total Unsecured liabilities P 275,000
17. A

Realizable value of all assets ($585,000 + $250,000 + $290,000)

$1,125,000 Allocated to:

Fully secured creditors (266,000)

Partially secured creditors (250,000)

Unsecured creditors with priority (50,000)

Remainder available to general unsecured creditors $ 559,000

Payment rate to general unsecured creditors

(Including balance due to partially secured creditors)

$559,000 / ($1,115,000 + ($350,000 - $250,000) 46 %

Realizable value of assets:

Assets pledged to fully secured creditors $585,000

Assets pledged to partially secured creditors 250,000

Free assets 290,000

Total realizable value $1,125,000

Amounts to be paid to:


Fully secured creditors $266,000
Partially secured creditors
[$250,000 + (0.46 × $100,000)] 296,000
Unsecured creditors with priority

50,000 General unsecured creditors (0.46 × $1,115,000)

513,000* Total

$1,125,000
*Round off
18. D

(₱500,000 + ₱100,000) – ₱200,000 = ₱400,000

19. B

(₱95,000 + ₱17,000) – ₱49,000 = ₱63,000

20. D

Estimated losses on realization of assets ₱2,000,000


Less: Estimated gains on realization of assets ₱1,440,000
Additional assets* 1,280,000 2,720,000
Estimated net (gain) or loss in assets (₱720,000)
realization
Add: Additional liabilities** 960,000
Estimated net (gain) or loss ₱240,000
Less: Stockholder’s equity
Capital stock ₱2,000,000
Deficit 800,000
1,200,000
Estimated amount to be recovered by ₱560,000
stockholders

Therefore, the pro-rate payment on the peso is:


𝑃𝑟𝑜 − 𝑟𝑎𝑡𝑒𝑝𝑎𝑦𝑚𝑒𝑛𝑡 𝑜𝑛 𝑡ℎ𝑒 𝑝𝑒𝑠𝑜
𝐸𝑠𝑡𝑖𝑚𝑎𝑡𝑒𝑑 𝑎𝑚𝑜𝑢𝑛𝑡 𝑡𝑜 𝑏𝑒 𝑟𝑒𝑐𝑜𝑣𝑒𝑟𝑒𝑑 𝑏𝑦 𝑠𝑡𝑜𝑐𝑘ℎ𝑜𝑙𝑑𝑒𝑟𝑠
=
𝑆𝑡𝑜𝑐𝑘ℎ𝑜𝑙𝑑𝑒𝑟𝑠′𝐸𝑞𝑢𝑖𝑡𝑦
₱560,000
₱800,000 =
= P .70

*Additional assets are assets completely written-off in the books in the past

years but subsequently have a realizable value.

**Additional liabilities are liabilities in addition to the recorded liabilities in the


balance sheet. In other words, they are unrecorded liabilities and expenses.
Examples are liquidation expenses such as administrative and trustee fees,
liability on damage suits, acquired interest on mortgage payable, unbilled
creditor’s fees, and the like.
21. C

Statement of Realization and Liquidation Credits:


Assets realized ₱1,200,000
Assets not realized 1,375,000
Liabilities to be liquidated 2,250,000
Liabilities assumed 1,625,000
Supplementary credits* 2,800,000
Total credits ₱9,250,000

Statement of Realization and Liquidation Debits:

Assets to be realized ₱1,375,000


Assets acquired 750,000
Liabilities liquidated 1,875,000
Liabilities not liquidated 1,700,000
Supplementary charges** 3,125,000
Total debits ₱8,825,000
Net gain for the three-month period
₱425,000

*Supplementary credits are revenue or income items such as sales,

interest income, etc.

**Supplementary debits are cost and expense items such as purchases,

expenses, etc.
22. C

Assets = Liabilities + Stockholder’s Equity

Common stock ₱1,500,000


Deficits (500,000)
Stockholder’s equity (SHE) ₱1,000,000
Add: Liabilities not liquidated 1,700,000
Total Liabilities and SHE ₱2,700,000
Less: Assets not realized (or end) 1,375,000
Cash balance, ending ₱1,325,000

23. D

Unsecured debits ₱200,000

Free assets ₱100,000

(80,000) 20,000

Deficiency ₱ 180,000

24.B

Partially secured creditors ₱60,000


Less: Assets pledged to partially secured creditors 30,000 ₱ 30,000
Unsecured creditors 90,000
Total unsecured liabilities ₱120,000
Total free assets ₱ 90,000
Less: Unsecured liabilities with priority (25,000)
Net free assets ₱ 65,000
Estimated deficiency to unsecured creditors 55,000
Total unsecured liabilities ₱ 120,000

25. C

(P60,000 + P15,000) - P25,000 = P50,000


CONSTRUCTION CONTRACTS

THEORIES

1. When work to be done and costs to be incurred on a long-term contract can be


estimated dependably, which of the following methods of revenue recognition
is preferable?
a. Instalment-sales method
b. Percentage-of-completion method
c. Completed-contract method
d. None of these
The percentage-of-completion method applies the accrual assumption. This
method is used when the outcome of the construction contact can be estimated
reliably.
2. In accounting for a long-term construction-type contract using the
percentage-of- completion method. The gross profit recognized during the first
year would be the estimated total gross profit from the contract, multiplied by the
percentage of the costs incurred during the year to the
a. Total costs incurred to date
b. Total estimated cost
c. Unbilled portion of the contract price
d. Total contract price

The percentage of completion is computed as follows:


Cost incurred to date
= Percent complete
Total estimated cost (Most recent)

3. How should earned but unbilled revenues at the balance sheet date on a
long- term construction contract be disclosed if the percentage-of-completion
method of revenue recognition is used?
a. As construction in process in the current asset section of the
balance sheet
b. As construction in process in the noncurrent asset section of the balance sheet
c. As a receivable in the noncurrent asset section of the balance sheet
d. In a note to the financial statements until the customer is formally billed
for the portion of work completed.
Construction in process is reported in the statement of financial position as
“Current asset-Contract asset” as it comprises total costs incurred on the contract,
plus the cumulative recognized profit (or less cumulative recognized loss), less
progress billings.
4. In selecting an accounting method for newly contracted long-term
construction project, the principal factor to be considered should be?
a. The terms of payment in the contract
b. The degree to which a reliable estimate of the costs to complete and
extent of progress toward completion is practicable
c. The method commonly used by the contractor to account for other long-
term construction contracts.
d. The inherent nature of the contractor’s technical facilities used in construction
In order that there is a reliable measurement, allows contract revenue and costs
to be recognized when the outcome of the contract can be predicted or when it is
probable that the economic benefits attached to the contract will flow to the
enterprise.

5. When the outcome of a construction contract cannot be estimated reliably,


what accounting method shall be used by the long-term constructor for the
recognition of construction revenue and construction cost?
a. Percentage of completion method
b. Cost recovery method
c. Instalment method
d. Accrual basis
Cost recovery method of construction accounting is used when the contract’s
outcome cannot be reliably estimated.
6. The percentage-of-completion method of accounting for long-term
construction contracts is an exception to the
a. Matching principle.
b. Going-concern assumption.
c. Economic-entity assumption.
d. Revenue recognition principle.

The revenue recognition principle states that revenue should be recognized


(recorded) when realized or realizable and earned. Revenue is earned when the
earning process is essentially complete. In effect, revenue is recorded when the
most important event in the earning of that revenue has occurred. Thus, revenue
is normally recorded at the time of the sale or, occasionally, at the time cash is
collected. However, sometimes neither the sales basis nor the cash basis is
appropriate, such as when a construction contract extends over several
accounting periods. As a result, contractors ordinarily recognize revenue using
the percentage-of-completion method so that some revenue is recognized each
year over the life of the contract. Hence, this method is an exception to the
general principle of revenue recognition, primarily because it better matches
revenues and expenses.

Answer (A) is incorrect because the percentage-of-completion method attempts


to match revenues and expenses with the appropriate periods. Answer (B) is
incorrect because the going-concern assumption is appropriate for a contractor
using the percentage-of-completion method, as for any other type of company.
Answer (C) is incorrect because the economicentity assumption is appropriate for
a contractor using the percentage-of-completion method, as for any other type
of company.

7. The theoretical support for using the percentage-of-completion method


of accounting for longterm construction projects is that it
a. is more conservative than the completed-contract method.
b. reports a lower Net Income figure than the completed-contract method.
c. more closely conforms to the cost principle.
d. produces a realistic matching of expenses with revenues.

The completed-contract method does not recognize any gross profit until the
contract is completed. The percentage-of-completion method recognizes a
portion of revenues and gross profit each period, based upon the ratio of costs
incurred to date to total estimated costs of completion. Accumulated gross profit
and accumulated construction costs are included in the construction in progress
inventory account under the percentage-ofcompletion method.

Answer (A) is incorrect because progress billings are accumulated in the billings
on construction in progress account under both methods. Answer (B) is incorrect
because accumulated construction costs are included in the construction in
progress inventory account under both methods. Answer (C) is incorrect
because the percentage-of- completion method recognizes a percentage of
revenues and gross profit each period.

8. The principal disadvantage of using the percentage-of-completion method


of recognizing revenue from long-term contracts is that it
a. is unacceptable for income tax purposes.
b. gives results based upon estimates which may be subject to considerable
uncertainty.
c. is likely to assign a small amount of revenue to a period during which much
revenue was actually earned.
d. none of these.

(D) GAAP requires that revenue be recognized when it is realized or


realizable and earned. Under the completed-contract method, revenue recognition
is appropriate only at the completion of the contract. Neither the recording nor the
collection of progress billings affects this recognition.

Answers (A), (B), and (C) are incorrect because neither the issuance of a
progress billing (debit accounts receivable, credit progress billings) nor the
collection of cash (debit cash, credit accounts receivable) results in
recognition of income.

9. The theoretical support for using the percentage-of-completion method


of accounting for long-term construction projects is that it
a. is more conservative than the completed-contract method.
b. reports a lower Net Income figure than the completed-contract method.
c. more closely conforms to the cost principle.
d. produces a realistic matching of expenses with revenues.

A. Under the completed-contract method, the gross profit on the contract should
be recognized upon the completion of the contract. If a loss is anticipated,
however, the loss should be recognized immediately. Under GAAP, the entries to
record progress billings and their collection do not affect the recognition of profit
or loss. Thus, the 3rd year progress billings have not effect on the income
statement, but the loss anticipated
in the 3rd year should be recognized in full in that year. Answers (B), (C), and (D)
are incorrect because, under the completed-contract method, progress billings
have not effect on the recognition of income, and an anticipated loss should be
recognized in the year it occurs.

10. The principal disadvantage of using the percentage-of-completion method


of recognizing revenue from long-term contracts is that it
a. is unacceptable for income tax purposes.
b. gives results based upon estimates which may be subject to considerable
uncertainty.
c. is likely to assign a small amount of revenue to a period during which much
revenue was actually earned.
d. none of these.

D. GAAP requires that revenue be recognized when it is realized or realizable and


earned. Under the completed-contract method, revenue recognition is
appropriate only at the completion of the contract. Neither the recording nor
the collection of progress billings affects this recognition. Answers (A), (B), and
(C) are incorrect because neither the issuance of a progress billing (debit
accounts receivable, credit progress billings) nor the collection of cash (debit
cash, credit accounts receivable) results in recognition of income.

11. The accounting method most clearly consistent with basic revenue
recognition principles is the
A. Percentage-of-completion method.
B. Installment sales method.
C. Completion-of-production method.
D. Completed-contract method.

Answer (D) is correct. According to the revenue recognition principle, revenue


should be recognized when (1) realized or realizable and (2) earned. Under the
completed- contract method, revenue is not recognized until a long-term
construction contract is complete. At this stage, the entity is most clearly entitled
to the resulting revenues and is most likely to have been involved in an
exchange.

Answer (A) is incorrect because the percentage-of-completion method allows for


revenue to be recognized at various stages of the contract although the entire job
is not complete. Answer (B) is incorrect because, if the collectibility of assets is
relatively uncertain, revenues and gains may be recognized as cash is received
using the installment sales method. Answer (C) is incorrect because the
completion-of-production method is an appropriate basis for recognition if
products or other assets are readily realizable, e.g., precious metals and some
agricultural products.

12. Under the completed-contract method


a. revenue, cost, and gross profit are recognized during the production cycle.
b. revenue and cost are recognized during the production cycle, but gross
profit recognition is deferred until the contract is completed.
c. revenue, cost, and gross profit are recognized at the time the contract is completed.
d. none of these.

B. Under the completed-contract method of accounting for long-term construction


contracts, recorded progress billings have no effect on the recognition of
income. Answers (A), (C), and (D) are incorrect because under the completed-
contract method, progress billings are recorded when issued and removed at
the completion of the contract.

13. If the completed-contract method is used, what is the basis for


determining the income to be recognized in the second year of a three-year
contract?
a. Cumulative actual costs incurred only.
b. Incremental cost for the second year only.
c. Latest available estimated costs.
d. No income would be recognized in year 2.

A. According to ARB 45, the percentage-of-completion method provides for the


recognition of income based on the relationship between costs incurred to date
and estimated total costs for completion of the contract. (But ARB 45 permits any
other measure of progress “as may be appropriate having due regard to work
performed.”) The amount of income recognized in the 3rd period of a 5-year
contract is calculated as follows: The total anticipated income (based on the latest
available estimated costs) is multiplied by the ratio of costs incurred to date to the
latest available total estimated costs, and the product is reduced by the previously
recognized income.

Answer (B) is incorrect because the ratio of total costs incurred to date to total
billings to date is not relevant. Answer (C) is incorrect because total costs incurred
must be used. Answer (D) is
incorrect

14. The percentage-of-completion and the completed-contract


methods of accounting for longterm construction
projects in progress differ in that
a. It is only under the percentage-of-completion method that progress
billings are accumulated in a contra-inventory account called billings on
construction in progress.
b. It is only under the completed-contract method that accumulated construction
costs are included in a construction in progress inventory account.
c. Only the percentage-of-completion method recognizes all revenues and gross
profit on the contract when the contract is completed.
d. It is only under the percentage-of-completion method that gross profit
earned to date is accumulated in the construction in progress inventory
account.

The completed-contract method does not recognize any gross profit until the
contract is completed. The percentage-of-completion method recognizes a portion
of revenues and gross profit each period, based upon the ratio of costs incurred to
date to total estimated costs of completion. Accumulated gross profit and
accumulated construction costs are
included in the construction in progress inventory account under the percentage-
ofcompletion method.

Answer (A) is incorrect because progress billings are accumulated in the billings
on construction in progress account under both methods. Answer (B) is incorrect
because accumulated construction costs are included in the construction in
progress inventory account under both methods. Answer (C) is incorrect because
the percentage-of- completion method recognizes a percentage of revenues and
gross profit each period.

15. Which of the following is not a difference between the percentage-of


completion and completed-contract methods of accounting for long-term
construction contracts?
a. They report different amounts for inventory during the construction period.
b. They report different amounts for progress billings during the construction period.
c. They cause a different cash inflow during the construction period.
d. They report different amounts for accounts receivable during the construction period.

Under the completed-contract method, the gross profit on the contract should be
recognized upon the completion of the contract. If a loss is anticipated, however,
the loss should be recognized immediately. Under GAAP, the entries to record
progress billings and their collection do not affect the recognition of profit or loss.
Thus, the 3rd year progress billings have not effect on the income statement, but
the loss anticipated in the 3rd year should be recognized in full in that year.

Answers (B), (C), and (D) are incorrect because, under the completed-contract
method, progress billings have not effect on the recognition of income, and an
anticipated loss should be recognized in the year it occurs.

16. How should the balances of progress billings and construction in progress be
shown at reporting dates prior to the completion of a long-term contract?
a. Progress billings as deferred income, construction in progress as a deferred expense.
b. Progress billings as income, construction in progress as inventory.
c. Net, as a current asset if debit balance and current liability if credit balance.
d. Net, as income from construction if credit balance, and loss from construction
if debit balance.

ARB 45, Long-Term Construction-Type Contracts, requires that the difference


between construction in progress (costs and recognized income) and progress
billings to date be shown as a current asset if construction in progress exceeds
total billings, and as a current liability if billings exceed construction in progress.
Separate recognition is required for each project.

17. How should earned but unbilled revenues at the balance sheet date on a long-
term construction contract be disclosed if the percentage-of-completion method of
revenue recognition is used?
a. As construction in progress in the current asset section of the balance sheet.
b. As construction in progress in the noncurrent asset section of the balance sheet.
c. As a receivable in the noncurrent asset section of the balance sheet.
d. In a note to the financial statements until the customer is formally billed
for the portion of work completed.

Answer (D) is correct. According to the revenue recognition principle, revenue


should be recognized when (1) realized or realizable and (2) earned. Under the
completed- contract method, revenue is not recognized until a long-term
construction contract is complete. At this stage, the entity is most clearly entitled
to the resulting revenues and is most likely to have been involved in an
exchange.

Answer (A) is incorrect because the percentage-of-completion method allows for


revenue to be recognized at various stages of the contract although the entire job
is not complete. Answer (B) is incorrect because, if the collectibility of assets is
relatively uncertain, revenues and gains may be recognized as cash is received
using the installment sales method. Answer (C) is incorrect because the
completion-of-production method is an appropriate basis for recognition if
products or other assets are readily realizable, e.g., precious metals and some
agricultural products.

18. If a company uses the completed-contract method of accounting for long-term


construction contracts, then during the period of construction, financial information
related to a long-term contract will
a. appear on both the income statement and balance sheet during the
construction period.
b. appear only on the income statement during the period of construction.
c. appear only on the balance sheet during the period of construction.
d. not appear on the financial statements.

Answer (B) is correct. Under the percentage-of-completion method, a current-


period loss on a profitable contract is treated as a change in accounting estimate.
Thus, a current-period adjustment is required. Prior-period adjustments are made
to correct errors, not to reflect changes in estimates. Answer (A) is incorrect
because, under the percentage-of-completion method, a current-period loss on a
profitable contract requires a current-period adjustment. Answer (C) is incorrect
because, under the completed- contract method, no profit is recognized until the
contract is completed. Cost estimate adjustments while construction is in progress
do not result in profit or loss recognition prior to completion unless an overall loss
is expected on the contract. Answer (D) is incorrect because, under the
completed-contract method, no profit is recognized until the contract is
completed. Cost estimate adjustments while construction is in progress do not
result in profit or loss recognition prior to completion unless an overall loss is
expected on the contract.

19. The percentage-of-completion method of accounting for long-term


construction contracts is an exception to the
A. Matching principle.
B. Going-concern assumption.
C. Economic-entity assumption.
D. Revenue recognition principle.

The revenue recognition principle states that revenue should be recognized


(recorded) when realized or realizable and earned. Revenue is earned when the
earning process is essentially complete. In effect, revenue is recorded when the
most important event in the earning of that revenue has occurred. Thus, revenue
is normally recorded at the time of the sale or, occasionally, at the time cash is
collected. However, sometimes neither the sales basis nor the cash basis is
appropriate, such as when a construction contract extends over several
accounting periods. As a result, contractors ordinarily recognize revenue using the
percentage-of-completion method so that some revenue is recognized each year
over the life of the contract. Hence, this method is an exception to the general
principle of revenue recognition, primarily because it better matches revenues and
expenses.

Answer (A) is incorrect because the percentage-of-completion method attempts


to match revenues and expenses with the appropriate periods. Answer (B) is
incorrect because the going-concern assumption is appropriate for a contractor
using the percentage-of-completion method, as for any other type of company.
Answer (C) is incorrect because the economicentity assumption is appropriate for
a contractor using the percentage-of-completion method, as for any other type of
company.

20. The percentage-of-completion method of accounting for long-term


construction contracts is an exception to the
a. Matching principle.
b. Going concern assumption.
c. Historical cost principle.
d. Revenue recognition principle.

Revenue is recognized when realized or realizable and the earning process is


substantially complete. This ordinarily occurs at the time of sale and delivery of
goods or services. Thus, the percentage-of-completion method is essentially an
exception to the revenue recognition principle. Production rather than sale and
delivery is considered to be the culmination of the earning process.

Answer (A) is incorrect because the percentage-of-completion method attempts a


more accurate association of cost incurrence and revenue recognition. Answer
(B) is incorrect because the percentage-of-completion method is completely
consistent with the going concern assumption. Answer (C) is incorrect because
the percentage-of-completion method is completely consistent with the historical
cost principle.

21. Although a transfer of ownership has not occurred, the percentage-of-


completion method is
acceptable under the revenue recognition principle because
a. The assets are readily convertible into cash.
b. The production process can be readily divided into definite stages.
c. Cash has been received from the customer.
d. The earning process is completed at various stages

SFAC 5 states that revenue should be recognized when it is both realized or


realizable and earned. If a project is contracted for before production and covers
a long time period in relation to reporting periods, revenues may be recognized by
a percentage of- completion method as they are earned (as production occurs),
provided reasonable estimates of results at completion and reliable measures of
progress are available. Thus, contractors traditionally use the percentage-of-
completion method because some revenue can be recognized during each period
of the production process. In a sense, the earning process is completed in
various stages; thus, revenues should be recorded in each stage.

Answer (A) is incorrect because, depending upon the terms of the contract, the
assets may not be readily convertible into cash. Answer (B) is incorrect because,
on a large construction project, the production process often cannot be easily
divided into definite stages. Answer (C) is incorrect because cash is sometimes
not received until the project is completed.

22. A building contractor has a fixed-price contract to construct a large building. It


is estimated that the building will take 2 years to complete. Progress billings will
be sent to the customer at quarterly intervals. Which of the following describes the
preferable point for revenue recognition for this contract if the outcome of the
contract can be estimated reliably?
a. After the contract is signed.
b. As progress is made toward completion of the contract.
c. As cash is received.
d. When the contract is completed.

Under the percentage-of-completion method, revenues and expenses are


recognized based on the stage of completion at the balance sheet date if the
outcome of the contract can be estimated reliably. For a fixed-price contract, the
outcome can be estimated reliably if (1) total revenue can be measured reliably,
(2) it is probable that the economic benefits of the contract will flow to the
enterprise, (3) contract costs to complete and stage of completion can be
measured reliably, and (4) contract costs can be clearly identified and measured
reliably so that actual and estimated costs can be compared.

Answer (A) is incorrect because revenue is not recognized until progress has
been made toward completion. Answer (C) is incorrect because the cash basis is
inappropriate. An accrual method, that is, the percentage-of-completion method,
should be used. Answer (D) is incorrect because the completed-contract method
is not a permissible method.
23. A Rationale for recognizing revenue over the life of a contract rather than at a
single point in time is that:
a. Results are more conservative
b. It provides a better measure of periodic accomplishment
c. It is a better match with legal ownership
d. It results in a lower income tax

Recognizing revenue over time better conveys the benefit to the company of satisfying
performance obligations over time.

24. Revenue on a long-term contract should not be recognized according to the


proportion of the performance obligation that has been completed if:
a. Completion rates are certain
b. Profits are low
c. Projects are more than five years to completion
d. The arrangement does not qualify for revenue recognition over time.

The same Three criteria for determining whether it is appropriate to recognize


revenue over time apply to long-term contracts as apply to other contracts

25. When accounting for revenue over time for a long-term contract, the
percentage of completion used to recognize revenue in the first year usually is
determined by measuring
a. Cost incurred in the first year, divided by estimated remaining cost to
complete the project
b. Cost incurred in the first year, divided by estimated total cost for the
completed project
c. Cost incurred in the first year, divided by estimated gross profit
d. Cost incurred in the first year, divided by estimated total cost to be incurred
in the remaining years of the project.

Use of a “cost-to-cost” ratio to estimate percentage of completion is typical


PROBLEMS

Hurricane Builders, Inc. employs the cost-to-cost method in determining the


percentage- of-completion for revenue recognition. The company’s records show
the following information on a recently completed project for a contract price of
P5,000,000.
2006 2007 2008
Cost incurred to date P900,000 P2,550,000
Gross profit or loss 100,000 350,000 (P50,000)

1. The estimated costs to complete the project at December 31,


2007. a. P850,000
b. P1,700,000
c. P2,300,000
d. P2,550,000

2. The actual costs incurred during the year


2008. a. P2,550,000
b. P2,300,000
c. P2,200,000
d. P2,050,000

Solution for 1 and 2:


2006 2007 2008
Contract Price P5,000,000 P5,000,000 P5,000,000
Cost incurred each year P900,000 1,650,000 2,050,000
Add: Cost incurred in prior year - 900,000 2,550,000
Costs incurred to date (1) P900,000 P2,550,000 P4,600,000
Add: Estimated costs to complete 1,700,000 -
Total estimated costs (2) 4,250,000 P4,600,000
Estimated gross profit 750,000 400,000
Multiply by: percentage of completion (1/2) 60% 100%
Recognized gross profit to date 100,000 P450,000 P400,000
Less: Recognized gross profit in prior years - -100,000 -450,000
Recognized gross profit each year P100,000 P350,000 (P50,000)

Eaton Construction Co. uses the percentage-of-completion method. In 2007,


Eaton began work on a contract for $3,300,000 and it was completed in
2008. Data on the costs are:
Year Ended December 31
2007 2008
Costs incurred $1,170,000 $840,000
Estimated costs to complete 780,000 —
For the years 2007 and 2008, Eaton should recognize gross profit of
2007 2008
a. $0 $1,290,000
b. $774,000 $516,000
c. $810,000 $480,000
d. $810,000 $1,290,000

3. Reese Construction Corporation contracted to construct a building for


$1,500,000. Construction began in 2007 and was completed in 2008. Data
relating to the contract are summarized below:

Year ended, December 31

2007 2008
Cost incurred $600,000 $450,000
Estimated costs to complete 400,000 -

Reese uses the percentage of completion method as the basis for income
recognition. For the years ended December 31, 2007, and 2008, respectively.
Reese should report gross profit of

a. $270,000 and $180,000


b. $900,000 and $ 600,000
c. $300,000 and $150,000
d. $0 and $450,000

Solution:
2007 2008
Contract Price 1,500,000 1,500,000
Cost incurred each year 600,000 450,000
Add: Cost incurred in prior year - 600,000
Costs incurred to date (1) 600,000 1,050,000
Add: Estimated costs to complete 400,000 -
Total estimated costs (2) 1,000,000 1,050,000
Estimated gross profit 500,000 450,000
Multiply by: percentage of completion (1/2) 60% 100%
Recognized gross profit to date 300,000 450,000
Less: Recognized gross profit in prior years - 300,000
Recognized gross profit each year 300,000 150,000
4. Winsor Construction Company uses the percentage of completion
method of accounting. In 2007, Winsor began work on a contract it had
received which provided for a contract price of $15,000,000. Other details
follow:
2007
Costs incurred during the year $7,200,000
Estimated costs to complete as of December 31 4,800,000
Billings during the year 6,600,000
Collections during the year 3,900,000

What should be the gross profit recognized in

2007? a. $600,000
b. $7,800,000
c. $1,800,000
d. $3,000,000

Solution:
2007
Contract Price 15,000,000
Cost incurred each year 7,200,000
Add: Cost incurred in prior year -
Costs incurred to date (1) 7,200,000
Add: Estimated costs to complete 4,800,000
Total estimated costs (2) 12,000,000
Estimated gross profit 3,000,000
Multiply by: percentage of completion (1/2) 60%
Recognized gross profit to date 1,800,000
Less: Recognized gross profit in prior years -
Recognized gross profit each year 1,800,000

5. In 2007, Crane Corporation began construction work under a three-year


contract. The contract price is $2,400,000. Crane uses the percentage of
completion method for financial accounting purposes. The income to be
recognized each year is based on the proportion of costs incurred to total
estimated costs for completing the contract. The financial statement presentations
relating to this contract at December 31, 2007 is as follows:

Balance Sheet
Accounts receivable - construction contract billings 100,000
Construction in progress 300,000
Less: Contract billings 240,000
Costs and recognized profit in excess of billings 60,000
Income Statement
Income (before tax) in the contract recognized in 2007 60,000

How much cash was collected in 2007 on this


contract? a. $100,000
b. $140,000
c. $20,000
d. $240,000

Solution:
Contract billings 240,000
Less: Accounts receivable - Contract billings 100,000
Cash received 140,000

6. Mediocre Inc. has entered into a very profitable fixed price contract for
constructing a high-rise building over a period of three years. It incurs the
following costs relating to the contract during the first year:

Cost of material = 2.5


million Site labor cost
= 2.0 million
Agreed administrative cost to be reimbursed by the customer = 1
million Depreciation of the plant used for the construction = 0.5
million Marketing costs for selling apartments, when they are
ready = 1.0 million Total estimated cost of the project = 18 million

The percentage of completion of this contract at the year-end is:


a. 33 1/3%
b. 27%
c. 25%
d. 39%

Solution:
Costs incurred to date:
Materials 2,500,000
Direct Labor

2,000,000 Overhead
Administrative costs reimbursable 1,000,000
Depreciation during construction 500,000
6,000,000
Divided by: Total Estimated Cost

18,000,000 Percentage of completion 33


1/3%
7. Miracle Construct, Inc. is executing a gigantic project of constructing the
tallest building house in the country. The project is expected to take three years to
complete. The company has signed a fixed price contract of 12,000,000 for the
construction of this prestigious boarding house. The details of the cost incurred to
date in 2020 are:

Site labor cost 1,000,000


Cost of construction material

3,000,000 Depreciation of special plant and equipment used in


Constructing to build the boarding house

500,000 Marketing and selling costs to get the boarding house


In the country the right exposure

1,000,000 Total

5,000,000
Total contract cost estimated to complete 5,500,000

Calculate the revenue costs and profit to be recognized in 2020.

Revenue Cost Gross Profit


a. 5,400,000 4,500,000 900,000
b. 5,400,000 5,500,000 (100,000)
c. 6,000,000 4,500,000 1,500,000
d. 6,000,000 5,500,000 900,000

Solution:
Contract Price 12,000,000
Less: Total Estimated Cost:
Cost Incurred to date 4,500,000
Estimated cost to complete 5,500,000
Total Estimated Cost 10,000,000
Estimated Gross Profit 2,000,000
% of completion (4,500,000/10,000,000) 45%
Recognized gross profit in 2020 900,000
Recognized revenue in 2020 (12,000,000x45% 5,400,000
Costs recognized in 2020 (10,000,000x45%) 4,500,000

8. The company signed an 800,000 contract to build an environmentally friendly


access trail to Morayta, Manila. The project was expected to take approximately 3
years. The following information was collected for each year of the project – Year
1, Year 2, and Year 3:

Year 1 Year 2 Year 3


Cost to Expended during the year 100,000 150,000 250,000
Expected additional cost to completion 450,000 280,000 0
Support timbers laid during the year 150 300 500
Additional support timbers to be laid 850 520 0
Trail feet constructed during the year 3,000 7,500 8,000
Additional trail feet to be constructed 15,200 8,200 0
Compute the amount of revenue to be recognized in Year 3, assume that the
company employs the efforts-expended method of estimating percentage of
completion and the company measures its progress by the number of support
timbers laid in the trail:

a. 428,866
b. 422,640
c. 350,000
d. cannot be determined

Solution:
Input Measures: Efforts- Expended Method- using timbers laid
Year 2 Year 3
Timbers laid each year 300 500
Add: Timber laid in prior years 150 450
Timbers laid to date 450 950
Add: Additional support timbers to be laid 520 0
Total Estimated Timbers 970 950
Percentage of Completion 45/97 100%
X: Contract Price 800,000 800,000
Recognized Revenue to Date 371,134 800,000
Recognized Revenue in prior years 371,134
Recognized Revenue in Current 428,866
Year

9. Using the same information in No. 3, compute the amount of revenue to be


recognized in Year 3, assume that the company employs an output measure and
the company measures its progress by the number of trail feet that have been
completed:

a. 428,864
b. 422,640
c. 350,802
d. Cannot be determined

Solution:
Output measures – Number of trail feet
Year 2 Year 3
Timbers laid each year 7,500 8,000
Add: Trail feet in Prior Years 3,000 10,500
Trail feet to date 10,500 18,500
Add: Additional trail feet to be constructed 8,200 0
Total Estimated Trail feet 18700 18,500
Percentage of Completion 10.5/18.7 100%
X: Contract Price 800,000 800,000
Recognized Revenue to Date 449,198 800,000
Recognized Revenue in prior years 449198
Recognized Revenue in Current 350,802
Year
10. Dante Construction Company uses the percentage-of-completion method of
accounting. During 2020, Dante contracted to build an apartment house for Rizza
for 10,000,000. Dante estimated that the total costs would amount to 8,000,000
over the period of construction. In connection with this contract, Dante incurred
1,000,000 of construction during 2020. Dante billed and collected 1,500,000 from
Rizza in 2020. How much gross profit should Dante recognize in 2020?

a.
300,
000
b.25
0,00
0 c.
187,
500
d. 125,000

Solution:
Contract price 10,000,000
Less: Total Estimated Costs 8,000,000
Estimated gross profit 2,000,000
Multiplied by: percentage of completion (1M/8M)

12.5% Recognized gross profit to date

250,000 Less: Recognized gross profit in prior year 0 Recognized gross profit in
current year – 2020

250,000

11. DJ Builders Inc. has consistently used the percentage of completion method
of accounting for construction type contracts. During 2020, DJ started work on a
9,000,000 fixed price construction contract that was completed in 2021. DJ’s
accounting records disclosed the following:

12/31/2020 12/31/2021
Cumulative contract costs incurred 3,900,000
6,300,000
Estimated total costs at completion 7,800,000
8,100,000

How much income would DJ have recognized on this contract for the year
ended December 31, 2020?

a. 100,000
b. 300,000
c. 600,000
d. 700,000

Solution: 2020 2021


Contract Price 9,000,000 9,000,000
Less: Total Estimated Costs 7,800,000 8,100,000
Estimated gross profit 1,200,000
900,000 Multiplied by:
percentage of completion 39/78
63/81 Recognized
gross profit to date 600,000
700,000 Less:
Recognized gross profit in prior years -
600,000 Recognized
gross profit in each year 600,000
100,000
12. Ube Construction Company has consistently used the percentage of
completion method. On January 10, 2020, Ube began work on a 6,000,000
construction contract. At the inception date, the estimated cost of construction
was 4,500,000. The following data relate to the progress of the contract:

Income recognized at 12/31/2020 600,000


Cost incurred 1/10/2020 through 12/31/2021 3,600,000
Estimated cost to complete at 12/31/2021 1,200,000

How much income should Ube recognized for the year ended December 31, 2021?

a. 300,000
b. 525,000
c. 600,000
d. 900,000

Solution:
Contract Price 6,000,000
Less: Total Estimated Costs:
Cost incurred to 1/10/2020 – 12/31/2021 3,600,000
Estimated costs to complete 1,200,000 4,800,000
Estimated gross profit 1,200,000
Multiplied by: percentage of completion 36/48
Recognized gross profit 900,000
Less: Recognized gross profit in prior year 600,000
Recognized gross profit in current year – 2021 300,000

13. The following information related to a flood control project of JJD


Construction Co. which was started in 2020 and completed in 2021:

Cost incurred to date: Estimate total cost on


completion: As at June 30, 2020 9,750,000 As at June 30, 2020
19,500,000
As at June 30, 2021 15,750,000 As at June 30, 2021 20,250,000

The project is a 22,500,000 fixed-price construction contract, and JJD uses the
percentage of completion method of revenue accounting. On June 30, 2021, how
much income would JJD report on the project?
a. 250,000
b. 300,000
c. 750,000
d. 900,000
Solution:
June 30, 2020 June 30, 2021
Contract Price 22,500,000 22,500,000
Less: Total Estimated Costs 19,500,000 20,250,000
Estimated gross profit 3,000,000 2,250,000
Multiplied by: percentage of completion 9750/19500 15750/20250
Less: recognized gross profit in prior year - 1,500,000
Recognized gross profit each year 1,500,000 250,000

14. During 2020, Mitch Corporation started a construction job with a total
contract price of 600,000. Any costs incurred are expected to be recoverable. The
job was completed on December 15, 2021. Additional data are as follows:

2011 2012
Actual costs incurred 225,000 255,000
Estimated remaining costs 225,000 -
Billed to customer 240,000 360,000
Received from customer 225,000 375,000

Under the cost recovery method of construction accounting (zero-profit


approach), what amount should Mitch recognize as gross profit for 2020 and
2021?

2011 2012
a. 0 0
b. 75,000 120,000
c. 0 120,000
d. 120,000 120,000

Solution:
2020 2021
Recognized revenue 225,000 375,000*
Less: Costs of long-term construction contract 225,000 255,000
Recognized gross profit in current year 0 120,000

*Contract price 600,000


Less: Recognize revenue in 2020 225,000
Recognized Revenue in 2021 375,000

15. The following data relate to a construction job started by Jay Company during 2020:

Total contract price 100,000


Actual costs during 2020 20,000
Estimated remaining costs 40,000
Billed to customer during 2020 30,000
Received from customer during 2020 20,000
Any costs incurred are expected to be recoverable. Under the cost recovery
method construction method construction accounting (Zero-profit approach), what
amount should Jay Company recognize as gross profit for 2020:

a. 0
b. 4,000
c. 10,000
d. 12,000

Solution:

Recognized revenue in 2020 20,000*


Less: Cost of long-term construction (cost incurred in 2011)

20,000 Recognized gross profit in current year – 2020

0
*Revenue should be recognized only up to the extent of the contract costs
incurred that it is probable will be recoverable.

Use the following information for questions 16-19

Pepper Construction is constructing an office building under contract for Salt


Company. The contract calls for progress billings and payments of P1,240,000
each quarter. The total Contract price is 14,880,000 and Pepper estimates total
cost of P14,200,000. Pepper estimates that the building will take 3 years to
complete, and commences construction on January 2,20x8.

16. At December 31, 20x8, Pepper estimates that it is 30% complete with the
construction, based on cost incurred. What is the total amount of Revenue from
Long- term Contracts recognized for 20x8 and what is the balance in the accounts
receivable account assuming Cannon Company has not yet made its last
quarterly payment?
Revenue Accounts Receivable Revenue Accounts Receivable
a. 4,960,000 4,960,000 c. 4,464,000 1,240,000
b. 4,260,000 1,240,000 d. 4,260,000 4,960,000

Solution:
14,880,000 x .30 = 4,464,000

17. At December 31, 20x8, Pepper Construction estimates that it is 75%


complete with the building; however, the estimate of total cost to be incurred has
risen to P14,400,000 due to unanticipated price increases. At December 31, 20x7,
Pepper estimated it was 30% complete. What is the total amount of Construction
Expenses that Pepper will recognize for the year ended December 31, 20x8?
a. 10,800,000
b. 6,540,000
c. 6,390,000
d. 6,300,000
Solution:
(14,400,000 x .75) – (14,200,000 x .30) = 6,540,000

18. At December 31, 20x8, Pepper Construction estimates that it is 75%


complete with the building; However, the estimate of total cost to be incurred has
risen to P14,400,000 due to unanticipated price increases. What is reported
in the balance sheet at December 31, 20x8 for Pepper as the difference between
the Construction in Process and the Billings on Construction in Process accounts,
and is it a debit or a credit?

Difference between the accounts Debit/Credit


a. 3,380,000 Credit
b. 1,240,000 Debit
c. 880,000 Debit
d. 1,240,000 Credit

Solution:
(14,880,000 x .75) – (1,240,000 x 8) = P1,240,000 Debit

19. Pepper Construction completes the remaining 25% of the building


construction on December 31, 20x9, as scheduled. At the time the total cost of
construction are P15,000,000. At December 31, 20x8, the estimates were 75%
complete and total cost of P14,400,000. What is the total amount of Revenue from
Long-Term Contracts and Construction Expenses that Pepper will recognize for
the year ended December 31, 20x6?

Revenue Expenses Revenue Expenses


a. P14,880,000 P15,000,000 c. P3,720,000 P4,200,000
b. P3,720,000 P3,750,000 d. P3,750,000 P3,750,000

Solution:
14,880,000 x .25 = 3,720,000
15,000,000 – (14,400,000 x .75) = 4,200000

20. Bentley Construction Company has consistently used the percentage-of-


completion method. On January 10, 20x5, Ube began work on a P6,000,000
construction contract. At the inception date, the estimated cost of construction
was P4,500,000. The Following data relate to the progress of the contract:

Income recognized at 12/31/20x5........................................P 600,000


Cost Incurred 1/10/20x5 through 12/31/20x6.....................P3,600,000
Estimated cost to complete at 12/31/20x6..........................P1,200,000

How much income should Bentley recognize for the year ended Dec. 31 20x6?
a. P300,000
b. 525,000
c. 600,000
d. 900,000

Solution:

Contract Price P6,000,000


Less: Total Estimated cost
Cost incurred to 1/10/20x5-12/31/20x6 3,600,000
Add: Estimated cost to complete 1,200,000 4,800,000
Estimated Gross profit 1,200,000
Multiplied by: Percentage of completion 36/48
Recognized gross profit to date 900,000
Less: Recognized gross profit in prior year 600,000
Recognized gross profit in current year 20x6 300,000

21. The Maligaya Construction Corporation uses the percentage-of-completion


method of recognizing income from long-term construction contracts. In 20x7.
Maligaya entered into a fixed-price contract to construct a bridge for P30,000,000.
Estimated cost to complete the construction and contract cost incurred up to 20x4
were as follows:

Cumulative cost incurred Estimated cost to complete


As of December 31, 20x7 2,000,000 16,000,000
As of December 31, 20x8 11,000,000 11,000,000
As of December 31, 20x9 20,000,000 4,000,000

What is the Percentage of completion during the year


20x9? a. 25.00%
b. 33.33%
c. 50.00%
d. 83.33%

Solution:
20x8 20x9
Cost incurred to date P11,000,000 P20,000,000
Add: Estimated cost to complete 11,000,000 4,000,000
Total estimated Cost 22,000,000 24,000,000
Percentage of completion (over time)
Cost incurred to date/Total estimated cost 50% 83 1/2%
Percentage of Completion as of 20x6 83 1/3%
Less: Percentage of completion as of 12/31/20x8 50%
Percentage of completion 20x9 33 1/3%
Use the following information for questions 22-23.

In 20x8, Fred Corporation began construction work under a three-year contract.


The contract price is P4,800,000. Fred uses the percentage-of completion (over
time) method for financial accounting purposes. The income to be recognized
each year is based on the proportion of cost incurred to total estimated cost for
completing the contract. The financial statement presentations relating to this
contract at Dec. 31, 20x8, follow:

Statement of financial Position


Accounts Receivable-Construction contract billings P200,000
Construction in progress P600,000
Less contract billings 480,000
Cost and recognized profit in excess of billings 120,000

Income Statement
Income (before tax) on the contract recognized in 20x5 P120,000

22. How much cash was collected in 20x8 on this contract?


a. 40,000
b. 200,000
c. 280,000
d. 480,000

Solution: (c) P480,000 – P200,000 = P280,000

23. What was the initial estimated total income before tax in this contract?
a. 600,000
b. 640,000
c. 800,000
d. 960,000

Solution (d) 600,000 – 120,000= 480,000


480,000Total estimated cost x 4,800,000-Total estimated cost=120,000

Total estimated cost= 3,840,000; 4,800,000 – 3,840,000= P960,000

24. In 20x6, TVD Construction Co. was contracted to build Selena Company’s
private road network for P100 million. The project was estimated to be completed
in two years, and the contract provided for:
1. 5% mobilization fee (to be deducted from the last billing)
payable within 15 days after the signing of the contract
2. 10% retention provision on all billings, and
3. Payment of progress billings within 10 days from acceptance.

TVD, which uses the percentage-of-completion method of accounting,


estimated a 25% gross margin on the project. By the end of 20x6, TVD had
presented progress billings corresponding to 50% completion. All of the
progress billings presented in 20x6 were accepted, except the last one for
10% which was accepted on January 5, 20x7. With the exception of the bill for
8% which was due on January 7, 20x7, all of the billings accepted in 20x6
were settled. Payments made by Village Company in 20x6 amounted to:

a. P33,800,000
b. 38,500,000
c. 40,000,000
d. 45,000,000

Solution:
Mobilization Fee: 100M x 5% 5,000,000
Collections on Progress Billings:
Contract Price 100,000,000
Multiplied by Progress Billings
(net of late billings of 10% and 8%) 32%
Billings, Net 32,000,000
Multiplied by: Collection % (Net of 10%
contract retention) 90% 28,800,000
Collection/ Payments 33,800,000

25. On September 30, 20x6. Niall Co. Inc. was awarded the contract to build a
1,000- room hotel for P120 million. Among others, the parties agreed the
following:
1. 10% mobilization fee (deductible from “final billing”)
payable within 10 days from the signing of the
contract:
2. Retention of 10% on all billings (to be paid with the final
billing, upon completion and acceptance of the
project); and
3. Progress billings are to be paid within 2 weeks upon acceptance.

By the end of 20x6, the company had presented one progress billing,
corresponding to 10% completion, which was evaluated and accepted by the
client on December 29, 20x6 for payment in January (over time) of the next
year. In 20x6, assuming use of the percentage-of-completion method of
accounting, Niall Co., Inc. received cash a total fee of:

a. P1,200,000
b. 11,880,000
c. 12,000,000
d. 13,200,000
Solution:
Contract Price 120,000,000
Multiplied by Mobilization Fee 10%
Cash Received 12,000,000
JOINT ARRANGEMENTS

THEORIES

1. What is the classification of the joint arrangement when the arrangement is


structured without a separate vehicle such as when the rights of each party
to the total assets and obligations for total liabilities relating to the
arrangement are clearly established?
a. It shall be classified as joint venture.
b. It shall be classified as joint operation.
c. Neither joint venture nor joint operation.
d. It can be either a joint operation or a joint venture depending
on the company policy of the parties to the Joint arrangement.
Joint arrangement that is not structured through a separate vehicle is a joint

operation. In such cases, the contractual arrangement establishes the

parties' rights to the assets, and obligations for the liabilities, relating to

the arrangement, and the parties' rights to the corresponding revenues and

obligations for the corresponding expenses. [IFRS 11:B16]

2. It is a type of joint arrangement whereby the parties that have joint control
of the arrangement have the right to the total assets and obligations for the
total liabilities relating to the arrangement.
a. Joint venture
b. Jointly controlled asset
c. Joint operation
d. Joint business
Joint operation is “a joint arrangement whereby the parties that have joint

control of the arrangement have rights to the assets and obligations for the

liabilities, relating to the joint arrangement. Those parties are called joint

operators.” (PFRS 11.15) – Conceptual Framework and Accounting

Standards (Millan, 2018, p.545)


3. Under IFRS 11, how shall the joint venture account for its Investment in
Joint Venture?
a. Equity method
b. Cost method
c. Fair value method under IFRS 9
d. Proportionate consolidation
A joint venturer recognizes its interest in a joint venture as an

investment and shall account for that investment using the equity method

in accordance with IAS

28 Investments in Associates and Joint Ventures unless the entity is exempted

from applying the equity method as specified in that standard. [IFRS 11:24]

4. It is the contractually agreed sharing of control of an arrangement which


exists only when decisions about relevant activities require unanimous
consent of the parties sharing control.
a. Control
b. Significant influence
c. Joint control
d. Solidary Control
Joint control is the “the contractually agreed sharing of control of an

arrangement, which exists only when decisions about the relevant

activities require the unanimous consent” of the parties sharing control

(PFRS 11.7). – Conceptual Framework and Accounting Standards (Millan,

2018, p.544)

5. A jointly controlled entity is:


(a) an entity over which the investor has significant influence.

(b) an entity over which the investor has joint control.

(c) an entity over which the investor has significant influence or joint

control and that is not a subsidiary.


(d) an entity over which the investor has significant influence and that is

neither a subsidiary nor an interest in a joint venture.

A jointly controlled entity is a joint venture that involves the establishment

of a corporation, partnership or other entity in which each venturer has an

interest. The entity operates in the same way as other entities, except that a

contractual arrangement between the venturers establishes joint control

over the economic activity of the entity.

6. Joint control is:


(a) the power to participate in the financial and operating policy decisions

of the investee but is not control or joint control over those policies.

(b) active participation in the financial and operating policy decisions

of the investee but is not control or joint control over those policies.

(c) the power to govern the financial and operating policies of an entity so

as to obtain benefits from its activities.

(d) the contractually agreed sharing of control over an economic activity.

Joint control is the contractually agreed sharing of control over an

economic activity, and exists only when the strategic financial and

operating decisions relating to the activity require the unanimous consent

of the parties sharing control (the venturers).


7. An entity must account for its investments in jointly controlled entities after
initial recognition using:
(a) either the cost model or the fair value model (using the same

accounting policy for all investments in jointly controlled entities).

(b) either the cost model or the fair value model (model can be elected on

an investment-by-investment basis).

(c) either the cost model, the equity method or the fair value model

(using the same accounting policy for all investments in jointly

controlled entities).

(d) either the cost model, the equity method or the fair value model (model

can be elected on an investment-by-investment basis).

A venturer shall account for all of its interests in jointly controlled entities

using one of the following:

 the cost model in paragraph


 the equity method in paragraph
 the fair value model in paragraph
8. Investments in jointly controlled entities must be tested for impairment in
accordance with Section 27 Impairment of Assets, if the entity uses:
(a) the cost model, equity method or fair value model.

(b) the cost model or the equity method.

(c) the cost model or the fair value model.

(d) the equity method or the fair value model.


A venturer shall measure its investments in jointly controlled entities, other

than those for which there is a published price quotation (see paragraph

15.12) at cost less any accumulated impairment losses recognized in

accordance with Section

27 Impairment of Assets. A venturer shall measure its investments in jointly

controlled entities by the equity method using the procedures in paragraph

14.8 (substituting ‘joint control’ where that paragraph refers to ‘significant

influence’).

9. Which of the following statements is correct?


I. Joint Arrangements classified as joint ventures are accounted for
under PAS 28.
II. All Joint Arrangements are accounted for under PAS 28.
III. Joint Arrangements classified as joint ventures are accounted for under PFRS

(a) Only one statement is correct.

(b) All of the statements are correct.

(c) Statement II and III are correct.

(d) All of the statements are wrong.

Statement I and III are wrong because according to PAS 28 applies to all

entities that are investors with joint control of, or significant influence over,

an investee (associate or joint venture) [IAS 28(2011).2] and PFRS 11

applies to all entities that are a party to a joint arrangement.

10. Loey Company and Hyun Company agreed to form a joint operation to
offer health services. To start the operation the joint operators agreed to
contribute cash of P300,000 each. The joint operation will record which of
the following entries to recognize this event?
a. Joint operator contributions 600,000
Cash 600,000
b. Cash 600,000
Joint operator contributions 600,000

c. Cash 600,000
Joint operation contribution – LOEY 300,000

Joint operation contribution – HYUN 300,000

d. Venturer’s equity – LOEY 300,000


Venturer’s equity – HYUN 300,000

Cash 600,000

The contributed cash amounting to P600,000 should be debited to cash

account of the joint operation as it increases its assets and the contribution

of each party should credit to the joint operation contribution and it should

be proportionate to the amount, they contribute each.

11. The matters generally dealt with in a joint arrangement contract includes
I. voting rights of venturers
II. sharing of the output, expenses or results
III. capital contributions of the venturers
IV. activity, duration and purpose

(a) Statement I and II are included

(b) Only statement IV is not included

(c) Two statements are included

(d) All of the statements are included

The contractual arrangement sets out the terms upon which the parties

participate in the activity that is the subject of the arrangement. The

contractual arrangement generally deals with such matters as:


(a) the purpose, activity and duration of the joint arrangement.

(b) how the members of the board of directors, or equivalent governing

body, of the joint arrangement, are appointed.

(c) the decision-making process: the matters requiring decisions from the

parties, the voting rights of the parties and the required level of support for

those matters. The decision-making process reflected in the contractual

arrangement establishes joint control of the arrangement.

(d) the capital or other contributions required of the parties.

(e) how the parties share assets, liabilities, revenues, expenses or profit or

loss relating to the joint arrangement. [IFRS 11: Appendix B4]

12.A A separately identifiable financial structure, including separate legal


entities or entities recognized by statute, regardless of whether those
entities have a legal personality.
(a) Goodwill

(b) Separate Vehicle

(c) Joint venturer

(d) Contractual arrangement

A separate vehicle is a separately identifiable financial structure, including

separate legal entities or entities recognized by statute, regardless of

whether those entities have a legal personality. [IFRS 11: Appendix A]


13. Which of the following statements are true?
I: A joint venture is a joint arrangement whereby the parties have joint
control or rights to the net assets of the arrangement.
II: Joint venturers have rights over the individual assets or obligations
towards the individual liabilities of the joint venture.
III: In a two-party joint venture, it is not necessary that the joint
venturers have equal equity stake in the joint venture for a joint control
to exist.

(a) I & II

(b) I & III

(c) II & III

(d) I, II & III

Statement I & III are true since a joint venture is an arrangement where the

parties that have joint control of the arrangement have rights to the Net

Assets of the arrangement and it does not necessary that venturers must

have equal equity stake in a two-party joint venture. Statement II is false

since this pertains to a joint operation which is where the parties that have

joint control of the arrangement have rights to Assets and Obligations for

the Liabilities, in the arrangement. See page 761 of Advanced Financial

Accounting and Reporting (AFAR- 2021 edition) of Antonio J. Dayag for

further details about a joint venture.

14. Control exists in a joint arrangement when _ exposed or have


rights to variable returns.

(a) One investor in the arrangement is


(b) One or more investors in the arrangement are

(c) Two or more investors in the arrangement are

(d) Three investors in the arrangement are

Control in a joint arrangement exists when two or more investors are

exposed or have rights to variable returns. This means that in a joint

arrangement, unanimous consent of the parties is required before the

decisions about the relevant activities of the arrangement are made. It

cannot be done by a single entity only. See page 758 of Advanced Financial

Accounting and Reporting (AFAR- 2021 edition) of Antonio J. Dayag for

further details about the steps in assessment of the existence of joint

control.

15. The objective of PFRS 11 is to establish:

(a) Principles and guidelines for financial reporting.

(b) Principles and rules for financial reporting by entities that have an

interest in joint operations.

(c) Principles and guidelines for financial reporting by entities that have an

interest in joint arrangements.

(d) Principles for financial reporting by entities that have an interest

in joint arrangements.

PFRS 11 which is entitled Joint Arrangements prescribes the principles for


financial reporting by parties to a joint arrangement. This means that all
parties
that have joint control or is under any type of joint arrangement shall apply PFRS
11. See page 756 of Advanced Financial Accounting and Reporting (AFAR-
2021 edition) of Antonio J. Dayag for further details about the objective of
joint arrangements.

16. When a joint arrangement is structured through separate vehicle, it can be


classified as either joint operation or joint venture. If there is a separate
vehicle, the following factors are needed to be considered:
I: Legal form of the separate
vehicle II: Contractual terms
and conditions
III: Economic substance of the separate vehicle

(a) I & II

(b) I & III

(c) II & III

(d) I, II & III

Separate vehicle means a separately identifiable financial structure, which


includes separate entities or entities recognized by statute regardless of
whether those entities have a legal personality (PFRS 11- Appendix A). So,
if a joint arrangement is structured through a separate vehicle, the legal
form, contractual terms and conditions and other facts and circumstances
should be taken into consideration. See page 763 of Advanced Financial
Accounting and Reporting (AFAR- 2021 edition) of Antonio J. Dayag for
further details about arrangements structured and not structured in a
separate vehicle.

17. In a Joint Operation, the revenues and expenses are established by the
contractual agreement wherein:
(a) The allocation of revenues and expenses is on the basis of the

relative performance of each party to the joint arrangement.

(b) Each party’s share in the revenues and expenses is related to the

activities of the arrangement.

(c) Each party’s share in the revenues and expenses is on the basis

of the performance of each party.

(d) The allocation of revenues, expenses, profit or loss is related to the

activities of the arrangement.

In a joint operation, revenues and expenses are allocated on the basis of


the relative performance of each party to the joint arrangement. For
example, the parties might establish that revenues and expenses are
allocate on the basis of the capacity that each party uses in a plant
operated jointly. In a joint venture, the contractual arrangement establishes
each party’s share in the profit or loss in relation to the activities of the
arrangement. See page 764-765 of Advanced Financial Accounting and
Reporting (AFAR- 2021 edition) of Antonio J. Dayag for further details about
the difference of joint operations and joint ventures.

18. Under PFRS 11, as an exception to the general rule of mandatory equity
method accounting for Investment in Joint Venture, what is alternative
treatment available to joint venturer for an investment in joint venture held
or is held indirectly through an entity that is a venture capital organization,
mutual trust fund, unit trust and similar entities including insurance-linked
fund?
(a) It may elect to measure the investment in joint venture at fair
value through profit or loss
(b) It may elect to measure the investment in joint venture at fair value
through other comprehensive income
(c) It may elect to measure the investment in joint venture at cost method
(d) It may elect to measure the investment in joint venture at
proportionate consolidation
In general, all associates and joint ventures should be accounted for using

the equity method. However, for certain types of entities, such as venture

capital organization mutual trust funds, unit trusts and similar entities

including insurance-linked funds, they may elect to measure their

investments in associates and joint ventures at fair value through profit or

loss in accordance with PFRS 9. When availed, such entities need not apply

the equity method of the Standards but they must disclose their

involvements in accordance with PFRS 12.

19. Under PFRS 11, how shall the joint operator account for its interest in a
joint operation?
(a) The joint operator shall account for its its interest under Equity Method
(b) The joint operator shall account for its its interest under Cost Method
(c) The joint operator shall account for its its interest using
proportionate consolidation
(d) The joint operator shall account for its its interest by recognizing
its assets, its liabilities, its revenue, its expenses and its share in the
jointly controlled assets, jointly incurred liabilities, jointly earned
revenue, and jointly incurred expenses in accordance with the
contractual arrangement
The key feature of a joint operation is the joint operator has an interest in

the individual assets and liabilities of the joint operation. In the situation

where the joint operation produces an output distributed to the joint

operators, the joint operator will receive a share of the output of the joint

operation in the same way he be responsible for a share of the expenses of

the operation of the operation that are not capitalized into the cost of the

output.
20. How do joint ventures differ from private corporations?
(a) A joint venture does not have a board of directors
(b) There can only be two parties in a joint venture
(c) Venturers cannot make unilateral decisions
(d) The joint venturers must share the risks and profits of the joint venture equally
Usually, joint ventures are carried on as separate private corporation. The

difference is that, in private corporations, a single stockholder or a small

group may exercise control whereas in a joint venture, no investor or

subgroup of investors can control the joint venture. Major decisions require

the consent and agreement of all the co-venturers.

21. The existence of significant influence by an investor is usually evidenced in


which of the following?
(a) Representation on the board of directors or equivalent governing body
(b) Participation in policy-making process
(c) Interchange of management personnel
(d) All of the above
The existence of significant influence by an investor is usually evidenced in

one or more of the following ways:

A. Representation on the board of directors or equivalent governing body


B. Participation in policy-making process
C. Material transactions between investor and the investee
D. Interchange of management personnel
E. Provision of essential technical information
22. PFRS 11 Joint Arrangements, provides that joint control exists where:
(a) no single party is in a position to control the activity unilaterally
(b) the decisions in areas essential to the goals of the joint arrangement do not
require the consent of the parties;

(c) no one party may be appointed as the manager of the joint arrangement;
(d) one party alone has power to control the strategic operating decisions
of the joint arrangement.

PFRS 11 - JOINT ARRANGEMENTS - Joint Control

Joint control exists when all the parties sharing joint control over the

arrangement act collectively (or together) in control directing the activities

that significantly affect the returns of the arrangement.

An arrangement is considered a joint arrangement even if not all of the

parties to the arrangement have joint control. It is sufficient that at least two

of those parties share joint control.

23. In relation to the supply of a service to a joint operation by one of the joint

operators, which of the following statements is correct?

(a) A joint operator can recognize 100% of the earned through the
supply of services to the joint operation;
(b) A joint operator is entitled to recognize a profit from the supply of
services to itself;
(c) a joint operator cannot earn a profit on supplying services to itself;
(d) A joint operator is not able to recognize the service revenue or
service cost for the services supplied to the joint operation.

Accounting Treatment for a Joint Operation (by an Unincorporated Joint

Operation)
If the joint operation does not sell the output produced, but rather

distributes it to the operators, there is no profit or loss account raised by

the operation. In preparing accounts for the joint operation, the main

purpose is to accumulate costs as incurred. These are capitalized into a

work in progress account, which is transferred to the operators as

inventory. Further, the joint operation accounts provide information about

the assets and liabilities relating to the joint operation as well as the

contributions from the operators. Hence, a statement of financial position is

the joint operation's main financial statement.

24. For the purposes of equity accounting for an investment in an associate, it is

presumed that the investor has significant influence over the other entity where

the investor holds:

(a) between 1% and 5% of the voting power of the investee:


(b) between 5% and 10% of the voting power of the investee:
(c) 20% or more of the voting power of the investee
(d) 50% or more of the voting power of the investee

PAS 28 INVESTMENST IN ASSOCIATES AND JOINT VENTURES

Significant influence is presumed to exist if the investor holds, directly

or indirectly (e.g. through subsidiaries), 20% or more of the voting power of

the, investee. Conversely, significant influence is presumed not to exist if

the voting power is less than 20%


However, these are only presumptions, meaning they are generally

held to be true in the absence of evidence to the contrary. Thus, an

investor may have significant influence even if it has less than 20% voting

power, and conversely, may not have significant influence even if it has

more than 20% voting power, if these can be clearly demonstrated.

25. When eliminating any unrealized profit arising when a joint operator provides

services to a joint operation the profit is eliminated against:

(a) The investment in the joint operation;


(b) Retained earnings;
(c) Work-in-Progress, finished goods and other inventory related accounts;
(d) Cost of Goods sold.

Accounting Treatment for a Joint Operation (by an Unincorporated Joint

Operation)

If the joint operation does not sell the output produced, but rather

distributes it to the operators, there is no profit or loss account raised by

the operation. In preparing accounts for the joint operation, the main

purpose is to accumulate costs as incurred. These are capitalized into a

work in progress account, which is transferred to the operators as

inventory. Further, the joint operation accounts provide information about

the assets and liabilities relating to the joint operation as well as the

contributions from the operators. Hence, a statement of financial position is

the joint operation's main financial statement.


PROBLEMS

Use the following information to answer items nos. 1 and 2.

On January 1, 2018, Yeppeu, a public entity, and Da, a public entity, incorporated

Joo which has its fiscal and operational autonomy. The contractual agreement of

the incorporating entities provided that the decisions on relevant activities of Joo

will require the unanimous consent of both entities. Yeppeu and Da will have

rights to the net assets of Joo.

Yeppeu and Da invested P1,000,000 and P1,500,000 respectively, equivalent to

40:60 capital interest of Joo. The financial statement of Joo provided the following

data for its two-year operation:

Net Income (Loss) Dividends declared


2018 200,000 100,000
2019 (2,000,000)

1. What is the balance of Investment in Joo to be reported by Yeppeu in


its Statement of Financial Position on December 31, 2019?
a. P1,080,000
b. P1,040,000
c. P240,000
d. P200,000
2. What is the balance of Investment in Joo to be reported by Da in its
Statement of Financial Position on December 31, 2019?
a. P1,500,000
b. P1,620,000
c. P360,000
d. P900,000
Solution for Nos. 1 and 2

2018
Yepppeu (40%) Da (60%)
Beginning Investment 1,000,000 1,500,000
Net Income 200,000 80,000 120,000
Dividend 100,000 (40,000) (60,000)
Ending Investment 1,040,000 1,560,000
2019
Beginning Investment 1,040,000 1,560,000
Net Loss (2,000,000) (800,000) (1,200,000)
Ending Investment 240,000 360,000

Use the following information to answer items nos. 3 and 4.

On January 1, 2020, Aegyo Incorporated invested P2M cash in a joint venture for

50% interest. For the years ended December 31, 2020, 2021 and 2022, the joint

venture reported the following net incomes and dividend distributions:

Year Net Income (Loss) Dividend Distribution


2020 P1,000,000 P300,000
2021 (P6,000,000) -
2022 P7,000,000 P500,000

3. What is the share in net loss or investment loss to be reported by Aegyo


Incorporated for the year ended December 31, 2021?
a. P3,000,000
b. P2,500,000
c. P2,350,000
d. P2000,000
4. What is the book value of investment in Joint Venture to be reported by
Aegyo Incorporated as of December 31, 2022??
a. P1,600,000
b. P2,600,000
c. P1,250,000
d. P1,450,000
Solution for Nos. 3 and 4

Aegyo Incorporated
50%
2020
Beginning Investment 2,000,000
Net Income 1,000,000 500,000
Dividends 300,000 (150,000)
Ending Investment 2,350,000
2021
Beginning Investment 2,350,000
Net Loss (6,000,000) (3,000,000)
Unabsorbed Net Loss (650,000)
Ending Balance -
2022
Beginning Investment -
Net Income 7,000,000 3,500,000
Dividends 500,000 (250,000)
Unabsorbed Net Loss (650,000)
Ending Investment 2,600,000

** Aegyo Incorpoated’s share in net loss is equal to that of his balance in

investment in the joint venture.

For items number 5 and 6:

Banks Yoon and Han (the parties) agreed to combine their corporate, investment

banking, asset management and services activities by establishing a separate

vehicle (bank X). Both parties expect the arrangement to benefit them in different

ways.

The assets and liabilities held in Bank X are the assets and liabilities of the Bank

X and not the assets and liabilities of the parties. Banks Yoon and Han

each have a 40
percent ownership interest in Bank X, with the remaining 20 percent being listed

and widely held. The stockholder’s agreement between Bank Yoon and Bank

Han establishes joint control of the activities of bank X.

Transactions for the year 2019 and 2020 follows:

2019 2020
Investments: Bank Yoon P 50M P 25M
Bank Han 50M 5M
Revenues 10M 12M
Cost and Expenses 6M 7M
Dividends paid - Bank X - 4M
5. What is the interest of Bank Yoon in the joint arrangement at December 31,
2019? a. P 50,000,000
b. P 55,000,000
c. P 52,000,000
d. P 56,000,000
6. What is the interest of Bank Han in the joint arrangement at December 31, 2020?
a. P 52,900,000
b. P 60,000,000
c. P 52,800,000
d. P 54,500,000

Solution for Nos. 5 and 6

Investment- Bank Yoon P50,000,000


Profit Share (P10,000,000-6,000,000) x 50 % 2,000,000
Interest-Bank Yoon, December 31, 2019 P52,000,000

Investment-2019 P50,000,000
Profit Share-2019 2,000,000
Profit Share-2020 (P12,000,000-7,000,000) x 50% 2,500,000
Dividends Received (P4,000,000 x 40%) (1,600,000)
Interest- Bank Han, December 31,2020 P52,900,000
For items number 7 - 9

On January 1, 2020, Hosh, Seok and Han (the joint operators) jointly buy a

helicopter for P30million cash. The Joint arrangement includes the following

agreements:

a) The parties are the joint owners of the helicopter.


b) The helicopter is at the disposal of each party for 70 days each year.
c) The parties may decide to use the helicopter or lease it to a third party.
d) The maintenance and disposal of the helicopter require the unanimous
consent of the parties.
e) The contractual arrangement is for the expected life (20 years) of the
helicopter and can be change only if all the parties agree. The residual
value of the helicopter is P2million.
f) Revenues and expenses are to be shared equally among the joint
operators. In 2020 the parties paid P300,000 to meet the costs of
maintaining the helicopter.

In 2020 each party also incurred costs of running the helicopter when they

made use of the helicopter (e.g., Hosh incurred costs of P200,000 on pilot

fees, aviation fuel and landing costs). In 2020, the parties earned rental

income of P2.5 million by renting the helicopter to others.

5. What is the net income (loss) of the joint operation on December 31,
2020? a. P 1,000,000
b. P 500,000
c. P 2,000,000
d. P 1,500,000

6. What is the book value of the helicopter in the books of Hosh on


December 31, 2020?
a. P 30,000,000
b. P 28,500,000
c. P 26,500,000
d. P28,000,000
7. What is the share of Seok in the net income (loss) of the joint
operation on December 31?
a. P 500,000
b. P 333,333
c. P 375,000
d. P 166,667

Solution for Nos. 7 and 9

Rental income 2,500,000


Operating expense:
Maintenance costs 300,000
Pilot fees, aviation fuel,
200,000 (500,000)
landing costs
Depreciation expense
(P 30M / 20 years) (1,500,000)
Net income 500,000

Cost 30,000,000
Accumulated
(1,500,000)
depreciation
Book value 28,500,000

Net income 500,000


x Percentage of allocation 1/3
Share of Seok in Net income 166,667
8. Jaxon and Radleigh agreed on a joint operation to purchase and sell car
accessories. They agreed to contribute P25,000 each to be used in
purchasing the merchandise, share equally the gain or loss, and record
their joint operations transactions in their individual books. After one year,
they decided to terminate the arrangement and data from their records
were:

Joint operations account credit balances: in books of Jaxon, P18,000; in


books of Radleigh, P20,000, Cost of car accessories taken: by Jaxon,
P1,000; by Radleigh, P1,800, Expenses paid: by Jaxon, P1,850; by
Radleigh, P2,600. How much was the joint operations sales?

a. P 83,750
b. P 91,000
c. P 92,650

Investment in Joint Operations


Purchases P 25,000 P92,650 Sales
25,000
Expenses 1,850
2,600
P54,450 P92,650
Credit balances: P18,000 – Jaxon
38,200
20,000 – Radleigh
Unsold Merch.: P 1,000 – Jaxon
2,800
P 1,800 – Radleigh
P41,000 PROFIT
d. P 86,550

9. The following joint operations account reflects the transactions of the


arrangements of HISOKA, SUKUNA, and MEREUM as recorded by each
operator in year ended 20x9.
a. Nov. 05 – Merchandise costing P12,750 by MEREUM
b. Nov. 17 – Merchandise costing P10,500 by SUKUNA
c. Nov. 18 – Cash sales made by HISOKA amounted to P30,600
d. Nov. 22 – Freight in incurred by HISOKA amounted to P525
e. Dec. 03 – HISOKA purchased merchandised amounted to P5,250
f. Dec. 12 – Cash sales made by HISOKA amounted to P6,300
g. Dec. 13 – Selling expenses amounting to P600 were paid by HISOKA
h. Dec. 28 – Unsold inventories of SUKUNA at year end amounted to P1,815
The joint operation profit (loss) is:

a. P 9,090
b. P 7,275
c. P 25,980
d. P 29,625

Investment in Joint Operations


20x9
11/5 Merchandise – MEREUM P12,750 P30,600 11/18 Cash sales - HISOKA
11/17 Merchandise – SUKUNA 10,500 6,300 12/12 Cash sales - HISOKA
11/22 Freight in - HISOKA 525 1,815 12/28 Unsold Merchandise - SUKUNA
12/03 Purchases - HISOKA 5,250
12/13 Selling expense -
600
HISOKA
P38,
P29, 626
715
P9,090 PROFIT

For items 12 and 13

BOKUTO and AKAASHI enter into a contract to speculate on the stock market,

each using approximately their personal cash. The earnings are to be divided

equally, and settlement is to be made at the end of the year after all securities

have been sold. A summary of the monthly brokerage statements for the year as

follows.

BOKUTO AKAASHI
Total of all purchase confirmations P45,000 P18,000
Total of all sales confirmations 48,700 16,800
Interest charged on margin
80
accounts 50
Dividends credited to accounts 40 100
12. The joint operation profit
(loss) is: a. P (3,370)
b. P 2,510
c. P 2, 640
d. None

13. Using the same information in No. 12, final settlement will require
payments as follows:
a. BOKUTO and AKAASHI receive P1, 255 each.
b. BOKUTO receives from AKAASHI P1, 150.
c. BOKUTO pays AKAASHI P2,405.
d. None

Solution for 12 and 13

Investment in Joint Operations


Purchases P 45,000 P48,700 Sales
18,000 16,800
Interest
80 40 Dividend
expense
50 100
P63,130 P65,640
P2,510 PROFIT
BOKUTO,
CAPITAL
Sales P 48,700 P45,000 Purchases
Dividend 40 80 Interest expense
Distribution of
1, 255 profit
( P 2510 x 50%)
P 2, 405
AKAASHI, CAPITAL
Sales P 16,800 P18,000 Purchases
Dividend 100 50 Interest expense
Distribution of
1, 255 profit
( P 2510 x 50%)
P 2, 405

14. Itadori and Gojo formed a joint arrangement to acquire and sell a special
type of merchandise. Gojo is the one who is assigned to manage the joint
arrangement and furnish the capital. The joint operations are to share
equally any gain or loss on the joint operations. On February 1, 2021,
Itadori sent Gojo P10,000 cash, which was all used to purchase
merchandise. Gojo paid freight of P240 on the merchandise purchased. On
February 27, one half of the merchandise was sold for P7,200 cash. Gojo
paid the cost of delivering merchandise to customers which amounted to
P260. No further transactions occurred until the end of the month. (Adapted
from Antonio J. Dayag’s reviewer - 2021 edition, page 657)

The profit(loss) of the joint arrangement for the month of February 2021 is:
a. P 1,820
b. P 1950
c. P (1,700)
d.

None

Solution:

Investment in Joint Operation


Purchases P10,000 P7,200 Sales
Freight-in 240 5,120 Unsold*
Freight-out 260
P10,500 P12,320
P1,820 Joint Operation’s Profit
*P10,000 + 240) x ½ = P5,120

15. Using the same information in No. 1, the account of Itadori in the books of
Gojo shows a debit (credit) balance on February 28, 2021 after recognizing
the profit (loss) on the uncompleted joint arrangement:
a. P (10,910)
b. P 10,975
c. P 10,850
d.

Zero

Solution:

Itadori, Capital
P10,000 Investment in
Joint
Operations
910 Profit*
P10,910 Balance

16. Itadori, Megumi and Gojo formed a joint arrangement in 2021 and agreed
to divide profits and losses equally. The arrangements is terminated on
December 31,2022 even though there are still unsold merchandise. On this
date, Megumi’s trial balance contains the following account balances before
profit or loss distribution: (Adapted from Antonio J. Dayag’s reviewer - 2021
edition, page 659)

Debit Credit

Cash P30,000

Joint operation 6,000

Itadori, capital 14,000

Gojo, capital P16,000


Megumi receives P4,500 for his share in the joint operations profit.
Furthermore, he agrees to be charged for the unsold merchandise as of
December 31, 2022. The cost of the unsold merchandise charged to
Megumi is:
a. P 3,000
b. P 15,000
c. P 13,500
d. P 19,500

Solution:

Investment in Joint Operation

JO profit before P6,000 P19,500 Unsold


profit or loss merchandise

P13,500 JO profit*

*P4,500 x 3 (Itadori, Megumi & Gojo) = P13,500

17. Gojo corporation purchased 25 percent of Megumi company’s stock in


January 1, 2021 for P600,000. At the acquisition date, Megumi has
equipment with a market value of P250,000 greater than its book value. On
that same date, Gojo corporation gives the ability to have joint control with
another entity over Megumi company. The equipment has an estimated
remaining life of 10 years. In 2021, Megumi has net income of P320,000
and pays P80,000 of dividends. (Adapted from Antonio J. Dayag’s reviewer
- 2021 edition, page 665)

What is the balance in the investment account on Gojo’s financial records


at the end of 2021?
a. P 600,000
b. P 660,000
c. P 653,750
d. P 673,750
Solution:

Acquisition cost (01/01/2021) P600,000

Add: Share in Net Income (P320,000 x 25%) 80,000

Deduct: Share in Dividends (P80,000 x 25%) (20,0000

Amortization of allocated excess (6,250)

(P250,000 x 25% / 10 years)

Investment balance (12/31/2021) P653,750

For items 18 and 19:

Because the scale of the project exceeded the capacity of entities RM and JK

individually, they tendered jointly for a public contract with a government to

construct a motorway between two cities. Following the tender process, the

government awarded the contract jointly to entities RM and JK.

In accordance with the contractual arrangement’s entities RM and JK are

jointly contracted with the government for delivery of the motorway in return

for 39,200,000 (a fixed price contract). In 20X5, in accordance with the

agreement between entities RM and JK:

 Entities RM and JK each used their own equipment and employees


in the construction activity
 Entity RM constructed three bridges needed to cross rivers on the
route at a cost of 11,200,000
 Entity JK constructed all of the other elements of the motorway at a
cost of 16,800,000
 Entities RM and JK shared equally in the 39,200,000 jointly invoiced
to (and from) the government
18. Determine the net income generated by Joint Operator -
Entity RM: a. 19,600,000 c.
8,400,000
b. 11,200,000 d. Nil
19. Determine the net income generated by Joint Operator -
Entity JK: a. 19,600,000 c.
2,800,000
b. 16,800,000 d. Nil
Solution for 18 and 19:

RM JK
Construction revenue 19,600,000 19,600,000
Less: Construction costs 11,200,000 16,800,000
Net Income 8,400,000 2,800,000
For items 20 and 21:

On 1 January 20X4 entities Big Hit, JYP, YG, SM, and Cube (the joint operators)

jointly buy a jet aircraft for 14,000,000 cash. The operators are the registered as

equal joint owners of the aircraft. They enter into an agreement whereby the

aircraft is at the disposal of each operator for 70 days each year. The aircraft is in

maintenance for the remaining days each year. The operators may decide to use

the aircraft, or, for example, lease it to a third party. Decisions regarding

maintenance and disposal of the aircraft require the unanimous consent of the

operators. The contractual arrangement is for the expected life (20 years) of the

aircraft and can be changed only if all of the operators agree. The residual value

of the aircraft is zero.

In 20X4 the operators each paid 140,000 to meet the joint costs of maintaining

the aircraft (e.g., hangar rental and aviation license fees).


In 20X4 each operator also incurred costs of running the aircraft when they

made use of the aircraft (e.g., entity Big Hit incurred costs of 70,000 on pilot fees,

aviation fuel and landing costs).

In 20X4 entity Big Hit also earned rental income of 532,000 by renting the aircraft

to others.

20. Determine the net income generated by Joint Operator - Entity Big Hit:

a. 182,000 c. 392,000

b. 322,000 d. None of the above

21. The net book value of property, plant and equipment Determine the net
income generated by Joint Operator - Entity Big Hit:

a. 2,660,000 c. 2,268,000

b. 2,590,000 d. None of the above

Solution for 20 and 21:

Rental Income 532,000


Less: Aircraft operating
210,000
expenses
Depreciation expense 140,000
Net Income 182,000

Property, plant and equipment 2,800,000


Less: Accumulated
140,000
depreciation
Net Book Value 2,660,000
Use the following information for Questions 22 to 23:

January 1, 20x5. Black Company and Pink Company signed an agreement to


form a joint operation of manufacture a product called plasma. This product is
used in the manufacturing of television. The following are transactions transpired
in relation to joint operations for 20x5:

a. To commence the operation, both operators contributed P252,000 in cash


b. Contributions of cash by the operators.
c. Use of cash and loan to buy machinery & equipment costing P134,400
(cash paid, P84,000 and the balance on a loan account) and raw materials
purchase on account costing P109,200
d. Labor incurrence amounting to P120,960 with P131,600 paid in cash.
e. Loans from the bank, P100,800.
f. Repayment of loan-machinery and equipment, P16,800, raw materials
amounting to P70,560 and other factory expenses, of P218,400.
g. Depreciation of machinery and equipment, P13,440.
h. Transfer of materials, labor and overhead to Work-in-Process: payroll,
P120,960; Materials, P80,640; Factory overhead- heat, light and power,
P218,400 and depreciation of P13,440.
i. Transfer of Work-in-Process to Finished Goods Inventory, P302,400
j. Transfer of Finished Goods Inventory, P268,800 to Joint Operators
throughout the year.

22. Determine the ending balance in cash:


d. P 0
e. P 80,640
f. P 151,200
g. None of the above

Solution:

Cash
Contribution - Black 252,000 84,000 Machinery and equipment
Contribution - Pink 252,000 117,600 Labor
Bank loan 84,000 16,800 Machinery and equipment
70,560 Accounts payable
218,400 Factory overhead control
Balance, 12/31/x5 80,640
23. Determine the work in process ending balance amounted to:

a. 117,600
b. 131,040
c. 433,440
d. None of the above
Soluti
on:

Work-in-Process
Labor 120,960 302,400 To Finished goods
Materials 80,640
Factory overhead - heat, 218,400
ect.
Factory overhead - 13,440
depreciation
Balance, 12/31/x5 131,040

24. The December 31, 20x5, total assets amounted to:

a. P263,760
b. P381, 360
c. P394,800
d. None of the

above Solution:

December 31, 20x5


Assets
Current Assets
Cash P 80,640
Finished Goods Inventory 33,600
Work-in-Process inventory 131,040
Materials inventory 28,560
Total current assets P 273,840
Non-current Assets
Equipment P 134,400
Less: Accumulated depreciation 13,440 120,960
Total Assets P 394,800
Liabilities and Net Assets
Current liabilities
Accrued payroll P 3,360
Accounts payable 38,640 42,000
Non-current liabilities
Bank loan payable P 84,000
Loan payable - machinery and equipment 33,600 117,600
Total liabilities P 159,600
Net Assets 235,200
Total liabilities and net assets P 394,800

25. The December 31, 20x5, Black's investment amounted to:

a. 117,600
b. 235200
c. 252000
d. None of the

above Solution:

Joint Operator's Equity


Black Company: Contribution - January 1, 20x5 P 252,000
Less: Cost of inventory distributed (134,400) P 117,600

Pink Company: Contribution - January 1, 20x5 P 252,000


Less: Cost of inventory distributed (134,400) P 117,600
Total Joint Operator's Equity P235, 200
CONSIGNMENT

THEORIES
1. The party which send goods is called:

a. Consignee
b. Acquiree
c. Principal
d. Acquirer

The party which sends the goods (consignor) is called principal.

2.. Which of the following statements is correct?

a. The party to whom goods are sent (consignee) is called principal.


b. Consignment account relates to accounts dealing with a situation
where one person (or firm) sends goods to another person (or firm) on
the basis that the goods will be sold on behalf of and at the risk of the
former.
c. Consignment means that the ownership of the goods transfers with the
transfer of goods from the seller to the buyer.
d. The relationship between consignor and consignee is that of a creditor and
a debtor.

A is incorrect because the party to whom goods are sent (consignee) is called agent.

C is incorrect because consignment means that the ownership of the goods rests with
the consignor till the time they are sold by the consignee, no matter the goods are
transferred to the consignee.

D is incorrect because the relationship between the consignor and the consignee is
that of a principal and agent.

Therefore, only B is correct.


3. Statement 1: The ownership of the goods, i.e., the property in the goods,

remains with the consignor or the principal-the agent does not become their

owner even though they are in his possession. On sale, of course, the buyer will

become the owner.

Statement 2: The consignee can return the unsold goods to the consignor.

a. All statements are correct


b. Only one statement is correct
c. All statements are incorrect
d. None

All the statements are correct and within the principle of consignment.

4. Which of the following statements is incorrect.

a. The shipment of goods to the consignee is not treated as a sale


b. The merchandise is carried throughout the consignment as the
inventory of the consignee.
c. In consignment, the dealer or distributor has not obtained control of the product.
d. In consignment sales, the consignor uses a modified version of the point-
of-sale basis of revenue recognition.

B is incorrect because the consignor is the one that carries the merchandise as

inventory and not the consignee.

5. Statement 1: The consignee (dealer), who act as an agent for the


consignor in selling merchandise, remits to the consignor cash received from
customers after deducting any chargeable expenses.
Statement 2: The report that the consignor periodically receives from the
consignee is called account sales. This report shows the merchandise received
merchandise sold, expenses chargeable to the consignor, and the cash remitted.
a. All statements are correct
b. Only one statement is correct
c. All statements are incorrect
d. None

Only statement 2 is correct.. Statement 1 is incorrect because the the consignee

(dealer), who act as an agent for the consignor in selling merchandise, remits to the

consignor cash received from customers after deducting sales commission and any

chargeable expenses.

6. The following are chargeable to the sold units except

a. Commissions
b. Advertising
c. Delivery and installation
d. Amortization expense

Items chargeable to the sold units:

● Commissions
● Delivery and installation
● Advertising
● Reconditioning on delivered units to customers
● Insurance in transit to customers
● Expenses related to returned units delivered

Only D (amortization expense) is not chargeable to sold units.


7. Which of the following is a correct journal entry to record the loss

incurred by a consignment:

a. Profit and loss on consignment Dr. & Consignment Cr.


b. Consignment Dr. & Profit and loss on consignment Cr.
c. Profit and loss on consignment Dr. & Consignee Cr.
d. Consignee Dr. & Profit and loss on consignment Cr.

When the consignor sends the goods, losses may arise during consignment
transactions. Such losses affect the consignor account and not the consignee
account. So the consignor makes some entries to adjust the loss. The balance of
consignment account represents a profit or a loss on consignment and is transferred
to “Profit and Loss on Consignment Account“.

8. The profit generated or loss incurred by a consignment is shown by the:

a. goods sent on consignment account


b. consignee account
c. consignment account
d. stock reserve account

Consignment account is prepared to ascertain the profit earned or loss incurred by


the consignor on a specific consignment. This account can be viewed as a combined
trading and profit and loss account prepared specifically for consignment business.

9. If the consignor is a manufacturer, which of the following journal entries

should he make to close the goods sent on consignment account?

a. Goods sent on consignment Dr. and Trading account Cr.


b. GTrading account Dr. and Goods sent on consignment Cr.
c. Goods sent on consignment Dr. and Purchases account Cr.
d. Purchases account Dr. and Goods sent on consignment Cr.
The goods sent on consignment account is closed at the end of the financial year by making one of th

(i). In case the consignor is a manufacturer of goods:


GoodssentonconsignmentA/C
xxx
Trading A/Cxxx

(ii). In case the consignor is a wholesaler of goods:


Goods sent on consignment A/C [Dr] xxx
Purchase A/C [Cr] xxx

10. Which of the following accounts will be credited by the consignee when he

sells the goods?

a. Sales account
b. Consignor account
c. Inventory account
d. Commission received account

In the books of consignee, sale of books is credited to the consignor's account. This

is because the goods belong to the consignor. So, the entry for sales is passed as if it

were sales made by the consignor, by crediting his account

11. Salena Co. paid the in-transit insurance premium for consignment goods

shipped to Zamantha Co. In addition, Salena advanced part of the commission

that will be due when Zamantha sells the goods. Should Salena include the in-

transit insurance premium and the advanced commissions in inventory costs?


Insurance Premium Advanced Commission

a. Yes Yes
b. No No
c. Yes No
d. No Yes

Inventoriable costs include all costs of making the inventory ready for sale.
Cost, incurred by a consignor on the transfer of goods to a consignee are
costs necessary to prepare the inventory for sale. Consequently, they are
inventoriable. Thus, the in-transit insurance premium paid is inventories; while
the advanced commissions constitute a receivable or prepaid expense and not
an element of inventory costs.

13. Which is the following is most true regarding consignment arrangements?

a. Revenue is recognized at the point in time when the consignment


arrangement is made
b. Revenue is recognized when the goods are transferred to the consignee
c. Revenue is recognized upon sale by consignee to an end customer
d. Revenue is sometimes recognized because GAAP does not
allow such arrangements
Revenue is recognized when the dealer or distributor sells the product to a customer,
or when the dealer or distributor obtains control of the product (i.e. after a specified
period of time expires)

Items 14 to 15 are based on the following information:

Marlou Ponte is an artist who sells his work under consignment (he displays his

work in local boutique shops, and customers purchase his work there). Ponte

recently transferred a painting on consignment to a local boutique shop.

14. Ponte most likely should recognize revenue when:


a. He paints the painting, because the painting is produced while he works
b. When he transfers the painting to a boutique shop
c. When the boutique sells the painting
d. When the barbershop’s right of return expires

Consignment arrangements normally do not qualify revenue recognition until the


delivery is made to an end-customer. Prior to the point, control of goods is viewed as
having been retained by the consignor, not by the consignee.

15. After Ponte has transferred a painting to a boutique, the painting:

a. Should be counted in Ponte’s inventory until the boutique sells it.


b. Should be counted in boutique’s inventory, since it holds the painting
c. Should be counted in either Ponte’s or boutique’s inventory, depending
who has more expenses
d. Cannot give an answer because of lack of information
Ownership of the goods rest with the consignor until the time they are sold by the
consignee. Therefore, goods should be counted in the consignor's inventory, which in
this case is Marlou Ponte.

16. I. Any consignment expenses paid by the consignor is added to cost

but not recognized as part of inventory

II. The consignee makes a journal entry for the receipt of the inventory

in the general ledger.

a. True; True c. True; False


b. False; True d. False; False

Any consignment expenses paid by the consignor are added to the inventory balance
as added costs. The consignee does not make an entry for receipt of the inventory in
the general ledger; however, memorandum control records usually are kept.

17. The following are rights of consignee, except:

a. Compensation
b. Reimbursement for expenses not related to consignment
c. Granting credit
d. Warrant of consigned goods.

Consignee has the right of reimbursements for advances and necessary expenses.
Unless otherwise provided for in the agreement, the consignor, is responsible for all
the costs incurred that are directly related to the sale of the goods (i.e. freight and
insurance)

18. I. Only the consignor is interested in selling

II. An indicator of a consignment arrangement is when the dealer does not

have an unconditional obligation to pay the entity for the product

a. True; True c. True; False


b. False; True d. False; False

Both consignor and consignee are interested in selling - the former to make a profit or

develop a market, the latter to make a commission on the sale.

Consignment arrangement can be observed when the entity maintains control over

the product, the entity can require return/transfer of goods and when then the dealer

has no unconditional obligation to pay the entity for the goods.


19. In accounting for sales on consignment, sales revenue and the related

cost of goods sold should be recognized by the

a. Consignor when the goods are shipped to the consignee.


b. Consignee when the goods are shipped to the third party.
c. Consignor when notification is received that the consignee has
sold the goods.
d. Consignee when cash is received from the customer.

When goods are received by the consignee from the consignor, there is no

sales.Sales revenue and the related cost of goods sold are only recognized by the

consignor when the notification is received that the consignee has sold the goods.

20. When goods are consigned out, profits should be recognized by the

consignor when the

a. Goods are sold by the consignee.


b. Goods are received by the consignee.
c. Consignee agrees to the terms of the consignment.
d. Goods are shipped by the consignor.

When goods are consigned or sent by the consignor to a consignee, the consignee is

only an agent receiving the goods. Thus, there is no sale. It is only when the goods

are sold by the consignee and the consignor gets notified that the latter can

recognize sales and profit.


21. Jel Co., a consignee, paid the freight costs for goods shipped from Dale Co.,

a consignor. These freight costs are to be deducted from Jel's payment to Dale

when the consignment goods are sold. Until Jel sells the goods, the freight costs

should be included in Jel's

a. Cost of goods sold.


b. Freight-out costs.
c. Selling expenses.
d. Accounts receivable.

Cost related to the consigned goods is an expense to the book of the consignor.

Should the consignee incurred cost to the consigned goods, it is a receivable from

the consignor.

22. A form used to show the type of merchandise, quantity received, quantity

sold, and balance on hand.

a. Stock Record
b. Stock Ledger
c. Purchase Order
d. Inventory Record
Stock ledger is a file of stock records for all merchandise on hand.

Purchase Order is a completed form authorizing a seller to deliver goods with


payment to be made later

Inventory Record a form used during a periodic inventory to record information about
each item of merchandise on hand.
23. Goods on consignment should be included in the inventory of .

a. the consignor but not the consignee.


b. both the consignor and the consignee.
c. the consignee but not the consignor.
d. neither the consignor nor the consignee.

Goods on consignment are owned by the consignor. The consignee is just an agent

to receive the goods and earns a commission based on their agreement. There is no

transfer of possession or rights, thus, the goods are part of the consignor’s inventory.

24. Revenue is recognized by the consignor when the _.

a. goods are shipped to the consignee.


b. consignee receives the goods.
c. consignor receives an advance from the consignee.
d. consignor receives an account sales from the consignee.

When goods are consigned or sent by the consignor to a consignee, the consignee is

only an agent receiving the goods. Thus, there is no sale. It is only when the goods

are sold by the consignee and the consignor gets notified that the latter can

recognize sales and profit.

25. In the books of consignee the expenses incurred by him on consignment are debited to:
a. consignment account
b. cash account
c. consignor’s account
d. no entry
Cost related to the consigned goods is an expense to the book of the consignor.

Should the consignee incurred cost to the consigned goods, it is a receivable from

the consignor.
PROBLEMS
1. On August 1, 2021, BBC. Inc. consigned to CNN store 20 flower vases costing

P4,000 each, paying a freight charge of P2,000. At the end of the month, CNN

store reported sales of 8 flower vases at P7,000 each and expenses incurred of

3,000, and remitted the net proceeds due to BBC. Inc. after deducting a 20%

commission. How much net income did the BBC. Inc. realize in August on the

consignment?

a. P8,000
b. P9,000
c. P11,200
d. P7,800
Consignment Soles: P7,000 x 8..........................................................P56,000
Less: Applicable costs and expenses related to consignment soles:
Consigner:
Cost of goods sold: P4,000 x 8...................P32,000
Freight: P2,000 x 8/20....................................P 800
Consignee:
Expenses..........................................................P 3,000
Commission ………………………………….P 11,200
( P47,000) Net Income..........................................................................P9,000

2. On June 30, 2021, Mr. Queen consigned 90% rice cookers to Start Up

Company for sale at P1,600 each and paid P1,200 in transportation costs. A

report of sales was received on July 31 from Start Up Company reporting the sale

of 15 rice cookers, together with a remittance of the P27,200 balance due. The

remittance was net of the agreed 15% commission. How much should Northup

recognize as consignment sales revenue in July?

a. P24,000
b. P27,200
c. P24,480
d. P20,000
A sale takes place when there is a transfer of ownership of goods. A consignment

does not transfer ownership of the goods to another person who is to sell the goods

but the owner retains title to such goods until the consignee makes a bona fide sale.

Since the sales of twenty (20) rice cookers were made in July by the consignee (Start

Up Company), therefore, the sales revenue equivalent to the number of rice cookers

(i.e.. 15 rice cookers x P1,600 = P24,000) by the consignee should be recognized

by the consignor.

Items 3 to 6 are based on the following information

On June 1, Bell Corporation consigned 10 cabinets to Bee Store. Manufacturing

cost is P4,000. Consignment profits are not recorded separately by the company.

At the end of June, the dealer reported the sale or 4 cabinets at P7,000 each and

remitted the net sales proceeds after deducting the following: 20% commission

on the cabinets sold and P1,600 freight paid upon receipt of the 10 cabinets.

3. The entry on the books of Bell Corporation to record the shipment

assuming consignment profits are calculated separately includes:

a. a debit to Consignment out of P70,000


b. a debit to Consignment in of P40,000
c. a credit to Merchandise Shipment on Consignment of P 40,000
d. a credit to Merchandise Inventory of P70,000
4. Cash remitted to the consignor was?

a. P20,800
b. P24,960
c. P24,000
d. P40,000

Sales (4 x P7,000) P28,000


Charges: Commission (20% x P28,000) P5,600
Freight 1,600 ( 7,200)
Remittance P20,800

5. The balance of the consignor's inventory relative to consigned goods is?

a. P20,800
b. P24,960
c. P24,000
d. P40,000

Cost (6 x P4,000) P24,000


Freight (6/10 x P1,600) 960
Balance of Merchandise on Consignment account P24,960

6. Net profit on consignment sales was

a. P12,000
b. P6,400
c. P5,760
d. P11,360
Sales (4 x P7,000) Cost (4 x P4,000) P 28,000
Less: Commission Freight (4/10 x P1,600) Net profit on consignment
16,000
P5,600
6406,240
P5,760

7. On June 1, 2016, Hyram Co. paid P15,000 for the insurance of consigned

goods, while in transit, shipped to a consignee, and P17,000 for the freight. In

addition, Hyram advanced P5,000 as part of the commission that will be due

when the consignee sells the goods. The consigned goods cost Hyram P70,000

and will be sold for a total amount of P150,000. What is the total amount of

inventory should Hyram report for the consigned goods on June 1, 2016?

a. 90,000
b. 102,000
c. 107,000
d. 97,000

Cost of goods on consignment P70,000


Add: Inventoriable costs

Insurance P15,000

Freight 17,000 32,000

Total cost of inventory on consignment P102,000

Explanation:
Inventoriable costs include all costs necessary to prepare goods for sale (e.g., the
purchase price or manufacturing costs, freight, and other costs necessary to get the
goods while in transit and freight are included in inventory costs. Commissions paid to
consignee are selling expenses in the period the consigned goods are sold which are
not required to prepare the goods for sale. The selling price of the goods was
ignored, because normally, inventory should be valued at cost.
8. The following items were included in Heart Co.’s inventory account on

December 31, 2020:

Merchandise out on consignment, at sales price, including 40%


markup on selling price P40,000
Goods purchased, in transit, shipped FOB shipping point 36,000
Goods held on consignment by Heart 27,000
By what amount should Heart’s inventory account on December 31, 2020 be reduced?

a. 103,000
b. 67,000
c. P43,000
d. 51,000

Mark-up on selling price of consigned goods (40,000 x 40%) Goods held onP16,000
consignment by Heart
Total reduction in inventory 27,000
P43,000

Notes:
Normally, inventories should be valued at cost; thus, the mark-up on cost must not be included. When g

Items 9 and 10 are based on the following information

Aiman Manufacturing Co. consigned to Aljohn Trading Corp. twelve (12) Sharp

TV sets which cost P9,000 each. Freight out was paid by the consignor in the

amount of P600. Aljohn sold eight (8) sets, rendered an account sales, and

remitted the amount of P82,600 after deducting the following from the selling

price of the sets sold:

Commission on selling price 12%


Selling expenses 1,200
Cost of antennae given free 1,400
Delivery and installation 2,800
9. The total selling price of the eight (8) sets sold by Aljohn Trading Corp. is:
a. 100,000
b. 88,000
c. 98,560
d. 78,571

Consignee’s remittance P82,600

Add: Consignee’s charges

Selling expenses P1,200

Cost of antennae given free 1,400

Delivery and installation 2,800 5,400

Selling price before 12% commission 88,000

Add commission (88,000/88% x 12%) 12,000

Selling price after 12% commission P100,000

Notes:

Account sales is a report submitted by the consignee usually upon his remittance to
the consignor, which shows the number of units received, goods sold and unsold,
expenses incurred by the consignee, advances made to the consignor, and the
amount owed or remitted. Therefore, the balance of the selling price after deducting
consignee’s charges is the amount owed or remitted by the consignee to the
consignor.

10. The net profit of Aiman Manufacturing Co. on the eight (8) sets sold by

Aljohn Trading Corp. is?

a. 4000
b. 9332
c. 10,200
d. 10,600
Sales P100,000

Less: Costs and expenses

Cost of sales (P9,000 x 8) P72,000

Freight out (P600 x 8/12) 400

Commission 12,000

Selling expenses 1,200

Cost of antennae given free 1,400


Delivery and installation
2,800 89,800
Consignment profit
P10,200

Notes:

The consignment profit that the consignor will earn is the excess of sales price over
the applicable cost and expenses charged by the consignor and the consignee. It
should be noted that the freight out paid by the consignor is applicable to the entire
goods delivered; thus, an allocation is needed to determine the amount applicable to
the number of units sold.

Items 11 and 12 are based on the following information

On January 1, 2021, Russell Electrical Shop received from Ray Trading 300

pieces of bread toasters. Russell was to sell those on consignment at 50% above

cost, for a 15% commission on the selling price. After selling 200 pieces, Russell

had the remaining unsold units repaired for some electrical defects for which

he spent P2,000 for the repair of 100 units. Ray subsequently increased the

selling price of the remaining units to P330 per unit.


On January 31, 2021, Russell remitted P64,980 to Ray after deducting the 15%

commission, P850 for delivery expenses of sold units, and P2,000 for the repair

of 100 units. The consigned goods cost Ray Trading P200 per unit, and P900

had been paid to ship them to Russell Electrical Shop. All expenses in

connection with the consignment were reimbursable to the consignee.

11. The consignment profit on the units sold was:

a. 13,000
b. 12,200
c. 12,880
d. Answer not given

Sales [(P64,980 +850+2000)/85%] P79,800

Less: Costs and expenses:

Cost of sold units


200
Old price @300 units
New price @330 [P79,800-(200 x P300)]/P330 60
units
Total sold units
260 P52,00
Multiply by unit cost units 0
Shipping cost (260/300 x P900) P200 780
Repair (P2,000 x 60/100) 1,200
Delivery expenses 850
Commission (15% x P79,800) 66,800
11,970
Consignment profit
P13,000
12. The value of inventory on consignment was:

a. 8,920
b. 8,920
c. 8,800
d. Answer not given

Cost of unsold goods (P200 x 40) P8,000

Shipping cost (P900 x 40/300) 120

Repair (P2,000 x 40/100) 800

Value of inventory on consignment P8,920

Items 13 and 14 are based on the following information

On December 1, 2021, Panpan Inc. consigned 60 VCD players to Ice bear Ltd.

The VCD player cost P140. Freight on the shipment paid by Ice bear Ltd. was

320. On December 30, Panpan Inc. received an account sales and P15,300 from

Ice bear Ltd. 30 VCD players had been sold and the additional expenses were

deducted:

Commission (20%)...................................................???
Advertising................................................................P 390
Delivery.....................................................................P 210

13. The total sales price of the VCDs sold by Ice bear
Ltd. was? a. 21,675
b. 14, 425
c. 20,257
d. P20,275
Sales - [(Sales x 20%) - P320 - P390 - P210] = P15,300
.80 of Sales = P15,300 + P320 + P390 + P210
.80 of Sales/.80 = P16,220/.80
Sales = P20,275

14. The inventory of VCDs will be reported on whose balance sheet and at

what amount?

a. Panpan Inc., P4,520


b. Panpan Inc., P4,360
c. Ice bear Ltd.,P4,200
d. Ice bear Ltd.,P4,360

Unsold players 30 Players P140 P4,200 P160


Multiplied by: Cost of VCD player Cost of Unsold Players
Freight attributable to unsold players [(320/60) xP4,360
30 Players]
Inventory balance

This inventory balance will be reported under the balance sheet of the consignor which is Panpan

Items 15 to 17 are based on the following information:

Paw Patrol Company consigned five sacks of dog food, with the cost of P800

each, to the Cat-tastrophy Company which was to sell these goods for the

account and ink of the former commission of 15% of selling price. The Paw

Patrol Company paid shipping costs of P200 on the shipment. Correspondingly,

Cat-tastrophy Company paid P320 on the freight of the shipment.


On the last day of the year, Cat-tastrophy Company reported that it had sold

three sacks of dog food, two for cash at P1,500 each and one on credit at

P1,800, of which 25% was collected as down payment. Cat-tastrophy remitted all

the cash due.

15. The amount of cash remitted by Cat-tastrophy Company is?

a. P2,410
b. P3,450
c. P3,130
d. P3,000

Collections:
Cash sale (1,500 x 2) P3,000
Credit (Down payment) (1,800 x 25%) 450
Total P3,450
Less: Expenses
Freight 320
Commission (4,800 x 15%) 720
Amount Remitted P2,410

16. The consignment profit

(loss) is? a. P3,432


b. P4,800
c. P1,368
d. P2,280

17. The amount of inventory on consignment of Paw Patrol

Company? a. P1,600
b. P1,680
c. P1,808
d. P1,728
Charges related to

Total Charges Consignment Inventory on


Sales Consignment

Consignor’s charges:

Cost P 4,000 P2,400 P1,600

Freight 200 120 80

Consignee’s charges:

Cost 320 192 128

Freight 720 720

Total P5,240 P3,432 17. ) P1,808

Sales Prices P4,800

Consignment Sales (3,432)

Profit 16. ) P1,368


Items 18 and 19 are based on the following information:

Information relating to the regular sales and consignment sales of The Ordinary

products for the year ended June 30, 2021 follows:

Regular Consignment Total


Sales Sales

Sales P120,000 P30,000 P150,000

Cost of Sales P84,000 P26,000 P110,000

Operating expenses ??? P1,760 P16,910

The merchandise costing P6,500 are in the possession of consignees and are

included in the cost of consigned merchandise sold. Operating expenses of

p15,150 (more than half of which are fixed) are to be allocated to regular sales

and to consignment sales on the basis of the volume. The P1,760 operating

expenses relating to consignment sales include a commission of 5% and P260

costs incurred by consignee relating to the entire shipment of merchandise worth

P26,000.

18. The net income on regular sales is?

a. P36,000
b. P32,550
c. P29,520
d. P23,880
19. The net income on the consignment sales is?

a. P10,500
b. P7,050
c. P7,500
d. P5,775

Regular Consignment Total


Sales Sales

Sales P120,000 P30,000 P150,000

Cost of 84,000 26,000 110,000

Sales Gross P36,000 P10,500 P46,500

profit

Operating expenses:

Commission (300K x 5%) 1,500 1,500

Freight-in [260 x (19,500*/26,000)] 1,950 1,950

Others

Regular [15150 x (19,500/26,000) 12,120

Consignment [15150 x (30K/250K) 3,030 3,030

Net Profit 18. ) P23,880 19. ) P5,775 P29,655


*Cost incurred by consignee

P26,000 Unsold merchandises included

In cost of consigned merchandise sold (6,500)

P19,500

For item 20 and 21:

On October 1,, 2021, the 13th Street Company consigned 100,000 Taylor Swif

folklore album in vinyl to Target, a mall. Each vinyl had a cost of P150.00. Freight

on the shipment was paid by the 13th Street Company for P200.00. On

December 31,2021, Target submitted an account sales stating that it had sold 60

pieces of the vinyl records and it was remitting the P12,840.00 balance due. The

remittance was net of the following deductions from the sales price of the vinyls

sold:

Commissions(20% of Sales Price) ?


Advertising Expense P500.00
Delivery and Installation Charges P100.00

20. What was the total sales price of the vinyl records sold by

Target? a. P13,440
b. P15,000
c. P16,800
d. P17,000
Solution: Sales
x
Advertising
Less charges and expenses: P500
Delivery and Intsallation Charges 100
Commission(20% OF SP) 0.20x
Remittance P12,840

x - (500 +100 + 0.20x) = 12,840


x - 0.20x = 12,840 + 600
0.80x = 13,440
x = 16,800
To check:
Sales P16,800
Less charges and expenses:
Advertising P500
Delivery and Installation Charges 100
Commission(20% OF SP) 3,360 3,960
Remittance P12,840

21. What was the cost of inventory on consignment?

a. P6,000
b. P6,080
c. P6,280
d. P6,320

Solution:
Cost(P150 each unit x 40 units) Freight on Shipment(P200
P6,000
x 40/100) Cost of Inventory on Consignmen
80
P6,080
For item 22-23:

In September 2021, iBookstore consigned 3,200 books, costing P60 and retailing

for P100 each to gBookstore, debiting accounts receivable and crediting sales for

the retail sales price. Freight cost of P3,200 was debited to freight expense by the

consignor. On September 30,2021, iBookstore received from gBookstore the

amount of P142,020 in full settlement of the balance due, and accounts

receivable was credited for this amount. The consignee deducted a P20

commission for each book sold, P180 for delivery, and 200 for advertising

expense.

22. How many books were actually sold by gBookstore?

a. 1,780
b. 1,778
c. 1,776
d. 1,424

Solution:
Remittance P142,020
Delivery 180
Advertising 200
Sales net of 20% commission P142,400*
Divide by 80
Units sold P1,780
*Divide by 80% to arrive at gross sales of P178,000.

To check:
Sales 178,000
Less charges and expenses: Delivery
Advertising Commission(20% OF SP) P180
Remittance 200
35,600 35,980
P142,020
23. What is the net profit of the consignor for the units sold?

a. P33,820
b. P33,280
c. P33,020
d. P33,080

Solution: Remittance
Less: Freight cost(1780 units) Cost(1,780 x 60) P142,020
Net profit P 1,780
106,800108,580
P33,820

For Item 24-25:

On November 2,2021, Tiltage Stage Appliance Center shipped five(5) of its

appliances to the Absidi Store on consignment. Each unit is to be sold at P25,000

payable P5,000 in the month of purchase and P1,000 per month thereafter. The

consignee is to be entitled to 20% of all amounts collected on consignment sales.

AbsiDi Store sold three units in November and one on December. Regular

monthly collections are made to the consignor at the end of each month. The

cost of the appliances shipped by the consignor was P15,500 per unit. The

consignor paid shipping costs to the consignee totalling P5,000.

24. What is the cost of the inventory on consignment on December 2021?

a. P15,500
b. P16,500
c. P19,600
d. P24,500
Solution:
Units shipped 5
Units sold (4)
Inventory, end 1

Cost P15,500
Shipping Cost 1,000
Toatl Cost P16,500

25. The profit on consignment for 2021 is:

a. P29,400
b. P15,500
c. P15,000
d. P14,000

Solution:
Sales on Consignment(4 x P25,000) P100,000
Less: COGS(15,500 X 4)P62,000
Shipping Cost(4 x 1000)4,000
Commission(20% of sales)20,00086,000 ProfitP14,000
HOME OFFICE AND BRANCH ACCOUNTING

THEORIES
1. In developing combined statements, the following are true, except:
a. Accounts for the home office and the branch need not be restated.
b. Combining branch and home office accounts results in those balances that
would have been obtained if one set of accounts had been maintained in
recording activities of both branch and the home office.
c. Any balance sheet accounts that report interoffice debits and credits have
no meaning when the related entities are recognized as one entity are eliminated.
d. The above statements are all correct.

Explanation
The affiliated entities are recognized as one entity. Accounts for the home office
and the ranch must be restated, so that, when combined, they will offer those
balances that would have resulted if the transactions of the related entities had
been recorded in one set of books.

2. Analyze the following statements.


I. The accounting records for branches may be centralized in the home office.
II. The accounting records for branches may be decentralized.

a. Statement I is correct. Statement II is incorrect.


b. Statement I is incorrect. Statement II is correct.
c. Statements I and II are both correct.
d. Statements I and II are both incorrect.

Explanation
The accounting records for branches may be centralized in the home office or
may be decentralized so that each branch maintains a complete set of accounting
records. If the accounting records are centralized in the home office, each branch
prepares daily reports and documents that are used as sources for journal entries
in the accounting records of the home office. If a branch maintains its own
accounting records, some transactions or events relating to the branch may be
recorded by the home office. Periodic financial statements are provided by the
branch to the home office so that combined statements may be prepared.

3. A branch is an organization that:

a. Is established to display merchandise.


b. Does not stock merchandise to fill customer’s orders or pass on
customer’s credit.
c. Has no separate accounting or business entity.
d. The statements above are all incorrect.
Explanation
All the given statements describe/define an agency.

4. Analyze the following statements.

I. At the end of an accounting period, the balance of the Investment in


Branch ledger account may not agree with the balance of the Home Office
account.
II. It is acceptable that the fiscal year for the home office does not coincide
with the fiscal year for the branch.

a. Statement I is correct. Statement II is incorrect.


b. Statement I is incorrect. Statement II is correct.
c. Statements I and II are both correct.
d. Statements I and II are both incorrect.

Explanation
At the end of an accounting period, the balance of the Investment in Branch
ledger account may not agree with the balance of the Home Office account. In
such cases the reciprocal ledger accounts must be reconciled and brought up to
date before combined financial statements are prepared.
The fiscal year for the home office must coincide with the fiscal year for the
branch to facilitate the preparation of combined financial statements.

5. As per AICPA SOP 98-5, start-up costs an entity undertakes when it introduces
a new product or service, conducts business in a new territory or with a new class
of customer or beneficiary, initiates a new process in an existing facility or
commences some new operation:
a. Must be expensed as the entity incurs them.
b. Must be deferred as charges.
c. Either A or B.
d. Neither A nor B.

Explanation
As per AICPA SOP 98-5, start-up costs an entity undertakes when it introduces a
new product or service, conducts business in a new territory or with a new class of
customer or beneficiary, initiates a new process in an existing facility or
commences some new operation must be expensed as the entity incurs them.

6. When billing at retail sales price, which of the following is incorrect?


a. Branch statements may be prepared and submitted to the home office.
b. If the branch is billed for goods at the sales price, the branch cost of goods
sold will not be equal to sales, and branch activities will show a loss from
operations equal to the expenses of operation.
c. The home office, when informed of branch sales currently, is provided with
a continuous record of the goods in the hands of the branch.
d. The statements above are all true.
Explanation
If the branch is billed for goods at the sales price, the branch cost of goods sold
will be equal to sales, and branch activities will show a loss from operations equal
to the expenses of operation. Branch accounts may be adjusted and closed in the
usual manner at the end of the fiscal period, and the home office account debited
for the reported loss.

7. In accounting for branch transactions, it is improper for the home office to:
a. Credit cash received from a branch to the Investment in Branch ledger account.
b. Maintain Common Stock and Retained Earnings ledger accounts for
only the home office.
c. Debit shipments of merchandise to the branch from the home office to the
Investment in Branch ledger account.
d. Credit shipments of merchandise to the branch to the Sales ledger account.

Explanation
Shipments of merchandise to the branch under the periodic inventory system are
recorded (if at cost) as debit to Investment in Branch and credit to Shipment to
Branch in the home office books while they are recorded as a debit to Shipment
from Home Office and credit to Home Office Equity in the books of the branch.
Shipments of merchandise to the branch under the perpetual inventory system
are recorded in the books of the home office as a debit to Investment in Branch
and a credit to Inventories. While in the books of the branch, they are recorded as
a debit to Inventories and a credit to Home Office, considering the transfer is at
cost.
8. When a home office ships merchandise to Branch A which is later shipped to
Branch B, the additional freight charged to ship the merchandise form Branch A to
Branch B should:

a. Treated as an expense on the Home Office books


b. Included as part of the cost of merchandise to Branch A
c. Included as part of the cost of merchandise to Branch B
d. Both B and C are correct

Explanation
Excess freight costs are recognized as expenses of the home office.
Excessive freight charges represent management mistakes or inefficiencies.
Therefore, they are not considered normal operating or freight expenses.
9. For a home office that uses the periodic inventory system of accounting for
shipments of merchandise to the branch, the credit balance of the Shipments to
Branch ledger account is displayed in the home office separate:
a. Income statement as an offset to purchase
b. Balance sheet as an offset to Investment in Branch
c. Balance sheet as an offset to inventories
d. Income statement as revenue.
Explanation
In its separate income statement, the home office displays the Shipment to
Branch ledger account as an offset to the net purchases to come up with the total
cost of goods available for its own sale.
10. A branch journal entry debiting Home Office and crediting Cash may be
prepared for:

a. The branch’s transmittal of cash to the Home Office


b. The branch’s acquisition for cash of plant assets to be carried in the home
office accounting records only
c. Either (a) or (b)
d. Neither (a) nor (b)

Explanation
Home Office Ledger Account is an account used by the branch to account for all
transactions with the home office. It is credited for all cash, merchandise or other
assets provided by the home office to the branch. It is debited for all cash,
merchandise, or other assets sent by the branch to the home office or to other
branches.

If a plant asset is acquired by a branch for its usage but the accounting record for
this plant asset is maintained by the home office, the accounting treatments are:
For the branch: debit Home Office, and credit cash or a liability account.
For the home office: debit a plant asset account: branch, and credit Investment in
Branch account.

11. In a working paper for combined financial statements of the home office and
the branch of a business enterprise, an elimination that debits Shipments to
Branch and credits Shipments from Home Office is required under:

a. The periodic inventory system only


b. The perpetual inventory system only
c. Both the perpetual inventory system and the periodic inventory system
d. Neither the perpetual inventory nor the periodic inventory system

Explanation
Under the perpetual inventory system, the ledger accounts “Shipment to Branch”
and “Shipment from Home” are not used. The shipment of merchandise to
branch, for instance, is recorded as debit to Investment in Branch and credit to
Inventories in the book of home office, and debit to Inventories and credit to Home
Office in the book of the branch.
Under periodic inventory system, on the other hand, in a working paper for
combined financial statements of the home office and the branch, the balance of
the Shipment to Branch ledger account is eliminated against the balance of the
Shipment from Home account together with the credit balance of the Allowance
for Overvaluation of Inventories account if there is any.

12. A journal entry debiting Cash in Transit and crediting Investment in


Branch is required for:
a. The home office to record the mailing of a check to the branch early
in the accounting period.
b. The branch to record the mailing of a check to the home office early
in the accounting period.
c. The home office to record the mailing of a check by the branch on the last
day of the accounting period.
d. The branch to record the mailing of a check to the home office on the last
day of the accounting period.

Explanation
Investment in Branch Ledger Account is a reciprocal ledger account to Home
Office account of a branch used by the home office to account for any
transactions with the branches. It is debited for cash, merchandise and services
provided to the branch by the home office and for the net income reported by the
branch. Thus, a debit in cash and a credit in Investment in Branch account
may be a home office entry for a check mailed by the branch, cash in transit if
the check that has been sent by the head office are still in transit, say on the
last day of the reporting period.

13. A Home Office‘s Allowance for Overvaluation of Inventories: Branch


ledger account, which has a credit balance, is

a. an asset valuation account


b. an equity account
c. a liability account
d. a revenue account

Explanation
The home office reports the credit balance of the Allowance for Overvaluation of
Inventories: Branch ledger account, in its separate balance sheet as a valuation
account of the Branch Current of the Investment in Branch ledger account.

14. Which of the following generally is not a method of billing merchandise


shipments by a home office to the branch?
a. Billing at cost
b. Billing at a percentage above cost
c. Billing at a percentage below cost
d. Billing at retail selling price

Explanation
Three alternative methods are available to the home office in billing the
merchandise shipped to the branches:
a. billed at the home office cost,
b. billed at a percentage above the home office cost, and
c. billed at the branch’s retail selling price

15. The appropriate journal entry for the home office to recognize the branch’s
expenditure of P10,000 for equipment to be carried in the home office accounting
records is:
a. Equipment 10,000
Investment in Branch 10,000
b. Home Office 10,000
Equipment 10,000

c. Investment in branch 10,000


Cash 10,000

d. Equipment-Branch 10,000
Investment in Branch 10,000

Explanation
If a plant asset is acquired by a branch for its usage but the accounting record
for this plant asset is maintained by the home office, the accounting treatments
are:
For the branch: debit Home Office, and credit cash or a liability account.
For the home office: debit a plant asset account: branch, and credit
Investment in Branch account.

16. Does the branch use a Shipments from Home Office ledger account
under the: Perpetual Inventory Periodic Inventory
Method Method
a. Yes Yes
b. Yes No
c. No Yes
d. No No
Explanation
When a periodic inventory system is adopted, inventory account cannot be
used for the shipments of merchandise between the home office and the branch.
Thus, accounts such as “Shipments to Branch” (used by the home office) and
“Shipments from Home Office” (used by the branch) are used.
17. If the home office maintains in its general ledger accounts for a branch’s
plant assets, the branch debits its acquisition of office equipment to:
a. Home Office
b. Office Equipment
c. Payable to Home Office
d. Office equipment carried by home office

Explanation
If a plant asset is acquired by a branch for its usage but the accounting record for
this plant asset is maintained by the home office, the accounting treatments are:
For the branch: debit Home Office, and credit cash or a liability account.
For the home office: debit a plant asset account: branch, and credit Investment in
Branch account.
18. May be Investment in Branch account of a home office be accounted for by the

Cost Method Equity Method


of accounting of accounting
a. Yes Yes
b. Yes No
c. No Yes
d. No No

Explanation
The home office keeps a reciprocal account called Branch Current or
Investment in Branch. This noncurrent asset account is debited for cash, goods,
or services to the branch; and for branch income. Conversely, the account is
credited for remittances from the branch or other assets received from the branch;
and for branch losses. Thus, the Investment in Branch account reflects the
equity method of accounting.
19. If Jibs Branch ships merchandise with a cost of $400 to Tibs Branch
and the periodic inventory system is used, the following journal entries are
required except:
a. Home office 400
Shipments from Home Office 400
b. Shipments from Home Office 400
Home Office 400

c. Investment in Tibs Branch 400


Investment in Jibs Branch 400

d. All are correct.

Explanation
Letter A are accounting records of Jibs
Branch. Letter B are accounting records
of Tibs Branch. Letter C are accounting
records of home office.

20. The following are limitations of branch accounting except:


a. There are certain expenses which are incurred for the organization
as a whole but cannot be attributable to the branches.
b. Inter branch and intra branch comparison can be done.
c. Separate accounts for each branch are maintained which increases
the accounting charges.
d. In case of foreign branch, conversion of foreign currency into
domestic currency cannot be properly done due to regular fluctuations
in exchange rate.

Explanation
Inter branch and intra branch comparison is an advantage of branch
accounting to assess the performance of each branch.
21. Among the interoffice transactions in the accounting records of the home
office of Sand Company was the following:
Investment in Box Branch 10,000
Shipment to Box Branch
10,000
This journal entry indicates:
a. A transfer of merchandise from home office at cost.
b. A payment by home office of branch expenses.
c. A transfer of merchandise from home office at above cost.
d. A transfer of cash from the home office.
Explanation
The correct journal entries for letters B, C, and D respectively are the
following: Investment in Box Branch xx
Cash xx

Investment in Box Branch xx


Allowance for overvaluation xx
Shipment to Box Branch xx

Investment in Box Branch xx


Cash xx

22. The home office ledger account in the accounting records of a branch is
best described as as:
a. A revenue account
b. An equity account
c. A deferred revenue account
d. None of the foregoing

Explanation
The Home Office Ledger account is quasi-ownership account equity that shows
the net investment by the home office in the branch. It indicates the extent of the
accountability of the branch to the home office.

23. If both the home office and the branch of a business enterprise use
perpetual inventory system, a Shipment to branch ledger account appears in
the accounting records of:
a. The home office only
b. The branch only
c. Both the home office and the branch
d. Neither the home nor the branch

Explanation
When a merchandise is transferred to the branch the home office debits the
branch account and credits Shipments to Branch, on the other hand, the branch
debits Shipments from Home Office. However, when both uses perpetual
inventory system, inventory accounts are being used, the home office will credit
the appropriate inventory account and the branch will debit the appropriate
inventory account.

24. In preparing the financial statements of the home office and its various branches:
a. Nonreciprocal accounts are eliminated but reciprocal
accounts are combines
b. Both reciprocal and nonreciprocal accounts are eliminated
c. Both reciprocal and nonreciprocal accounts are combines
d. Reciprocal accounts are eliminated and nonreciprocal accounts
are combined
Explanation
The process of combining home office and branch financial statement is similar to
the process of consolidating the parent and subsidiary statements. Reciprocity is
established between home office and branch records by eliminating reciprocal
accounts and combining nonreciprocal accounts. We eliminate unrealized profits
from internal transfers between the home office and the branches in preparing
combined financial statements for the enterprise.

25. Which of the following accounts is a reciprocal account to the


Investment in Branch account?
a. Branch Income c. Home Office Capital
b. Equity in Home Office d. None of the above

Explanation
Investment in Branch Ledger Account this account is a reciprocal ledger account
(to Home Office account) used by the home office to account for any transactions
with the branches. It is debited for cash, merchandise and services provided to
the branch by the home office and for the net income reported by the branch.
PROBLEMS

On November 2, 2015, Luffy Company created an agency in Davao, and


transferred merchandise samples costing P15,000, equipment worth P36,000 and
a cash working fund of P10,000 to be maintained on an imprest basis. The
agency transmitted to the home office sales orders which were billed at P90,000
of which P46,550 was collected net of a 2% discount. The agency also paid for its
operating expenses of P7,500 including supplies of P2,000. The agency received
replenishment thereof from the home office before the year ended. On December
31, the agency samples were estimated to be useful over a period of 5 months
while the equipment is estimated to have a useful life of eight years. Unused
supplies on Dec 31 amounted to P1,000. Home office maintains a gross profit
rate of 25% of cost.

1. How much is agency profit at the end of


December? a. P3,800 c) P4,800
b. P13,800 d) P14,800

2. Give the agency real accounts that will be presented in the Dec. 31
balance sheet:

Agency Samples:
a. P7,000 c)P9,000
b. P8,000 d)3,400

3. Working Fund, Davao


Agency a.
P10,000
c)P12,000
b. P7,000 d)P13,000

4. Supplies, Davao Agency


a. P200 c) P1,000
b. P1,200 d) P2,000

5. Equipment, Davao Agency


a. P36,000 c) P38,550
b. P35,250 d)P36,750

6. Give the cost of sales of Home Office if its Inventory on Nov 2 is


P1,250,000 while on hand at Dec 31 went down to 20% of total stock . Net
purchases was P2,750,000 including freight of P50,000.
a. P3,000,000 c) P3,113,000
b. P3,131,000 d) P3,010,300

a.) Income and Summary Expense b.) Real Accounts


Samples 6,000 Sales 90,000 (2)Agency Samples P 9,000
Used 1,000 (3)Working Fund, 10,000
Supplies Davao
Agency
Cost of Sales 72,000 (4)Supplies, 1,000
Davao Agency

Depreciation 750 (5)Equipment, P 36,000


Davao Agency
Other expenses 5,500 Accumulate ( 750)
d
Depreciatio
n
Sales discount 950 P 35,250
Balance (1) 3,800
c.) Home Office Cost of Sales
Inventory, Nov 2 P1,250,00
Net 0
Purchases 2,700,00
Freight In 0
50,000
Shipments ( 87,000
Inventory, Dec )
31
(6) Cost of sales (800,000)
P3,113,00
0
Trafalgar Corporation shows the following balance sheet accounts as at Jan 1, 2015:
Cash P 476,500 Accumulated Depreciation P 93,750
Accounts Receivable 1,050,000 Accrued
Expenses 7,250
Inventory 1,300,000 Accounts Payable 743,750
Furniture & Fixtures 750,000 Capital
Stock 2,500,000
Allowance for Bad Debts 31,500 Retained
Earnings 263,250

On this date, management decided to establish a branch in Baguio and reported


the following transactions for the first quarter of 2015:
I. Transferred cash P150,000, merchandise P500,000 with freight
prepaid based on 2% of its shipment cost.
II. Home office approved on Feb 1, the purchase by the branch of its furniture
and fixtures costing P75,000 for its own use. Home office policy was to maintain
and control all fixed assets.
III.The branch was authorized to take over P60,000 home office accounts from its
Baguio customers and make the necessary collections. Home Office issued a
debit memo for this.
IV. Summary of account sales and (collections) for the quarter : Home Office-
P1,865,000 and (1,499,400 net of a 2% discount) and for the Branch- P655,000
(P565,000).
V.Summary of account purchases and (payments): Home Office-P790,000 and
(P745,000) and for the Branch- P150,000 and (P88,755 net of a 3% discount).
VI. Operating Expenses paid including accrued expenses, if any- Home Office,
P230,000 and Branch, P 131,250.
VII. Returns from the original shipment amounted to P75,000. Freight on
returns was paid by the branch. Allocated freight and freight paid by branch were
all charged to home office as a loss account.
VIII.From the Baguio accounts turned over in c), collections were made accordingly
less a discount of 2% which the branch charged to the home office.
IX. P11,875 of the expenses paid by home office in f) were charged to the branch.
X. Cash remittance was made by the branch in excess of original cash transfer.
XI. Depreciation was recorded at a rate of 12%.
XII. Unpaid utility bills: P67,500 for home office and P18,500 for the branch.
XIII. Policy on doubtful accounts was to be maintained based on the previous rate.

1. The entry to record transaction I in the books of Home


Office a. Cash 150,000
Freight In

10,000 Shipments from H.O.

500,000
Home office Equity 660,000

b. Investment in branch 650,000


Cash 150,000
Shipments to Branch 500,000

c. Cash 150,000
Shipments from H.O.
510,000
Home office Equity 660,000

d. Investment in branch

660,000 Cash

160,000
Shipments to Branch 500,000

2. The entry to record transaction IV in the Branch books


a. Accounts Receivable 655,000
Sales 655,000

b. Accounts receivable

1,865,000 Sales

1,865,000

c. Accounts Receivable

565,000 Sales

565,000

d. Accounts Receivable

1,449,200 Sales

1,449,200

3. The entry to record transaction XIII in the books of Home Office


a. Bad Debts 2,700
Allowance for Bad Debts 2,700 90,000 x .03
b. Bad Debts 8,250
Allowance for Bad Debts 8,250 (1,325,000x.03=39,750-31,500)

c. Bad Debts 7,293.75


Allowance for Bad Debts

7,293.75 (1,293,125 x.03=38,793.75-


31,500)

d. Bad Debts 38,793.75


Allowance for Bad Debts

38,793.75 (1,293,125 x.03=38,793.75)

4. How much is the Plant Assets of the Home


Office? a. P825,000 c) P708,750
b. P822,750 d) P706,500

5. How much is the Home Office Net


Sales? a. P1,833,200
c) P1,338,200
b. P2,488,200 d) P2,884,200

6. How much is the Branch Net


Sales? a. P565,000
c) P1,865,000
b. P655,000 d) P1,449,200

7. How much is the Combined Net


Sales? a. P1,833,200
c) P1,338,200
b. P2,488,200 d) P2,884,200

8. How much is the Home Office Cost of


Sales? a. P879,500 c)
P897,500
b. P875,900 d)P895,700

9. How much is the Branch Cost of


Sales? a. P455,000 c)
P450,005
b. P450,000 d) P405,050

10. How much is the Combined Cost of


Sales? a. P1,392,550
c) P1,352,505
b. P1,330,900 d) P1,329,505

11. What is the Home Office Gross


Profit? a.
P1,158,695 c)
P240,950
b. P953,700 d) P1,150,690
12. How much is the Branch Gross
Profit? a. P204,995
c) P209,450
b. P209,445 d) 240, 950

13. How much is the Combined Gross


Profit? a. P953,700 c)
P975,300
b. P1,158,695 d) P1,555,200

14. How much is the Home Office Profit/loss from


Operations? a. P644,575 c) P567, 445
b. P654,447 d) P674,575

15. How much is the Branch Profit/loss from


Operations? a. P38,420 c) P32,480
b. P34,800 d) P38,400

16. How much is the Combined Profit/loss from


Operations? a. P689,295 c) P692,558
b. P682,995 d)P628,995

17. How much is the Home Office


Net Profit a. P679,985 c)
P682,995
b. P687,995 d) P679,995

18. How much is the Branch Net


Profit? a. P38,400
c) P32,480
b. P34,800 d) P38,420

19. How much is the Combined Net


Profit? a. P679,995
c) P682,995
b. P687,995 d) P689,225

PLANTS AND ASSETS


#(4)
Furniture &Fixtures 750,000
Acc. Depreciation 116,250 633,750
Furniture & Fixtures-Baguio 75,000
Acc. Depreciation 2,250 72,750
Total P706,500
INCOME
STATEMENT HOME OFFICE BRANCH COMBINED
Sales P1,865,000 P655,000 P2,520,000
Less Discounts, ( 31,800) ( 31,800)
Returns
and
Allowances
Net Sales (5) P1,833,200 (6)P655,000 (7)P2,488,200
Cost of Sales:
Inventor 1,300,000 1,300,000
y,
Beginning
Net Purchases 790,000 147,255 937,255
Shipments to (425,000) -
Branch
Freight In 8,500 8,500
Shipments 425,000
from Home
Office
Inventor (785,500) (8)879, 500 (130,750) (9)450,005 (916,250 (10)1,329,505
y, Ending )
Gross Profit (11) P953,700 (12)P204,995 (13)P1,158,695
Less
Operating
Expenses:
Bad Debts 8,250 2,700 10,950
Depreciatio 22,500 2,250 24,750
n
Utilit 67,500 18,500 86,000
y
Othe 210,875 309,125 143,125 166,575 354,000 475,700
r operating
Expenses
Net Profit from (14) P644,575 (15) P38,420 (16)P 682,995
Operation
Branch Profit 38,420
Loss on Branch (3,000) (3,000)
Transfers
Net Profit (Loss) (17)P679,995 (18)P38,420 (19)P 679,995

You might also like